0% found this document useful (0 votes)
15 views135 pages

Capstone IBA Model

The document is a mathematics pre-mock test consisting of 50 questions covering various topics such as ratios, percentages, age problems, and grammar. Each question has multiple-choice answers, and students are instructed not to use calculators. The test is designed to assess students' mathematical and language skills within a one-hour time frame.

Uploaded by

Åvîjìt
Copyright
© © All Rights Reserved
We take content rights seriously. If you suspect this is your content, claim it here.
Available Formats
Download as PDF, TXT or read online on Scribd
0% found this document useful (0 votes)
15 views135 pages

Capstone IBA Model

The document is a mathematics pre-mock test consisting of 50 questions covering various topics such as ratios, percentages, age problems, and grammar. Each question has multiple-choice answers, and students are instructed not to use calculators. The test is designed to assess students' mathematical and language skills within a one-hour time frame.

Uploaded by

Åvîjìt
Copyright
© © All Rights Reserved
We take content rights seriously. If you suspect this is your content, claim it here.
Available Formats
Download as PDF, TXT or read online on Scribd
You are on page 1/ 135

CAPSTONE Pre-Mock Test-I

Time: 1 hour Marks: 50


Mathematics
Instructions: Solve each of the following problems and mark the conect answer on your Answer Sheet.
DO NOT USE A CALCULATOR. Figures are not drawn to scale.

1. BDT. 2250 is divided among three friends Raju, Anik, and Shohag in such a way that —6 th
of Raju’s

share, —1 th of Anik’s share and — 111 of Shohag's share is equal. Find Raju's share.
A. BDT. 1080 B. BDT. 720 C. BDT. 450 D. BDT. 1240 E. None of these
2. A dishonest merchant sells his grocery using weights 15% less than the true weights and makes a profit
of 20%. Find his total gain percentage.
A. 35% B. 47.34% C. 41.17% D. 32.75% E. None of these
3 . Which of the following describes all values of ‘x’ for which 1 - x2 0 ?
A. xs-l B. x≤-lorx≥l C. x 1 D. -l<x≤l E. None of these
4. Sadib, Tazul and Abir invested Tk. 8000, Tk. 4000 and Tk. 8000 respectively in a business. Sadib left
after six months. If after eight months, there was a gain of Tk. 4005, then what will be the share of
Tazul?
A. 980 B. 1335 C. 1700 D. 890 E. None of these
5. a, b, c, d, e are five consecutive numbers in increasing order of size. Deleting one of the five numbers
from the set decreased the sum of the remaining numbers in the set by 20% of the sum of all five
numbers. Which one of the numbers was deleted from a, b, c, d and e?
A. a B. b C. c D. d E. e
6. Two alloys A and B are composed of two basic elements. The ratios of the composition of two basic
elements in the two alloys are 5:3 and 1:2 respectively. A new alloy ‘X’ is formed by mixing the two
alloys A and B in the ratio 4:3. What is the ratio of the composition of the two basic elements in alloy X?
A. 2:1 B. 1:1 C. 2:3 D. 3:2 E. 7:2
7. A frain leaves Chittagong at 6 a.m. and reaches Sylhet at 10 a.m. Another train leaves Sylhet at 8 a.m.
and reaches Chittagong at 11:30 a.m. At what time do the 2 trains cross one another?
A. 8:36 B. 10:56 C. 8:56 D. 10:36 E. 9:56
8. A machine P can print one lakli books in 8 hours; machine Q can print the same number of books in 10
hours while machine R can print them in 12 hours. All the machines are stalled at 9 A.M. while machine
P is closed at 11 A.M. and the remaining two machines complete work. Approximately at what time will
the work (to print one lakli books) be finished?
A. 1:00 PM B. 1:30 PM C. 12:30 PM D. 12:00 PM E. None of these
9. The age of the father of two children is twice that of the elder one added to four times that of the
youngest one. If the arithmetic mean and product of the ages of the two children is 8 years and 48 years
respectively, then what is the age of tire father?
A. 48 years B. 32 years C. 40 years D. 42 years E. None of these
https://www.fb.com/capstonebd [01972277866] Page: 1
1 0. Four equal circles each of diameter 1 foot touch at four points as shown in the figure below. What is the
area (in square feet) of the shaded portion?
A. 1— 7t B. 1-471 C. 71 L JL
)

1 1. How many degrees are there in an angle formed by the hands of a clock at 2:30?
A. 105 B. 100 C. 110 D. 115 E. None of these
12. Rayhan completed a race at a certain speed. Had he moved 3 kmph faster, he would have taken 40
minutes less. If he had moved 2 kmph slower, he would have taken 40 minutes more. The distance is-
A. 35 k.m. B. 36— k.m.
3
C. 37 —
2
k.m. D. 40 k.m. E. None of these

13. A boy can only take four chips from a bag where there are 6 Mr. Twist and 4 Potato crackers; in how
many different ways can they be selected such that at least one Mr. Twist should be there?
A. 159 B. 194 C. 205 D. 209 E. None of these
14. Kishore gets on the elevator at the 11th floor of a building and rides up at the rate of 57 floors per minute. At
tire same time, Shomov gets on an elevator at the 51st floor of the same building and rides down at the rate of
63 floors per minute. If they continue travelling at these rates, then at which floor will their paths cross?
A. 19 B.28 C. 30 D. 37 E. 40
15. A man rows to a place 36 Km distant and back in 10.5 hours. He finds that he can row 4 km with the
stream in the same time as 3 Km against the stream. The rate of the stream is:
A. 1 B. 1.8 C. 0.5 D. 1.5 E. 1.25
16. hi a survey conducted for three MFS providers, it is found that 60% of the respondents use bKash, 45%
use Nagad, 30% use Rocket and 10% use none of these. It is also found that 22% of the respondents use
both bKash and Nagad and 15% of the respondents use exactly two of the three MFS providers. What
percent of the respondent use only Rocket?
A. 7 B. 8 C. 9 D. 10 E. Cannot be determined
17. Consider four digit numbers for which the first two digits are equal and the last two digits are also equal.
How many such numbers are perfect squares?
A. 2 B.4 C. 1 D. 0 E. 3
18. A batsman scored 110 runs which included 3 boundaries and 8 sixes and took 39 runs by running
between the wickets. What percent of his total score did he make from overthrows??
A. 25% B. 15% C. 10% D. 11% E. None of these
19. Saumik, Pial, and Fahim together can finish a task in 12 days. Saumik is twice as productive as Pial and
Fahim alone can do the task in 36 days. In how many days can Saumik and Pial do the task if Fahim
goes on leave?
A. 15 B.20 C. 18 D. 17 E. None of these
20. A cloth merchant has announced 25% rebate in prices. If one needs to have a rebate of TK.40, then how
many shirts each costing Tk.32 he should purchase?
A. 6 B.5 C. 10 D. 7 E. None of these

21. How many different 3 digit numbers can be formed, such that 1st and 3rd place should be filled up with
odd numbers?
A. 400 B.250 C. 150 D. 120 E. None of these
https://www.fb.com/capstonebd [01972277866] Page: 2
22. If x is not a negative number, what is the maximum possible value of 22-3x?
A. 4 B. 3 C. 2 D. 1 E. None of these

23. Ricky sells 7 different sized balls. Each ball costs n taka more than the next one below it in size and the
price of the biggest ball is 72. If the sum of the prices of 12 different balls is tk. 336, what is the value of
n?
A. 6 B. 7 C. 8 D. 9 E. None of these

24. If x -1 and x x5 , which of the following must be tine?


A. x>l B. 0<x<l —
C. l≤x≤0 D. x>0 E. None of these

25. 1 year ago the ratio between Niaz & Alvie’s salary was 5:6. Ratios of their individual salaries between
last year’s and this year’s salaries are 3:4 & 2:3. At present the total of their salary is 18800. Tire salary
of Niaz now, is-
A. 2000 B. 1600 C. 2560 D. 4120 E. 8000

Language & Communication


Questions 26 to 30 are based on the following instruction:
Instructions: The sentences below contain errors in grammar, usage and idiom. Parts of each sentence are
underlined & lettered. Decide which underlined part contains error. If the sentences is correct
as it stands, mark (E) on your Answer Sheet.
26. Dhaka has received mediocre master plans that failed (A) to fathom (B) the power of the city and the
direction of its future; however (C), DAP 2022-35 is by far the most comprehensive and well-research
(D) proposition for Dhaka with a lot of new ideas. No error (E).

27. Blare tried to live in accordance with (A) the canons (B) of fairness, honesty, and (C) responsibility
that his parents laid (D) down for their children. No error (E).

28. Tire organization aims (A) to nearly double (B) its revenues on the back (C) of a strongest product
pipeline (D). No error (E).

29. Teaching in this (A) school would certainly improve (B) if it had less bovs (C) in all the high classes
(D). No error (E).

30. During the debate, Joe attended closely (A) to the Red Party candidate's strategic plan, which ioe (B)
thought was better structured (C) than the other candidates (D). No error (E).

Instruction: Choose the most appropriate word(s)/phrase to fill in the blanks in die following questions 31-35
31. The breathing spell provided by the—arms shipments should give all the combatants a
chance to reevaluate their positions.
A. plethora of B. moratorium on C. reciprocation of
D. concentration on E. development of

32. Because they had expected die spacecraft Voyager 2to be able to gather data only about the planets
Jupiter and Saturn, scientists were the wealth of information it sent back from Neptune twelve
years after leaving Earth.
A. disappointed in B. concerned about C. confident in D. elated by E. anxious for
https://www.fb.com/capstonebd [01972277866] Page: 3
33. Despite assorted effusions to the contrary, there is no necessary link between scientific skill and
humanism, and quite possibly, there may be something of a between them.
A. generality B. dichotomy C. congiuity D. fusion E. reciprocity

34. Criticism is dangerous, because it wounds a person’s precious pride, hurts his sense of importance, and
arouses
A. Pugnacity B. Resentment C. Malice D. Ecstasy E. Bellicose

35. As much as we thirst for approval, we condemnation.


A. Dread B. Doubt C. Peril D. Wag E. Squander

Instruction: Answer the questions 36-40 based on the following words-


i. industrial working class v. To liberate
ii. Greedy vi. Stinking
iii. Noxious vii. Generous
iv. Avaricious viii. The art of public speaking
36. Which of the following options expresses the meaning of the word- ’Rapacious’?
A. vi B.vii C. ii & iv D. ii E. ii & vi

37. Which of the following options expresses the meaning of the word- ‘Proletariat’?
A. v B. vi C. i D. viii E. vii

38. Which of the following options expresses the meaning of the word- ‘Noisome’?
A. ii B. vi C. iii D. iii & vi E. viii

39. Which of the following options expresses the meaning of the word- ‘Munificent’?
A. i B. v C. vi D. vii E. viii

40. Which of the following options expresses the meaning of the word- ‘Emancipate’?
A. i B. iii C. v D. vii E. viii

Questions 41 to 45 are based on the following instruction:


Instructions: hr each of the following sentences, part or all of the sentence is underlined. The answer¬
choices offer five ways of phasing the underlined part. If you think the sentence as written is
better than the alternatives, choose A, which merely repeats the underlined part; otherwise
choose one of the alternatives.
41. The economy is giowing only at roughly 4 per cent a year and the unemployment rate is about 10 per
cent, although demand of cars, motorcycles and other consumer goods is booming.
A. Although demand of cars, motorcycles and other consumer goods is booming
B. Although demand for cars, motorcycles and other consumer goods are booming.
C. Although demand for cars, motorcycles and other consumer goods is booming.
D. Because demand for cars, motorcycles and other consumer goods have been booming.
E. Although demand for cars, motorcycles and other consumer goods is boomed.
https://www.fb.com/capstonebd [01972277866] Page: 4
42. The incident happened in 1532. Francisco Pizano and his troops anived in Cuzco, took hostage the
Incan King, Atahuallpa. and then they demanded ransom.
A. Atahuallpa, and then they demanded ransom.
B. Who was Atahuallpa, and then they demanded ransom.
C. Atahuallpa, and then demanded ransom.
D. Atahuallpa, and then there was a demand for ransom.
E. Atahuallpa, and then he demanded ransom.

43. Tire best known Iban textiles, large ceremonial cloths called “pua kumbu”, whose designs depict the
flora and fauna of Borneo as well as figures from the spirit realm.
A. textiles, large ceremonial cloths called “pua kumbu”, whose designs depict
B. textiles, large ceremonial cloths called “pua kumbu”, in whose designs are depicted
C. textiles are large ceremonial cloths called “pua kumbu”, whose designs depict
D. textiles are large ceremonial cloths called “pua kumbu”. their designs depict
E. textiles, large ceremonial cloths, are called “pua kumbu”, in their designs are depicted

44. The new inspector general's office in Iraq operates under most unique rules that greatly limit both its
powers and independence.
A. under most unique rules that greatly limit both its
B. under most unique rules that greatly limits both its
C. under unique rules that greatly limits both its
D. under most unique rules which greatly limit both its
E. under unique rules that greatly limit both its

45. Fifty years ago, movies on biblical themes, far from being the more controversial Hollywood offerings,
were among the least ones.
A. themes, far from being the more controversial Hollywood offerings, were among
B. themes, far from being the more controversial Hollywood offerings, was among
C. theme, far from being the most controversial Hollywood offerings, were among
D. themes, far from to be the more controversial Hollywood offerings, were among
E. themes, far from being the most controversial Hollywood offerings, were among

Read the passage and answer the following questions. (Questions: 46-50)
Philosophy of Education is a label applied to the study of the purpose, process, nature and ideals of
education. It can be considered a branch of both philosophy and education. Education can be defined as
the teaching and learning of specific skills, and the imparting of knowledge, judgment and wisdom, and
is something broader than the societal institution of education we often speak of.
Many educationalists consider it a weak and woolly field, too far removed from the practical
applications of the real world to be usefill. But philosophers dating back to Plato and the Ancient Greeks
have given the area much thought and emphasis, and there is little doubt that their work has helped shape
the practice of education over the millennia.
Plato is the earliest impoilant educational thinker, and education is an essential element in "The
Republic" (his most important work on philosophy and political theory, written around 360 B.C.). hi it,
he advocates some rather extreme methods: removing children from their mothers' care and raising them
as wards of the state, and differentiating children suitable to the various castes, the highest receiving the
most education, so that they could act as guardians of the city and care for the less able. He believed that
https://www.fb.com/capstonebd [01972277866] Page: 5
education should be holistic, including facts, skills, physical discipline, music and art. Plato believed that
talent and intelligence is not distributed genetically and thus is be found in children bom to all classes,
although his proposed system of selective public education for an educated minority of the population
does not really follow a democratic model.
Aristotle considered human nature, habit and reason to be equally important forces to be cultivated in
education, the ultimate aim of which should be to produce good and virtuous citizens. He proposed that
teachers lead their students systematically, and that repetition be used as a key tool to develop good
habits, unlike Socrates' emphasis on questioning his listeners to bring out their own ideas. He
emphasized the balancing of the theoretical and practical aspects of subjects taught, among which he
explicitly mentions reading, writing, mathematics, music, physical education, literature, history, and a
wide range of sciences, as well as play, which he also considered important.
During the Medieval period, the idea of Perennialism was first formulated by St. Thomas Aquinas in
Iris work "De Magistro". Perennialism holds that one should teach those things deemed to be of
everlasting importance to all people everywhere, namely principles and reasoning, not just facts (which
are apt to change over time), and that one should teach first about people, not machines or techniques. It
was originally religious in nature, and it was only much later that a theory of secular perennialism
developed.
During the Renaissance, the French skeptic Michel de Montaigne (1533 - 1592) was one of the first
to critically look at education. Unusually for his time, Montaigne was willing to question the
conventional wisdom of the period, calling into question the whole edifice of the educational system,
and the implicit assumption that university-educated philosophers were necessarily wiser than
uneducated farm workers, for example.
46. What is the difference between the approaches of Socrates and Aristotle?
A. Aristotle felt the need for repetition to develop good habits in students; Socrates felt that students
need to be constantly questioned
B. Aristotle felt the need for rote-learning; Socrates emphasized on dialogic learning
C. There was no difference
D. Aristotle emphasized on the importance of paying attention to human nature; Socrates emphasized
upon science
E. None of these

47. Why do educationists consider philosophy a ‘weak and woolly’ field?


A. It is not practically applicable B. Its theoretical concepts are easily understood
C. It is irr elevant for education D. Philosophical abstractions are suitable for practical application
E. None of these

48. What do you understand by the term ‘Perennialism’, in the context of the given comprehension passage?
A. It refers to something which is of ceaseless importance
B. It refers to something which is quite unnecessary
C. It refers to something which is abstract and theoretical
D. It refers to something which existed in the past and no longer exists now
E. It refers to something that has become outdated

49. Were Plato’s beliefs about education democratic?


A. He believed that only the rich have the right to acquire education
B. Yes
C. He believed that only a select few are meant to attend schools
D. He believed that all pupils are not talented
E. None of these

50. Wiry did Aquinas propose a model of education which did not lay much emphasis on facts?
A. Facts are not important B. Facts do not lead to holistic education
C. Facts change with the changing times D. Facts are frozen in time
E. None of these
https://www.fb.com/capstonebd [01972277866] Page: 6
CAPSTONE Pre-Mock Test-II
Time: 1 hour Marks: 50
Mathematics
Instructions: Solve each of the following problems and mark the correct answer on your Answer Sheet.
DO NOT USE A CALCULATOR. Figures are not drawn to scale.
1. Every year before the festive season, a shopkeeper increases the price of the product by 35% and then
introduces two successive discounts of 10% and 15% respectively. What is percentage loss and
percentage gain?
A. 3.27% loss B. 3.27% gain C. 4.15% loss D. 4.15% gain E. None of these
2. Shahed can do a piece of work in 45 days, but Almaf can do die same work in 5 days less, than Shahed,
when working alone. Shahed and Almaf both started the work together but Almaf left after some days
and Shahed finished the remaining work in 56 days with half of his efficiency but he did the work with
Almaf with his complete efficiency. For how many days they had worked together?
A. 6 B. 8 C. 9 D. 12 E. None of these
3. Two horses start hotting towards each other, one from A to B and another from B to A. They cross each
other after one hour and the first horse reaches B, 5/6 hour before the second horse reaches A. If the
distance between A and B is 50 km. what is the speed of the slower horse?
A. 30 B. 15 C. 25 D. 20 E. None of these
4. A papaya is cut into two pieces as their weight are in ratio 5:7. The bigger part is also cut into two pieces
as their weight are in ratio 2:3. Calculate the ratio of weight of the final three pieces.
A. 25:9:18 B. 21:25:10 C. 25:14:21 D. 13:23:27 E. None of these
5. Of 48 applicants, 28 had at least 5 years' experience, 30 had MBA degree, and 10 had less than 5 years’
experience and no MBA degree. How many of the applicants had at least 5 years' experience and an
MBA degree?
A. 8 B. 10 C. 20 D. 24 E. None of these
6. Alavi tr avels 20% distance of the total journey by car and 50% of the remaining by train and taxi in the
respective ratio of 5:3 and the remaining distance he covers on feet. If the sum of the distance which he
travels by car and by Taxi is 126 km, then find the total distance which Alavi travels during his journey?
A. 220km B. 400km C. 360km D. 160km E. None of these
7. Two numbers A and B are such that the sum of 5% of A and 4% of B is two-third of the sum of 6% of A
and 8% of B. Find the ratio of A : B is
A. 3:4 B. 5:3 C. 2:3 D. 3:5 E. None of these
8. Shanta prepares a budget to visit New York. However, she spends 12% of her budget on the first 10% days of
her travel when she stays in the city. She knows that she has to spend another 35% of days in city itself, after
which she would travel to the countryside. What should be the minimum decrease in spending in country side
as a percentage of her spending in city so as to complete her travel on the initial budget itself?
A. 30.10% B. 33% C. 33.33% D. 40% E. None of these
9. hr a class, the number of boys is more than the number of girls by 12% of the total students. The ratio of
boys to girls is:
A. 11:14 B. 14:11 C. 25:28 D. 28:25 E. None of these
1 0. hi how many ways can six different rings be worn on four fingers of one hand?
A. 16 B. 360 C. 24 D. 4096 E. 8192
https://www.fb.com/capstonebd [01972277866] Page: 1
11 . Ill a stream miming at 2km/lir. a motor boat goes 1 0 km upstream and back again to the starting point in
55 minutes. Find the speed of the boat in still water.
A. 20 B. 21 C. 22 D. 23 E. None of these

12. Price of a hammer is twice that of a screwdriver. If price of a hammer is raised by 5% & the price of a
screwdriver is decreased by 4%, how much more or less will it cost to buy 3 screwdrivers & 3 hammers?
A. 4% more B. 2% less C. 2% more D. 4% less E. None of these
13. A bag contains 4 white, 4 red and 6 blue balls. Three balls are drawn at random from the bag. The
probability that all of them are red, is-
13 2 2
A. —
22
B.
22
— C.
91
— D.
77 — E. None of these

14. hi a mixture 60 litres, the ratio of milk and water 2:1. If this ratio is to be 1 :2, then the quantity of water
to be further added is:
A. 20 litres B. 30 litres C. 40 litres D. 60 litres E. None of these

15. Avik can complete a work in 12 days working 8 hows a day. Foyez can complete the same work in 8
days working 10 hours a day. If both of them work together, working 8 hours a day, in how many days
can they complete the work?
a
A.
60

10
„11
B.
60 — „60
C.
11
— „60
D.
13 — „
E. None of these
XT

Language & Communication


Fill in the gaps with the most appropriate option. (Questions 16 to 21)
16. Having no sense of ethics, Kamal was as little subject to the reproaches of conscience after he acted as
he was motivated by its before he acted.
A. chastisement B. eloquence C. promptings D. allure E. atonement

17. Within the next century, sophisticated telescopes now orbiting the Earth will determine whether the
continents really are moving, the incipient rift among geologists about the validity of the theory of
continental drift.
A. obviating B. forestalling C. escalating D. engendering E. resolving

18. His imperturbability in the face of evidence indicating his deliberate fraud failed to reasswe supporters
of his essential ; instead, it suggested a talent for guile that they had never suspected.
A. culpability B. wisdom C. remorse D. probity E. combativeness

19. Mukit has a pleasing , making everyone impressed with speaking tone.

A. Credence B. Deconun C. Deference D. Ebullience E. Cadence

20. Mike’s dancing style is so that anybody who wants to follow his act looks like a drunk elephant
by comparison
A. Inimitable B. Exemplary C. Laudable D. Estimable E. Insuperable

21. Growth in South Asian region is dampening, says the World Bank in its twice-a-year update,
underscoring the need for countries to build
A. Fragility B. verbosity C. fidelity D. resilience E. tirade

Questions 22 to 26 are based on the following instruction:


Instructions: Tire sentences belowr contain errors in grammar, usage and idiom. Parts of each sentence are
underlined & lettered. Decide which underlined part contains error. If the sentences is correct
as it stands, mark (E) on your Answer Sheet.
22. Many people claim to have seen (A) UFOs, but (B) not one has (C) proved that such objects (D) exist.
No Error (E).
https://www.fb.com/capstonebd [01972277866] Page: 2
23. 1 was shocked (A) when he told me (B) that the old man (C) died bv cancer (D). No Error (E).

24. Putin has made thin (i) veiled threats to use nuclear weapons if he feels he has run out of options (ii) in their
(iii) bid to seize swaths of Ukrainian (iv) territory in the face of stiff resistance by Western-back Kyiv.
A. Only i B. only iv C. both i & ii D. only iii E. both i & iii

25. Doctors have found that (A) herbal medications, when combined with a more traditional (B) medical
approach shortens (C) the healing time for a number (D) of surgical procedures. No Error (E).

26. Tire lieutenant Galib reminded (A) his men that the only information to be given (B) to the captors was
each (C) individual’s name, rank and what his serial number was (D). No Error (E).

27. Find out the correct spelling from the following options-
A. Finess B. Finnes C. Finesse D. Fineas E. Finnesse

28. Find out the correct spelling from the following options-
A. Mellifluous B. Melifluous C. Melliflus D. Mallifluous E. Mellifluas

Questions 29 and 30 are based on the following instruction:


Instruction: Find out the sentences that is/are showing collect use of the underlined words
29. i. Danya, now nearly 14, was ambiguous about leaving, drawn toward being a teenager in New York
City.
ii. The time of action is upon us, but I remain ambivalent as to what action I should take.
iii. He was vague and ambivalent on matters of policy
A. i & ii B. only ii C. only iii D. ii & iii E. i, ii & iii

30. i. He set out for it, limping, while the sharp gravel rolled under his bleeding feet as he foundered up the
climbing trail
ii. Pratt resisted the impulse of most Mormons to head back to the foundering ship.
iii. Xinhua, in an English-language commentary, said China could not stand by while its largest hading
partner foundered.
A. i B. ii & iii C. i & iii D. iii E. i, ii & iii

Questions 31 to 35 are based on the following instruction:


Instructions: hr each of the following sentences, part or all of the sentence is underlined. The answer¬
choices offer five ways of phrasing the underlined part. If you think the sentence as written is
better than the alternatives, choose A, which merely repeats the underlined part; otherwise
choose one of the alternatives
3 1 . A popular lecturer who spoke as eloquently on Christianity as literature, Lewis combined faith and
fiction in his allegorical tales of Narnia.
A. as eloquently on Christianity as literature, Lewis combined faith and fiction in his allegorical
B. as eloquently on Christianity so literatrue, Lewis combined faith and fiction in its allegorical
C. as eloquently on Christianity as on literatrue, Lewis combined faith and fiction in its allegorical
D. as eloquently on Christianity as on literatrue, Lewis combined faith and fiction in his allegorical
E. as eloquently on Christianity and literature also, Lewis combined faith and fiction in his allegorical

32. hr many respects, California’s Tevis Cup race and Australia's Quilty Cup are very similar equestrian
event, but the Tevis Cup poses the greatest challenge to both horses and riders.
A. are very similar equestrian event, brit the Tevis Cup poses tire greatest challenge
B. are very similar equestrian events, but the Tevis Cup pose the greatest challenge
C. are very similar equestrian events, brrt the Tevis Cup pose the greater challenge
D. are very similar equestrian events, brrt the Tevis Crrp poses the gr eater challenge
E. are very similar equestrian events, but the Tevis Cup poses the greatest challenge
https://www.fb.com/capstonebd [01972277866] Page: 3
33. Ill most states where local property taxes fluid the public schools, communities with strong tax bases for
a commercial property can be supported its schools while maintaining low property tax rates.
A. with strong tax bases for a commercial property can be supported its schools while maintaining.
B. with strong tax bases for a commercial property can support its schools while maintaining.
C. with str ong tax bases from a commercial property can be supported their schools while maintaining.
D. with strong tax bases for a commercial property can be supported their schools while maintaining.
E. with strong tax bases from a commercial property can support their schools while maintaining.

34. Godard is part biography, part cultural analysis, and it partly pays tribute to an artist who, the author
believes, is one of the most influential of his time.
A. analysis, and it partly pays tribute to an artist who, the author believes, is one of the most
B. analysis, and part tribute to an artist who, the author believes, are one of the most
C. analysis, and part tribute to an artist who, the author believes, is one of the most
D. analysis, and part tribute to a artist who are one of the most
E. analysis, and part tribute to an artist who, the author believes, are the most

35. Examining the principal movements sweeping through the world, it can be seen that they are being
accelerated by the war.
A. Examining the principal movements sweeping through die world, it can be seen
B. Having examined the principal movements sweeping through the world, it can be seen
C. Examining the principal movements sweeping through the world can be seen
D. Examining the principal movements sweeping through the world, we can see
E. It can be seen examining the principal movements sweeping through the world

Read the passage and answer the following questions. (Questions 36-40)
The majority of successfill senior managers do not closely follow die classical rational model of first
clarifying goals, assessing the problem, formulating options, estimating likelihoods of success, making a
decision, and only then taking action to implement the decision. Rather, in their day-by-day tactical
maneuvers, these senior executives rely on what is vaguely termed “intuition” to manage a network of
interrelated problems that require them to deal with ambiguity, inconsistency, novelty, and surprise; and
to integrate action into the process to thinking.
Generations of writers on management have recognized that some practicing managers rely heavily
on intuition, higeneral, however, such wr iters display a poor grasp of what intuition is. Some see it as
the opposite of rationality; others view it as an excuse for capriciousness.
Isenberg's recent research on the cognitive processes of senior managers reveals that managers’
intuition is neither of these. Rather, senior managers use intuition in at least five distinct ways. First, they
intuitively sense when a problem exists. Second, managers rely on intuition to perform well-learned
behavior patterns rapidly. This intuition is not arbitrary or irrational, but is based on years of painstaking
practice and hands-on experience that build skills. A third function of intuition is to synthesize isolated
bits of data and practice into an integrated picture, often in an “Alia!” experience. Fourth, some
managers use intuition as a check on the results of more rational analysis. Most senior executives are
familiar with the formal decision analysis models and tools, and those who use such systematic methods
for reaching decisions are occasionally leery of solutions suggested by these methods which run counter
to their sense of the conect course of action. Finally, managers can use intuition to bypass in-depth
analysis and move rapidly to engender a plausible solution. Used in this way, intuition is an almost
instantaneous cognitive process in which a manager recognizes familiar patterns.
One of the implications of the intuitive style of executive management is that “thinking” is
inseparable from acting. Since managers often “know” what is right before they can analyze and explain
it, they frequently act first and explain later. Analysis is inextricably tied to action in thinking/acting
cycles, in which managers develop thoughts about their companies and organizations not by analyzing a
problematic situation and then acting, but by acting and analyzing in close concert.

https://www.fb.com/capstonebd [01972277866] Page: 4


Given the great uncertainty of many of the management issues that they face, senior managers often
instigate a course of action simply to learn more about an issue. They then use the results of the action to
develop a more complete understanding of the issue. One implication of thinking/acting cycles is that
action is often part of defining the problem, not just of implementing the solution.
36. According to the passage, senior managers use intuition in all of the following ways EXCEPT to-
A. stipulate clear goals B. identify a problem C. bring together disparate facts
D. speed up of the creation of a solution to a problem E. evaluate possible solutions to a problem
37. The passage suggests which of the following about the “writers on management” mentioned in
paragraph 2?
A. They have criticized managers for not following the classical rational model of decision analysis.
B. They have not based their analyses on a sufficiently large sample of actual managers.
C. They have relied in drawing their conclusions on what managers say rather than on what managers do.
D. They have misunderstood how managers use intuition in making business decisions.
E. They have not acknowledged the role of intuition in managerial practice
38. According to the passage, the classical model of decision analysis includes all of the following EXCEPT
A. evaluation of a problem
B. creation of possible solutions to a problem
C. establishment of clear goals to be reached by the decision
D. action undertaken in order to discover more information about a problem
E. comparison of the probable effects of different solutions to a problem
39. Tire passage provides support for which of the following statements?
A. Managers who rely on intuition are more successful than those who rely on formal decision analysis.
B. Managers cannot justify their intuitive decisions.
C. Managers’ intuition works contrary to their rational and analytical skills
D. Logical analysis of a problem increases the number of possible solutions.
E. Intuition enables managers to employ their practical experience more efficiently.
40. It can be inferred from the passage that which of the following would most probably be one major
difference in behavior between Manager X, who uses intuition to reach decisions, and Manager Y, who
uses only formal decision analysis?
A. Manager X analyzes first and then acts; Manager Y does not.
B. Manager X checks possible solutions to a problem by systematic analysis; Manager Y does not
C. Manager X takes action in order to arrive at the solution to a problem; Manager Y does not.
D. Manager Y draws on years of hands-on experience in creating a solution to a problem; Manager X
does not.
E. Manager Y depends on day-to-day tactical maneuvering; manager X does not

Analytical Ability
Directions: Solve each of the following problems & mark the con ect answer on your Answer Sheet.
Questions 41 to 46 are based on the following situation:
Six contestants, R, S, T, U, V, and W, are to be placed first (highest) through sixth (lowest), though
not necessarily in that order, at the start of a singles Ping-Pong challenge contest.
R is placed above S.
V is placed above both T and U.
W is placed two places above T.
R is placed either third or fourth.
During the contest, a player may challenge only the player placed immediately above him or the
player placed two places above him.
41. Which of the following is a possible initial placing from highest to lowest?
A. V, T, W, R, U, S B. W, U, T, V, R, S C. W, S, T, R, V, V
D. V, W, T, U, S
R, E. V, W, T, U, S
R,
https://www.fb.com/capstonebd [01972277866] Page: 5
42. If W is initially placed fust, which of the following must also be true of the initial placing?
A. V is placed second. B. T is placed second. C. R is placed third.
D. S is placed fifth. E. U is placed sixth.
43. If R is initially placed third, which of the following must also be true of the initial placing?
A. V is placed first. B. W is placed second. C. S is placed fourth.
D. U is placed fourth E. U is placed sixth.

44. If W is initially placed third, and if W make the first challenge, which of the following contestants could
W play in the first match?
I. R IE T III. U IV. V
A. I and II only B. I and III only C. II and IV only D. Ill and IV only E. I, II, and IV

45. If the first challenge of the contest is made by R against T, all of the following must be true of the initial
placing EXCEPT
A. W is placed first. B. V is placed second. C. T is placed third.
D. R is placed fourth. E. U is placed fifth.

46. If V makes the first challenge of the contest against W, then which of the following must be hue of the
initial rankings?
A. W is placed first. B. V is placed third. C. R is placed third.
D. T is placed fourth. E. S is placed fifth.

Data Sufficiency (Questions 47 to 49)


Instruction: Each of the following problems has two statements which are labeled (i) and (ii). Use the data
given in (i) and (ii) to decide whether the statements are sufficient to answer the questions.
A. Statement (i) alone is sufficient but statement (ii) alone not sufficient to answer the question asked.
B. Statement (ii) alone is sufficient but statement (i) alone not sufficient to answer the question asked.
C. Both statement (i), (ii) together are sufficient to answer the question but neither statement is
sufficient alone.
D. Each statement alone is sufficient to answer the question.
E. Statement (i) & (ii) are not sufficient to answer the question asked and additional data is needed to
answer the statements.

47. The 10 students in a history class recently took an examination. What was the maximum score on the
examination?
i. The mean of Die score was 75 ii. Ure standard deviation of the score was 5

48. If q, s, and t are all different numbers, is q < s < t ?


i. t-q=| t -s | +| s- q | ii. t > q

49. Is the positive two-digit integer N less than 40?


i. the units digit of N is 6 more than the tens digit. ii. N is 4 less than 4 times the units digit.

50. Now Mesa Electronics brings tomorrow’s technology to today’s home stereo. The same space-age circuitry
used by the Wanderer spacecraft to send images of the most distant planets back to earth has been
incorporated in the new Mesa X-2700 stereo system. If your home entertainment is important to you, why
settle for an old-fashioned stereo system when the Mesa X-2700 is priced at only a few dollars more?
The most serious logical weakness of this argument is its failure to
A. Provide technical data on the manufacturing specifications of the Mesa X-2700
B. Show the relevance of space-age circuitry to the requirements of home stereo
C. Specify the exact price difference between the Mesa X-2700 and old-fashined stereo systems
D. Acknowledge the contributions of scientists in the space program to the development of the
advanced circuitry mentioned
E. Explain the precise meaning of the technical terms used
https://www.fb.com/capstonebd [01972277866] Page: 6
CAPSTONE Pre-Mock Test-Ill
Time: 1 hour Marks: 50
Mathematics
Instructions: Solve each of the following problems and mark the correct answer on your Answer Sheet.
DO NOT USE A CALCULATOR. Figures are not drawn to scale.
1. Tire ratio of green, red and yellow marbles is 3:4:5. If 10 green marbles are removed and 5 red marbles
are added; the total number of marbles becomes 103. How many yellow marbles were in the box?
A. 48 B. 42 C.45 D. 40 E. None of these

2. Tire L.C.M. of two numbers is 48. The numbers are in the ratio 2:3. Then sum of the number is:
A. 28 B. 32 C.40 D. 64 E. None of these
3. Two trains, each 100 m long, moving in opposite directions, cross each other in 8 seconds. If one is
moving twice as fast the other, then the speed of the faster train is:
A. 30 km/hr B. 45 km/hr C. 60 km/hr D. 75 km/hr E. None of these
4. The ratio of water and milk in bottle A and bottle B was 5:2 and 7:6 respectively. What ratio of mixture
is taken from both pot that the ratio of milk and water will be 8:5?
A. 6:5 B. 2:3 C. 7:9 D. 4:7 E. 11:13
5. The ratio of milk and water in a mixture is 5:4. If 45 liters of the mixture is taken out and replaced by the
same amount of water then the ratio of milk and water becomes 5:7. Find the amount of milk in the
original mixture.
A. 100 B. 80 C. 90 D. 75 E. None of these
6. Nine places of paper numbered consecutively from 1 to 9 are put into a box. If a piece of paper is drawn
at random from the box, what is the probability that it will have a number greater than 6?
112 5
A.- B. - C. - D. - E. None of these
9 3 9 9

7. If the area of a square with sides of length 8 centimeters is equal to the area of a rectangle with a width
of 4 centimeters, what is the perimeter of the rectangle in centimeters?
A. 16 B. 20 C. 32 D. 40 E. None of these
8. The ratio between a two digit number and the sum of the digits of that number is 4:1. If the digit in the
unit’s place is 3 more than the digit in the ten’s place, then the number is?
A. 24 B. 36 C. 63 D. 96 E. None of these
9. Rakib and Zubaeer start a business with initial investments in the ratio 12:11 and their annual profits
were in the ratio 4: 1 . If Rakib invested the money for 11 months, Zubaeer invested the money for:
A. 3 B. 3.33 C.4 D. 6 E. None of these
10. The distance between two stations A and B is 365 km. A train starts at 10 a.m. from A and move towards
B at an average speed of 65 km/hr. Another train starts B at 11 a.m. and moves towards A at an average
speed of 35 km/hr. How far from B will the two trains meet and at what time?
A. 105,2:00PM B. 100, 4:00 PM C. 100,2:00PM D. 105,5:00PM E. 106,5:00PM

11. 12 men complete a work in 9 days. After they have worked for 6 days, 4 more men join them. How
many days will they take to complete the remaining work?
A. 2 days B. 2.25 days C. 2.5 days D. 3.25 days E. None of these
https://www.fb.com/capstonebd [01972277866] Page: 1
12. A shopkeeper offers a discount of 20% on the selling price. On a special sale day, he offers an extra 25%
off coupon after the first discount. If the article was sold for tk. 3600, find the marked price of the article.
A. 6000 B. 5500 C. 5200 D. 60000 E. None of these
13. 6 years ago, the ratio of the ages of Ayon and Saad was 6:5. 4 years hence, the ratio of their ages will be
11:10. What is Saad’s age at present?
A. 12 years B. 14 years C. 16 years D. 18 years E. None of these

14. Ali, Shahadat and Omor can do a piece of work in 24 days, 30 days and 40 days respectively. They
began the work together but Omor left 4 days before the completion of the work, hi how many days was
the work completed?
A. 13 B. 11 C. 10 D. 15 E.Noneofthese

15. 60% members of a club are female. 50% of the female members are doctors. The number of female
doctors is twice the number of male non-doctors. What percent of the members are doctors?
A. 55% B. 50% C.40% D. 45% E.Noneofthese

16. A triangle has a base b and an altitude a. A second triangle has a base twice the altitude of the first
triangle, and an altitude twice the base of the first triangle. What is the area of the second triangle?
A. —2 ab B. ab C. 2ab D. 4ab E. 6ab

17. A lending library charges c cents for the first week that a book is loaned and f cents for each day over
one week. What is the cost for taking out a book for d days, where d is greater than 7?
A. c + fd B. c + f(d-7) C.cd D. 7c + f(d-7) E. cd + f

1 8. A dealer buys 40 tables all at the same price. He sells 30 of them at a gain of 25% and the rest at a loss
of 10%. Percentage of profit on total transaction is:
A. 18% B. 20% C. 16.25% D. 12.50% E.Noneofthese

19. Umana and Maliha can do a piece of work in 30 days, while Maliha and Sayeda can do the same work in
24 days and Sayeda and Umana in 20 days. They all work together for 10 days when Maliha and Sayeda
leave. How many days more will Umana take to finish the work?
A. 20 days B. 17 days C. 19 days D. 18 days E.Noneofthese

20. hi June a baseball team that played 60 games had won 30% of its games played. After a phenomenal
winning streak this team raised its average to 50%. How many games must the team have won in a row
to attain this average?
A. 12 B. 20 C. 24 D. 30 E. 45

Language and Communication


Instruction: Choose the most appropriate word(s)/phrase in the following questions 21-25
21 . As long as the acquisition of knowledge is rendered habitually , so long will there be a prevailing
tendency to discontinue it when free fr om the of parents and teachers.
A. repugnant .... coercion B . academic.... authority C . gratifying restrictions
D. honorable influence E. irrelevant custody

22. When the disease is in state it is almost impossible to determine its existence by .
A. a dormant.... postulate B. a critical.... examination C. an acute.... analysis
D. a suspended estimate E. a latent....observation

23. When Jacob started her documentary of the shuns, she was immediately by groups of
children begging for food.
A. affluent... bedraggled B. impoverished.... Accosted C. redundant.... Infiltrated
D. slaughtered. ... Rebuked E. demonstrated .... Rationalized
https://www.fb.com/capstonebd [01972277866] Page: 2
24. Lack of is basic to good teamwork but our ability to work with others depends on our .
A. rigidity compatibility
- B. dogmatism motivation
- C. consideration - acumen
D. positivism - flexibility E. professionalism - vulnerability

25. Tire fear with feeling incompetent is the fear of being humiliated, embarrassed and .
A. endowed - criticized B. afflicted - downtrodden C. consistent - damaged
D. associated - vulnerable E. imbued - exposed

Instruction: Answer the questions (26-30) based on the following words-


i. Despondent ii. Sparkle iii. Awful
iv. Causing dismay v. Rejoice vi. Conspicuous
vii. Be euphoric viii. Experienced through another

26. Which of the following options expresses the meaning of the word- ‘Scintillating’?
A. vi B.vii C. ii & iv D. ii E. ii&vi

27. Which of the following options expresses the meaning of the word- ‘Vicarious’?
A. v B.vi C. i D. viii E.vii

28. Which of the following options expresses the meaning of the word- ‘Appalling’?
A. ii B.vi C. iii D. iii & iv E. viii

29. Which of the following options expresses the meaning of the word- ‘Doleful’?
A. i B.v C. vi D. vii E.viii
30. Which of the following options expresses the meaning of the word- ‘Exult’?
A. i B. iii C. v&vii D. vii E.viii

Read the passage and answer the following questions. (Questions: 31-35)
Federal efforts to aid minority businesses began in the 1960's when the Small Business
Administration (SBA) began making federally guaranteed loans and government-sponsored management
and technical assistance available to minority business enterprises. While this program enabled many
minority entrepreneurs to form new businesses, the results were disappointing, since managerial
inexperience, unfavorable locations, and capital shortages led to high failure rates. Even 15 years after
the program was implemented, minority business receipts were not quite two percent of the national
economy’s total receipts. Recently federal policymakers have adopted an approach intended to
accelerate development of the minority business sector by moving away from directly aiding small
minority enterprises and toward supporting larger, growth-oriented minority firms through intermediary
companies, hr this approach, large corporations participate in the development of successfill and stable
minority businesses by making use of government- sponsored venhire capital. The capital is used by a
participating company to establish a Minority Enterprise Small Business Investment Company or
MESBIC. The MESBIC then provides capital and guidance to minority businesses that have potential to
become future suppliers or customers of the sponsoring company.
MESBIC’s are the result of the belief that providing established firms with easier access to relevant
management techniques and more job-specific experience, as well as substantial amounts of capital,
gives those firms a greater opportunity to develop sound business foundations than does simply making
general management experience and small amounts of capital available. Further, since potential markets
for the minority businesses already exist through the sponsoring companies, the minority businesses face
considerably less risk in terms of location and market fluctuation. Following early financial and
operating problems, sponsoring corporations began to capitalize MESBIC’s far above the legal
minimum of $500,000 in order to generate sufficient income and to sustain the quality of management
needed. MESBIC’s are now emerging as increasingly important financing sources for minority
enterprises.
https://www.fb.com/capstonebd [01972277866] Page: 3
Ironically, MESBIC staffs, which usually consist of Hispanic and Black professionals, tend to
approach investments in minority firms more pragmatically than do many MESBIC directors, who are
usually senior managers from sponsoring corporations. The latter often still think mainly in terms of the
“social responsibility approach” and thus seem to prefer deals that are riskier and less attractive than
normal investment criteria would warrant. Such differences in viewpoint have produced uneasiness
among many minority staff members, who feel that minority entrepreneurs and businesses should be
judged by established business considerations. These staff members believe their point of view is closer
to the original philosophy of MESBIC’s and they are concerned that, unless a more prudent course is
followed, MESBIC directors may revert to policies likely to re-create the disappointing results of the
original SBA approach.

31 . Which of the following best states the central idea of the passage?
A. The use of MESBIC’s for aiding minority entrepreneurs seems to have greater potential for success
than does the original SBA approach.
B. There is a crucial difference in point of view between tire staff and directors of some MESBIC’s.
C. After initial problems with management and marketing, minority businesses have begun to expand at
a steady rate.
D. Minority entrepreneurs wishing to form new businesses now have several equally successfill federal
programs on which to rely.
E. For the fust time since 1960, large corporations are making significant contributions to the
development of minority businesses

32. According to the passage, the MESBIC approach differs from tire SBA approach in that MESBIC’s
A. seek federal contracts to provide markets for minority businesses
B. encourage minority businesses to provide markets for other minority businesses
C. attempt to maintain a specified rate of growth in the minority business sector
D. rely on the participation of large corporations to finance minority businesses
E. select minority businesses on the basis of their location

33. Which of the following does the author cite to support the conclusion that the results of the SBA
progr am were disappointing?
A. The small number of new minority enterprises formed as a result of the program
B. The small munber of minority enterprises that took advantage of the management and technical
assistance offered under the program
C. The small percentage of the nation's business receipts earned by minority enterprises following the
programs, implementation
D. Tlie small percentage of recipient minority enterprises that were able to repay federally guaranteed
loans made under the progr am
E. The small number of minority enterprises that chose to participate in the progr am

34. The author's primary objective in the passage is to


A. disprove the view that federal efforts to aid minority businesses have been ineffective
B. explain how federal efforts to aid minority businesses have changed since the 1960's
C. establish a direct link between the federal efforts to aid minority businesses made before the 1960’s
and those made in the 1980's
D. analyze the basis for the belief that job- specific experience is more useful to minority businesses than
is general management experience
E. argue that the “social responsibility approach” to aiding minority businesses is superior to any other
approach

35. It can be inferred from the passage that the attitude of some MESBIC staff members toward the
investments preferred by some MESBIC directors can best be described as
A. defensive B. resigned C. indifferent D. shocked E. disapproving
https://www.fb.com/capstonebd [01972277866] Page: 4
Analytical Ability
Questions 36 to 41 are based on the following situation:
Nine student—O, P, Q, R, S, T, U, V, and W—are the only student who can serve on three
commissions designated A, B, and C, and each student must serve on exactly one of the
commissions.
Commission A must have exactly one more member than does commission B.
It is possible that there are no members of commission C.
Neither O nor P nor Q can serve on commission A.
Neither R nor S nor T can serve on commission B.
Neither U nor V nor W can serve on commission C.

36. If U and O are the only students serving on commission B, how many of die nine student must serve on
commission C?
A. 3 B. 4 C. 5 D. 6 E. 7

37. Of the nine student, the gr eatest number that can seive together on commission C is
A. 9 B. 8 C. 7 D. 6 E. 5

38. IfW is the only student serving on commission B, which of the following must serve on commission A?
A. RandV B. SandT C. SandU D. TandV E. UandV

39. If none of the nine student serves on commission C, which of the following must be a student who serves
on commission A?
A. O B. P C. R D. U E. V

40. If U, V, and Q are the only students serving on commission B, the complete membership of commission
C must be
A. O and P B. O and R C. P and S D P and T E. Q and S

41. Which of the following gr oups could constitute tire membership of commission C?
A. PandU B. QandT C.P, Q,andR D. R, S, and T E. O, Q, T, and W

Questions 42 to 46 are based on the following situation:


Exactly six workers F, G, H, I, J, and K— take turns watching a vehicle. To minimize the chance that the
vehicle will observe them or will, escape their inspection, the workers will operate according to die
following conditions:
There must be exactly two workers watching the vehicle at all times.
Whenever the vehicle travels by car, either H or I must be one of the two workers watching, but H
and I cannot take a turn together.
Whenever the vehicle travels by train, either G or K must be one of the two workers watching, but G
and K cannot take a turn together.
If K is one of die workers watching the vehicle, F must be the other worker watching the vehicle.
F and I cannot work together watching the vehicle.

42. If the vehicle is traveling by car, which of the following is a pair of workers who could be watching?
A. FandG B. F and I C. Gand J D. H and I E. H and J

43. If the vehicle travels by train and G is not one of the workers watching, the pair watching must be
A.FandJ B.FandK C.HandK D.IandJ E.IandK

44. If J is one of the workers watching the vehicle, which of the following CANNOT be the other worker
watching the vehicle?
A. F B. G C. H D. I E. K

https://www.fb.com/capstonebd [01972277866] Page: 5


45. If the vehicle travels by car and F is one of the workers watching, the other worker watching the vehicle
must be,
A. G B. H C. I D. J E. K

46. If a pair of workers was watching while the vehicle traveled by car, and the same pair of workers
continued watching after the vehicle began to travel by train, one member of that pair of workers must
have been
A. G B. H C. I D. J E. K

Questions 47 and 48
Instructions: Answer the questions based on the following article.
hi a laboratory study, 160 rabbits in an experimental group were injected with Serum D, while 160
rabbits in a control group were injected with a harmless sugar solution. Within two weeks, 39% of the
experimental group rabbits had contracted jungle fever, a highly contagious and usually fatal disease.
Therefore, jungle fever must be caused by some substance similar to the substances found in Serum D.

47. The above argument would be most gr eatly strengthened if it were shown that
A. The normal rate of jungle fever among rabbits is less than 0.01%
B. 40 % of die rabbits in the control group had also contracted jungle fever within two weeks.
C. Serum D contains substances extracted from the root of a certain poisonous jungle wildflower
D. Tire blood of jungle fever victims invariably contains a high level of certain toxic substance also
found in Serum D
E. Nearly all the rabbits who contracted jungle fever died within two days of the appearance of the first
symptoms.

48. The above argument would be most seriously weakened if it were shown that
A. None of the substances in Serum D occurs naturally in the habitats of most species of rabbit
B. The rabbits in the experimental group had been kept strictly isolated form one another
C. Jungle fever is usually found only among victims of the bite of the South American Lesser Hooded
Viper
D. The scientists administering the injections were unaware of the contents of the solutions they were
using
E. One of the rabbits in the experimental gr oup had had jungle fever prior to the start of the experiment.

Data Sufficiency (Questions 49 and 50)


A. Statement (i) alone is sufficient but statement (ii) alone not sufficient to answer tire question asked.
B. Statement (ii) alone is sufficient but statement (i) alone not sufficient to answer the question asked.
C. Both statement (i), (ii) together are sufficient to answer the question but neither statement is sufficient
alone.
D. Each statement alone is sufficient to answer the question.
E. Statement (i) & (ii) are not sufficient to answer the question asked and additional data is needed to
answer the statements.

49. If X is a positive integer, then is x prime?


i. 3x+l is prime ii. 5x+l is prime

50. Are all of the numbers in a certain list of 15 munbers equal?


i. The sum of all the numbers in the list is 60 ii. The sum of any 3 munbers in the list is 12

https://www.fb.com/capstonebd [01972277866] Page: 6


“In the name of ACCah, the most Beneficent, the most merciful”
CAPSTONE
Contact: 01972 277 866, 016 3031 3031 Web: www.capstonebd.com facebook.com/capstonebd

Mock Test for IBA (DU) MBA Admission

INSTRUCTION: Please read carefully! Test Booklet No: 0001


1. The test consists of two parts: I. Multiple Choice Questions (MCQ) and IL Written Ability.
This part to be filled in
2. Total duration of the test will be 120 minutes (90 for Part-I &
30 minutes for Part II). The answer Sheet (For MCQs) will be by the Candidate
taken away after 90 minutes.
Admit No
3. On this Answer Sheet, write your Admit Number and Test Name
Booklet Number in the grids and your Name and Signature in
the boxes provided using a pen. Darken the corresponding
number boxes below the grids using a pen.

4. On this Test Booklet write your Admit Number and Name in the spaces provided using a pen.

5. Give all your answers in the Answer Sheet by completely darkening the lettered box, which represents
the correct answer to that the letter cannot be seen USE Pen ONLY.

6. Do rough work anywhere on the rest booklet,


This part to be filled in by the invigilator
except in the spaces provided for written answers.
Do not use the answer sheet or extra paper for Please verify that
rough work. Candidate’s Name & Admit Number matches the
information on the admit card.
7. For each MCQ, 25% of assigned marks will Candidate’s appearances resembles the photograph
be deducted for each wrong answer. Each on the Admit Card.
unanswered question will earn zero marks.
Admit No
8. Candidates are required to obtain a minimum
acceptable mark in each section and in each Signature of the Invigilator
part.

9. Switch off your mobile phones.

10. Your script will not be examined if you adopt any unfair means.

11. Do not use a calculator or any other electronic devices.

12. Remain seated silently at the end until you are asked to go.

|sTOP! DO NOT TURN THIS PAGE UNTIL YOU ARE ASKED TO DO SO.|
For Office Use Only
Part I: Multiple Choice Questions [75 marks]
Section 1: Mathematics 30x1=30 marks
Instructions: Solve each of the following problems and mark die correct answer on your Answer Sheet.
DO NOT USE A CALCULATOR. Figures are not drawn to scale.

1 . The product of two natural numbers is 1 7. Then, the sum of the reciprocals of their squares is:
A.—
1
189 —
290
B. 289 C.—
290
289
D. 289 E. None of these

2. If -1 < x < 2 and 1 < y < 3, then least possible value of (2x - 3y) is:
A. 0 B. —3 C. —4 D. 5 — E. None of these

3. A group of n students bought three identical round cakes to share. They divided the first cake into equal¬
sized pieces, one piece for each of them. They did the same witii die second cake. After 3 of the students
decided they did not want any more cake, the remaining students divided the third cake into equal-sized
pieces, one piece for each of them. If Shanta received 1 piece from each of the three cakes, then, in terms
of n, the amount of cake that she received was the same as what fr action of 1 cake?
A
A. —n+2
7
n(n-3)
H. —
2n—3
;
n(n-3)
C. —
3n-3
7
n(n—3)
r D.
3n—6
n(n— 3)
E. None

4. When 5125 is divided by 13, the remainder obtained is:


A. 10 B. 12 C. 2 D. 1 E. None of these

5 . How many tunes are the hands of a clock at right angel hi a day?
A. 22 B. 24 C.44 D. 48 E. None of these

6. If a, b, and c are consecutive integers and a < b < c , which of the following must be true?

I. b-c = l II. is an integer III. a + b + c is even.

A. I only B. II only C. Ill only D. I and II only E. II and III only

7. hi the tr apezoid, ZCAB = ZDBA = 60° and AC = DB = 10 cm. Find the area of the trapezoid.
A. 250V3 B. 150 C. 250 V B
D. 125V3 E. None of these C A)

8. If|x-2|< 5 , which of the following must be hue?


A. x>0 B. x>8 C. — 3<x<7 D. 0 < x < 3 E. None of these

9. If books bought at prices ranging from tk. 200 to tk. 350 are sold at prices ranging from tk. 300 to tk.
425, what is the greatest possible profit that might be made in selling eight books?
A.1800 B. 1500 C. 1200 D. 600 E. Cannot be determined

10. The bowling average of Saad was 12.4. He improves his bowling average by 0.2 points when he takes 5
wickets for 26 runs in his last match. The number of wickets taken by him before the last match was:
A. 125 B. 150 C. 175 D. 200 E. None of these
Page 1
11. If a square and a rhombus stand on the same base, then what is the ratio of the areas of the square and the
rhombus?
1 3
A. equal to - B. equal to - C. greater than 1 D. equal to 1 E. Insufficient data

12. If x, y, and z are nonzero integers and x > yz, which of the following must be true?

I.
T
—y > z
X
II.
TT
—z > y
X
III.
TTT

X

yz
>1

A. I only B. Ill only C. I and II only D. All of these E. None of these

13. If x is an odd integer, in which of the following equations must y be an even integer?
I. xy 5 = n. x + 2y = ll III. 2x+y = 12
A. Only I B. Only II C. Only III D. Both II and III E. All of these

14. Ricky solved 80 percent of the questions in an examination correctly. If out of 41 questions solved by
Ricky, 37 questions are correct and of the remaining questions, out of every 8 questions 5 questions have
been solved correctly by Ricky, then find the total number of question asked in the examination?
A. 75 B. 65 C. 60 D. 51 E. None of these

15. The rate of interest for the first 2 years is 5%, for the next 3 years is 8% and beyond this it is 10% per
annum. If the simple interest for 8 years is Tk. 1280. What is the principal?
A. Tk. 2000 B. Tk. 2500 C. Tk. 2440 D. Tk. 3200 E. None of these

16. Richard desires to run a certain course in 1/4 less time than he usually takes. By What percent must
Richard increase his average running speed to accomplish the goal?
A. 50.25% B. 20.12% C. 25.39% D. 33.33% E. None of these

17. After five years the age of a father will be thrice the age of his son whereas five years ago, he was seven
times as old as his son was. What is father’s present age?
A. 33 B. 40 C.47 D. 59 E. None of these

1 8. If a < 0 and b > 0, then which of the following is true?


I. ab-(a + b)>0 n. ab-(a + b) <0 III. ab-(a + b) = 0
A. Only I B. Only II C. Both I & II D. Both I and in E. All of them

19. hi the following diagram, AB || CD. Then X is equal to what? *


A. 93° B. 103° C. 83°
D. 97° E. None of these «—•—D C
>

20. An amount of money was divided between some people in such a way that if there had been 4 more
people, everyone would have got tk. 1 6 less. But if there had been 4 less people, everyone would have
got tk. 24 more. How many people were there in the group?
A. 32 B. 24 C. 20 D. 16 E. None of these

21. Ayon sells transistors at 15% above its cost price. If he had bought it at 5% more than what he paid for it
and sold it for Tk. 6 more, he would have gained 10%. The cost price of the transistor is?
A. Tk. 800 B. Tk. 1000 C. Tk. 1200 D. Tk. 1400 E. None of these
Page 2
22. For eveiy 4 leaps taken by a dog, a cat takes 5 leaps in the same time. But distance covered in 3 leaps of
dog is same as that of 4 leaps of cat. Then what is the ratio of speed of dog and cat?
A. 15:14 B. 16:15 C. 12:11 D. 10:9 E. None of these

23. Brothers A and B had some savings in the ratio 4:5. They decided to buy a gift for their sister, sharing
the cost in the ratio 3:4. After they bought, A spent two-third of his amount, while B is left with Tk. 145.
Then, the value of the gift is what?
A. Tk. 70 B. Tk. 105 C. Tk. 140 D. Tk.175 E. None of these

24. The difference between a two-digit number and the number obtained by interchanging the digits is 36.
What is the difference between the sum and the difference of the digits of the number if the ratio
between the digits of the number is 1: 2?
A. 4 B. 8 C. 16 D. 18 E. 12

25. There is a group of persons each of whom can complete a piece of work in 16 days, when they are
working individually. On the first day one person works, on the second day another person joins him, on
tlie third day one more person joins them and this process continues till the work is completed. How
many days are needed to complete the work?
A. 3- days
4
B. 4-
3
days C. 5-
6
days D. 6^5 days E. 7-2 days

Questions 26 to 30 are based on the following information:


A. Statement (i) alone is sufficient but statement (ii) alone not sufficient to answer the question asked.
B. Statement (ii) alone is sufficient but statement (i) alone not sufficient to answer the question asked.
C. Both statement (i), (ii) together are sufficient to answer the question but neither statement is sufficient alone.
D. Each statement alone is sufficient to answer the question.
E. Statement (i) & (ii) are not sufficient to answer the question asked and additional data is needed to
answer the statements.

26. If x + y = 3, what is the value of xy?


i. x and y are integers ii. x and y are positive

27. How many integers are there between, but not including integers r and s?
i. s-r=10 ii. There are 9 integers between, but not including, r + 1 , s + 1

28. When one piece of fruit is taken at random from a fruit bowl, what is the chance that it is an apple?
i. There are half as many apples as oranges in the fruit bowl.
ii. A third of the fruit in the fruit bowl are oranges.

29. If the three-digit integer x - ‘abc’, where a, b, and c represent nonzero digits of x, what is the value of x?
i. a 3b ii. b≥3c

30. Is n(n + 1 )(n + 2) divisible by 24?


i. n is even ii. (n+1) is divisible by 3 but not by 6

Page 3
Section 2: Language & Communication 30x1=30 marks
Questions 31 to 35 are based on the following instruction:
Instruction: In the following questions five alternatives are given for the idiom/phrase bold or underlined in
the sentence. Choose the alternative which best expresses the meaning of idioms/phrases.

3 1 . hi the organized society of today no individual or nation can plough a lonely furrow.
A. act independently B. remain unaffected C. survive in isolation
D. remain non-aligned E. None of these

32. The case was held over due to the great opposition to it.
A. dropped B. postponed C. stopped D. cancelled E. None of these

33. We planned for the dinner at the eleventh hour, so we decided to have a pot-luck dinner.
A. dinner where eveiy-body brings something to eat B. dinner where everybody pays for his food
C. dinner where people eat and play games at the same time D. dinner where only soup is served
E. None of these

34. The carrot and stick policy pays dividends in every organization.
A. fair and foul B. continuous vigilance C. democratic
D. reward and punishment E. None of these

35. We have appealed to him again and again; there is no use flogging a dead horse now.
A. Repeating our request B. Making him see reason C. Beating about the bush
D. Wasting time in useless effort E. None of these

Questions 36 to 40 are based on the following instruction:


Instruction: Underlined pails of the following sentences may contain error(s) in grammar, diction, usage or
idiom. Choose the underlined portion that contains such error(s). If there is no error then select E.
36. Dhaka North City Corporation and Dhaka South City Corporation have taken (A) no visibly effective
measmes to control the urban dog population, and this has led to (B) city residents treating (C)
neighborhood dogs in a cruel, violent and inhumane manner (D). No error (E).

37. New services and goods export bans, targeted at (A) vulnerable sectors of the Russian economy, have
been announced (B) by the Foreign Secretary today in response to (C) Russia declaring the illegal (D)
annexation of 4 regions of Ukr aine - violating their territorial integrity and political independence. No
error (E).

38. These measmes will also prohibit (A) Russia's access to other world-class professional (B) services,
including (C) auditing and advertise (D) services. No error (E).

39. Even though (A) a promotion might be a somewhat easy (B) method for a store to boost (C) sales,
they (D) may lead some people to shop irresponsibly. No error (E).

40. Finally, the UK will suspend the process of which (A) actions taken to manage the orderly failure (B)
of Russian banks are recognized (C) under the laws of the United Kingdom, in cases where (D) the
bank is a sanctioned entity. No error (E).
Page 4
Questions 41 to 45 are based on the following passage:
Most of Greenland’s interior is covered by a thick layer of ice and compressed snow known as the
Greenland Ice Sheet. The size of the ice sheet fluctuates seasonally: in summer, average daily high
temperatures in Greenland can rise to slightly above 50 degrees Fahrenheit, partially melting the ice; in
the winter, the sheet thickness additional snow falls, and average daily low temperatures can drop [Q41]
to as low as 20 degrees. Typically, the ice sheet begins to show evidence of thawing in late [Q42]
summer. This follows several week of higher temperatures. [Q43] For example, in the summer of
2012, virtually the entire Greenland Ice Sheet underwent thawing at or near its surface by mid-July, the
earliest date on record. Most [Q44] scientists looking for the causes of the Great Melt of 2012 have
focused exclusively on rising temperatures. The summer of 2012 was the wannest in 170 years, records
show. But [Q45] Jason Box, an associate professor of geology at Ohio State believes that another
factor added to the early thaw: the “dark snow” problem.
41. Which choice is the best expression?
A. to as low as 20 degrees B. to 12 degree Fahrenheit C. to their lowest point on December 13
D. to 10 degree Fahrenheit and stay there for months E. None of these

42. Which choice is the best expression?


A. summer, this follows B. summer, following C. summer, and this thawing following
D. summer, and such thawing follows E. summer, and this evidence follows

43. Which choice is the best expression?


A. For example B. As such, C. However, D. Moreover, E. Therefore,

44. Winch choree rs the best expressron?


A. scientists looking for the causes of the Great Melt of 2012 have focused
B. scientist looking to tire causes of the Great Melt of 2012 has focused
C. scientists looking for the causes of the Great Melt of 2012 focuses on
D. scientists look for the causes of the Great Melt of 2012 have focused
E. None of these

45. Which choice is the best expression?


A. Jason Box, an associate professor of geology at Ohio State
B. Jason Box, an associate professor of geology, at Ohio State
C. Jason Box: an associate professor of geology at Ohio State
D. Jason Box-an associate professor of geology at Ohio State,
E. Jason Box, an associate professor of geology at Ohio State,

Questions 46 to 53 are based on the following instruction:


Instruction: Each sentence below has one or two blanks, each blank indicating that something has been
omitted. Beneath the sentence are five words or sets of words labeled A through E. Choose
the word or set of words that, when inserted in the sentence, best fits the meaning of the
sentence as a whole.
46. There is no consensus among doctors on the value of vitamin C supplements, and such suggests
that much further research on the subject is .
A. validity .... expected B. controversy .... surprising C. discovery .... resented
D. disagreement .... necessary E. invariance .... irrelevant
Page 5
47. Wedding ceremonies often include the exchange of rings to symbolize the couple’s promises to
each other.
A. hirsute B. acrimonious C. plaintive D. deciduous E. votive

48. His sense of propriety was , and as such his behavior in social groups was often bizarrely
inappropriate.
A. corollary B. credible C. distorted D. routine E. coherent

49. Tire Rio Gila is part of a of livers and cultures as significant as the of the Tigris and
tire Euphrates.
A. disparity .... conjunction B. intermingling .... dichotomy
C. juxtaposition .... divergence D. conglomeration .... Diaspora
E. convergence .... confluence

50. His suit of armor made the knight to his enemy's attack, and he was able to escape safely to Iris
castle.
A. vulnerable B. churlish C. invulnerable D. static E. imprudent

51. Racha’s glance was a invitation to speak later in private about events of the meeting.
A. treacherous B. scintillating C. tactfill D. tacit E. taboo

52. She reached the of her career with her fourth novel, which won tire Pulitzer Prize.
A. harbinger B. apogee C. metamorphosis D. dictum E. synthesis

53. Even after a century of investigation, the relation of the solar cycle to terrestrial weather
remains .
A. meticulous .... apparent B. cursory .... clear C. sedulous .... pertinent
D. extensive .... enigmatic E. scientific .... unobscured

Questions 54 to 57 are based on the following comprehension:


Instruction: Read the following passage and answer the accompanying questions, base your answer on
what is stated or implied in the passage. Mark your answers on the Answer Sheet.
The first murder ever committed in the United States occurred in September 1630, shortly after the
Pilgi ims arrived in Massachusetts. John Billington was the father of two sons, one of the first to settle in the
new Plymouth Colony near what is today Boston, and one of the people who signed the Mayflower
Compact —and he was also the colony’s first murderer. Billington, in fact, was the first person to commit
any crime in the colony, as far as we know today. He was also the first to be executed by the state in the
New World. Billington's crime was to shoot a man named Jolin Newcomen, for reasons which are lost to
history. But Billington’s problems had not begun on the soil of North America. He and his sons had nearly
caused a mutiny aboard the Mayflower during the arduous trip across the ocean. One of his sons, in fact,
fired his gun aboard the Mayflower—near an open keg of gunpowder! The flash from the gun could easily
have ignited the powder, which would probably have sunk the ship. Once in Plymouth Colony, Billington’s
behavior did not improve. He refused to seive any form of military duty under the leadership of Miles
Standish, a duty that was seen by the colonists as part of every man’s responsibilities in the New World. He
Page 6
was later implicated in a plot to overthrow the entire leadership of Plymouth Colony, but a lack of evidence
prompted the town’s leaders to let him go free.
hi the end. Billington’s rebellious and angry nature caught up with him. He was found guilty of the
murder of John Newcomen, and died in disgrace on the gallows.

54. Based on the information in the passage, which of the following best describes John Billington?
A. Quiet and shy B. Angry and rebellious C. Clever- and amusing
D. Bold and patriotic E. Happy and joyous

55. Which of the following is NOT hue of John Billington, according to the passage?
A. He had two sons B. He served in the military under Miles Standish
C. He tried to lead a mutiny D. He attempted to overthrow the government
E. He murdered Iris sons

56. Why didn’t the leaders of Plymouth Colony punish Billington for rebelling against their authority?
A. He was not guilty B. Billington was Miles Standish’s nephew
C. He had two sons who needed him D. They didn’t have enough evidence
E. They were planning to blackmail

57. A good title for this passage would be


A. A History of Plymouth Colony B. Tire Injustices of Miles Standish
C. America’s First Murderer D. Early American Legal Battles
E. Mrgrant’s Psychologrcal Issues

Questions 58 to 60 are based on the following instruction:


Instruction: Tire underlined part of the given sentence may contain error. Select the choice that best
replaces the underlined part keeping the original meaning intact. If you think that the original
sentence is the best choice then select A.
58. Winsor McCay, widely regarded as one of America's greatest graphic artists, not only created one of the
most popular Sunday comic strips of the early 20th century, and produced one of the first animated
cartoons.
A. and produced B. but also produced C. producing
D. but also producing E. as well as producing

59. Tire US says it will also target organizations outside Russia which provide support for its military, or its
annexation of Ukrainian territory.
A. which provide support for its military, or its annexation of Ukrainian territory.
B. whom provide support for its military, or its annexation of Ukrainian territory.
C. which provide support for its military, and it is annexation of Ukrainian territory.
D. which provide support for its military, and it annexation of Ukr ainian terr itory.
E. which provide support for its military, and also it’s annexation of Ukrainian territory.

60. Tire monument stood on that site for over eighty years when it was toppled by an earthquake.
A. stood B. had stood C. was standing D. would be standing E. stood there
Page 7
Section 3: Analytical Ability 15x1=15 marks
Instruction: Solve each of the following problems and mark the correct answer on your Answer Sheet
Questions 61 to 65 are based on the following information:
Mr. Sarowar, foreman for Trans Construction Co., is hiring five persons to do wiring and plumbing on a
site. He must have a minimum of two electricians. Nine persons are sent by the union hiring hall:
Shourov, Kishore and Rifat are electricians, while Richard, Aminul, Zubaeer, Emon, Tanvir and Waris
are plumbers. Sarowar is unwilling to hire Emon and Tanvir together, because he knows from past
experience that they fight all the time. Aminul and Zubaeer are buddies and will only work together.
Rifat won’t work with Richard.

61. If Shourov, Kishore and Rifat are hired, the team of plumbers can consist of-
A. Aminul and Zubaeer only
B. Aminul and Zubaeer or Emon and Tanvir
C. Emon and Waris or Tanvir and Waris
D. Aminul and Zubaeer or Emon and Waris or Tanvir and Waris
E. Aminul, Zubaeer and either Emon, Tanvir or Waris

62. Sarowar has the greatest number of choices for hiring as plumbers if the electricians he chooses are -
A. Shourov, Kishore and Rifat B. Shourov and Kishore C. Shourov and Rifat
D. Kishore and Rifat E. Either Shourov or Kishore, plus Rifat

63. If Richard is hired, the other persons hired must be -


A. Shourov, Kishore, Aminul and Zubaeer
B. Shourov, Kishore, Rifat and either Emon, Tanvir or Waris
C. Shourov and Kishore, together with either Aminul and Zubaeer or Emon and Waris
D. Shourov and Kishore, together with either Emon and Tanvir or Tanvir and Waris
E. Shourov and Kishore, together with either Aminul and Zubaeer, Emon and Waris or Tanvir and Waris

64. Sarowar can put together the rest of his crew in the greatest number of different ways if he hires
A. Aminul and Zubaeer B. Rifat C. Emon D. Tanvir E. Waris

65. If Shourov is hired and Kishore is not, which of the following statements must be true?
i. Aminul and Zubaeer are hired. ii. Either Emon or Tanvir is hired, but not both.
A. i only B. ii only C. either i or ii, but not both
D. both i and ii E. neither i nor ii

Questions 66 to 69 are based on the following information:


A variety show producer is interviewing five entertainers in five consecutive interviews. Each entertainer
interviews alone, and only once. The five entertainers are: two singers (a tenor and a soprano), a dancer,
a juggler, and a actor. The interviews must be scheduled according to the following conditions:
Tire two singers cannot interview one after the other.
Tire juggler must interview immediately before a singer.
Tire actor must interview immediately before or immediately after the dancer.
Page 8
66. If the actor interviews first, which of the following must be true?
A. Tire soprano interviews third. B. Tire juggler interviews fourth.
C. Tire tenor interviews fifth. D. Tire soprano interviews sometime earlier than the dancer.
E. The dancer interviews immediately before the tenor.

67. If the tenor interviews first and the soprano interviews fifth, which of the following must be true?
A. The actor interviews sometime after the juggler.
B. Tire actor interviews immediately after the dancer.
C. Tire juggler interviews sometime after the dancer.
D. Tire juggler interviews sometime before the actor.
E. The dancer interviews immediately before the juggler.

68. If the actor, tire soprano, and the juggler interview one after the other, in that order, which of tire
following must be true?
A. Tire actor is the first of the five to interview.
B. Tire soprano is the second of the five to interview.
C. Tire juggler is the third of the five to interview.
D. Tire dancer is the fourth of the five to interview.
E. The tenor is the fifth of the five to interview.

69. If the juggler interviews sometime earlier than the dancer, a singer CANNOT interview in which of tire
following positions?
A. First B. Second C. Third D. Fourth E. Fifth

70. Josh has twenty years of typing experience behind him; therefore, if you are looking for an efficient
typist to enter your data into the new system, you need look no further.
The speaker assumes that:
A. Twenty years of practice ensures typing efficiency
B. Tire type of typing required for the new system is identical to what Josh has been doing
C. Josh's job profile is the best that the new employer is going to get
D. Josh is an outstandingly fast and accurate typist
E. Josh will fit well into the new office

71. Tire committee on sexual discrimination in the workplace has highlighted Supremo Company as a chief
offender. Of the twenty senior executives in the firm, only one is a woman. Aird of the forty junior
executives, only five are female.
Supremo could best defend itself against the charges by showing that:
A. male and female executives at the same level have the same qualifications
B. they pay the same salary to senior men and senior women
C. ten times more men than women apply for jobs with the company
D. the work pressrues and long hours make jobs with the company unattractive to married women
E. all job applicants who were rejected had fewer qualifications than those accepted
Page 9
72. Umana is planning a special birthday dinner for her husband's 30th birthday. She wants the evening to be
memorable, but her husband is a simple man who would rather be in jeans at a baseball game than in a
suit at a fancy restaurant.
Which restaurant below should Umana choose?
A. Alfredo's offers fine Italian cuisine and an elegant Tuscan decor. Patrons will feel as though they've
spent the evening in a luxurious Italian villa.
B. Pancho's Mexican Buffet is an all-you-can-eat family style smorgasbord with the best tacos in town.
C. Hie Parisian Bistro is a four-star French restaurant where guests are treated like royalty. Chef Dilbert
Olay is famous for his beef bourguignon.
D. Marty's serves delicious, hearty meals in a charming setting reminiscent of a baseball clubhouse in
honor of the owner, Marty, a former major league baseball all-star.
E. None of the above

73. The school principal has received complaints from parents about bullying in the school yard during
recess. He wants to investigate and end this situation as soon as possible, so he has asked the recess aides
to watch closely.
Which situation should the recess aides report to the principal?
A. A girl is sitting glumly on a bench reading a book and not interacting with her peers.
B. Four girls are surrounding another girl and seem to have possession of her backpack.
C. Two boys are playing a one-on-one game of basketball and are arguing over the last basket scored.
D. Three boys are huddled over a handheld video game, which isn't supposed to be on school grounds 74.
E. None of the above

74. Introducing a boy, a girl said, "He is the son of the daughter of the father of my uncle." How is the boy
related to the girl?
A. Brother B. Nephew C. Uncle D. Son-in-law E. None of these

75. If A + B means A is the sister of B; A x B means A is the wife of B, A % B means A is the father of B
and A - B means A is the brother of B. Which of the following means T is the daughter of P?
A.PxQ%R+S-T B.PxQ%R-T + S C.PxQ%R + T-S
D. P x Q % R + S + T E. None of these

Part II: Writing Ability [25 marks]


1. Write an Essay on “Corporate Social Responsibility” 15
2. Write a short note by using the words given below and underline those words after using them. Y on may
use a word more than once. Give a suitable title to your note. 10
Water Resource Depletion Future Rainwater | Conserving
Pollution Recycled Freshwater Sustainable Ground water level

Page 10
“In the name of ACCah, the most beneficent, the most merciful”
CAPSTONE
Contact: 01972 277 866, 016 3031 3031 Web: www.capstonebd.com facebook.com/capstonebd

Mock Test for IBA (DU) MBA Admission

INSTRUCTION: Please read carefully! Test Booklet No: 0002


1. The test consists of two pails: I. Multiple Choice Questions (MCQ) and IL Written Ability.
This part to be filled in
2. Total duration of the test will be 120 minutes (90 for Part-I &
30 minutes for Part II). The answer Sheet (For MCQs) will be by the Candidate
taken away after 90 minutes.
Admit No
3. On this Answer Sheet, write your Admit Number and Test
Booklet Number in the grids and your Name and Signature in Name
the boxes provided using a pen. Darken the corresponding
number boxes below the grids using a pen.

4. On this Test Booklet write your Admit Number and Name in the spaces provided using a pen.

5. Give all your answers in the Answer Sheet by completely darkening the lettered box, which represents
the correct answer to that the letter cannot be seen USE Pen ONLY.
@®® @ ©
6. Do rough work anywhere on the rest booklet,
This part to be filled in by the invigilator
except in the spaces provided for written answers.
Do not use the answer sheet or extra paper for Please verify that
Candidate’s Name & Admit Number matches the
rough work.
information on the admit card.
7. For each MCQ, 25% of assigned marks will Candidate’s appearances resembles the photograph
be deducted for each wrong answer. Each on the Admit Card.
unanswered question will earn zero marks.
Admit No
8. Candidates are required to obtain a minimum
acceptable mark in each section and in each Signature of the Invigilator
part.

9. Switch off your mobile phones.

10. Your script will not be examined if you adopt any unfair means.

11. Do not use a calculator or any other electronic devices.

12. Remain seated silently at the end until you are asked to go.

|STOP! DO NOT TURN THIS PAGE UNTIL YOU ARE ASKED TO DO SO.|
For Office Use Only

2 3
Part I: Multiple Choice Questions [75 marks]
Section 1: Language & Communication 25x1=25 marks
Questions 1 to 5 are based on the following instruction:
Instruction: Choose the answer that shows the best punctuation, clarity and meaning for the underlined part
of the sentence. If the sentence is con ect as is, choose E.
1. Forest department officials said that they held several meetings within their office and with the
bridge authority on conservation of the biodiversity after the publication of the gazette.
A. they held several meetings within their office and with the bridge authority on conservation of the
biodiversity after the publication of the gazette.
B. they held several meetings within their office and the bridge authority on conservation of the
biodiversity after the publication of the gazette.
C. they held several meetings within their office and the bridge authority on conservation of the
biodiversity after the publication of the gazette.
D. they held several meetings within their office and with the bridge authority on conservation of the
biodiversity after the publication the gazette.
E. they held several meetings within their office and also with the bridge authority on conservation of
the biodiversity after the publication for the gazette.

2. Before the student could be hired by the company, the student's adviser had to provide a letter of
recommendation.
A. company the students B. company the student's C. company, the students’
D. company the students’ E. conect as is

3 . The workers who would like to work the morning shift should sign their name on this sheet.
A. workers, who would like to work the morning shift
B. workers who would like to work the morning shift,
C. workers, who would like to work the morning shift,
D. workers who, would like to work the morning shift, E. correct as is

4. I must buy mv driver a new license. If I don't, I will have to pay a fine.
A. I must buy my driver a new license, and I will have to pay a fine.
B. I must buy my driver a new license; I will have to pay a fine.
C. Unless I buy my driver a new license, I will have to pay a fine.
D. I will have to pay a fine since I must buy my driver a new license. E. correct as is

5. The Lit fest this weekend promises to attract an even greater amount of people than attended tire last one.
A. an even greater number of people B. an ever larger amount of people
C. an amount of people even greater D. a number of people even larger E. correct as is

Questions 6 to 10 are based on the following instruction:


Instruction: Underlined pails of tire following sentences may contain enor(s) in grammar, diction, usage or
idiom. Choose the underlined portion that contains such error(s). If there is no error then select E.
6. Local hospitals were inundated (A) wounded people and a police station was (B) torched in rioting (C)
that continued (D) overnight. No error (E).
Page: 1
7. The intelligence (A) services issued a lengthy (B) statement on Friday blamed (C) the protest on (D)
foreign agents and terrorists. No error (E).

8. It was decided (A) at the committee meeting that the responsibility to organize (B) the football
tournament should be handed (C) over to Joy and me (D). No error (E).

9. The extraction should be done (A) following (B) the swing method to maintain (C) the
uniform (D) level of the river bed. No error (E).

10. Part of the problem with (A) accounting for the bank's climate finance activities is that (B) many
projects have a climate-related component, without being (C) mainly geared towards (D) the climate.
No error (E).

Questions 11 to 16 are based on the following instruction:


Instruction: Fill in the gaps with suitable word from the alternatives.
1 1. By gradually winning the support of both liberals and conservatives, both rich and poor, the governor
has demonstrated that her remarkable skills go side by side with her willingness to speak openly
and on controversial issues.
A. interpersonal . . . equivocally B. diplomatic . . . frankly C. organizational . . . covertly
D. personal . . . deceptively E. intimidating . . . obscurely

12. Again and again, out of indifference or sheer stupidity, we have our resources, assuming that
there was no end to the earth’s to recover from our mistakes.
A. invested . . . resolve B. expanded . . . ability C. wasted . . . failure
D. husbanded . . . inability E. squandered . . . capacity

13. Despite crores of taka spent on developments, the transportation system in Congo remains and
continues to the citizens who depend on it.
A. bombastic .... upset B. suspicious ....joint C. impertinent ....vex
D. primitive .... inconvenience E. outdated .... elate

14. When the Broker finally suggested a plot, the newly married couple could , they were outrageous

and shocked to witness a house that seemed as though it were on the verge of collapse.
A. design .... reserve B. purchase .... falling C. afford .... ramshackle
D. repair .... new E. modify .... dilapidated

15. Because of his fundamental . Shovon steered clear of any job that he suspected could turn out to
be an agony.
A. indolence B. insolence C. eminence D. impudence E. integrity

16. Tire policy makers of the National Democratic League vowed to all evidences of between
politicians and Criminals.
A. eradicate .... collusion B. exterminate .... controversy C. investigate .... contact
D. uproot .... enmity E. make .... connection
Page: 2
Questions 17 to 19 are based on the following instruction:
Instruction: Find out the closest meaning of the following underlined part.
17. A fair weather friend always hies to fish in troubled waters of his friends and benefactors.
A. to borrow money B. to steal belongings of C. to get benefit in bad situation
D. to extend a helping hand. E. to leave in intense situation

1 8. The Director of the corporation plays a second fiddle to the CEO.


A. To extend a helping hand B. To play a secondary role C. To be guiding person
D. To be facilitator E. To be a decision maker

1 9. He is plain, simple and sincere man. He will always call a spade a spade.
A. Find meaning or purpose in your action. B. Avoid controversial situations
C. Be outspoken D. Desist from making confroversial statement
E. Making vague comments

Questions 20 to 25 are based on the following comprehension:


Instruction: Read the following passage and answer the accompanying questions, base your answer on
what is stated or implied in the passage. Mark your answers on the Answer Sheet.
hi 1955 Maurice Duverger published The Political Role of Women, the first behavior list, multinational
comparison of women’s electoral participation ever to use election data and survey data together. His study
analyzed women’s patterns of voting, political candidacy, and political activism in four European countries
dining the first half twentieth century.
Duverger’s research findings were that women voted somewhat less frequently than men (the difference
narrowing the longer women had the vote) and were slightly more conservative. Durverger's work set an
early standard for the sensitive analysis of women’s electoral activities. Moreover, to Duverger’s credit, he
placed his findings in the context of many of the historical processes that had shaped these activities.
However, since these contexts have changed over time. Duverger’s approach has proved more durable than
his actual findings, hi addition, Duverger’s discussion of his findings was hampered by his failure to
consider certain specific factors important to women's electoral participation at the time he collected his
data: the influence of political regimes, the effects of economic factors, and the ramifications of political and
social relations between women and men. Given this failure. Duverger’s study foreshadowed the enduring
limitations of the behavior list approach to the multinational study of women's political participation.

20. The primary purpose of the passage is to -


A. Evaluate a research study
B. Summarize the history of a research area
C. Report new research findings
D. Reinterpret old research findings
E. Reconcile conflicting research findings
Page: 3
21. According to the passage, Duverger's study was unique in 1955 in that it
A. included both election data and survey data
B. gathered data from sources never before used in political studies
C. included an analysis of historical processes
D. examined the influence on voting behavior of the relationships between women and men
E. analyzed not only voting and political candidacy but also other political activities

22. Which of the following characteristics of a country is most clearly an example of a factor that Duverger,
as described in the passage, failed to consider in his study?
A. A large population B. A predominantly Protestant population
C. A predominantly urban population D. A one-party government
E. Location in the heart of Europe

23. The author implies that Duverger’s actual findings are -


A. limited because they focus on only four countries
B. inaccurate in their description of the four countries in the early 1950’s
C. out-of-date in that they are inapplicable in the four countries today
D. flawed because they are based on unsound data
E. blazed by Duverger’s political beliefs

24. The passage implies that, in comparing four European countries, Duverger found that the voting rates of
women and men were most different in the country in which women
A. were most politically active B. ran for office most often
C. held the most conservative political views D. had the most egalitarian relations with men
E. had possessed the right to vote for the shortest time

25. The author implies that some behavioralist research involving the multinational study of women’s
political participation that followed Duverger’s study did which of the following?
A. Ignored Duverger's approach B. Suffered from faults similar to those in Duverger’s study
C. Focused on political activism D. Focused on the influences of political regimes
E. Focused on the political and social relations between women and men

Section 2: Mathematics 30x1=30 marks


Instructions: Solve each of the following problems and mark the conect answer on your Answer Sheet.
DO NOT USE A CALCULATOR. Figures are not drawn to scale.
26. Two numbers when divided by a certain divisor give remainder 35 and 30 respectively and when their
sum is divided by the same divisor, the remainder is 20, and then the divisor is what?
A. 40 B. 45 C. 50 D. 55 E. None of these

+65x + 60 _ 5x+5 js
27. If fouowjng not a possible value of x?
x2+10x-24 x— 2
A. -36 B. 12 C. -214 D. 143 E. 2
Page: 4
28. hi a survey it was found that 10% people don't use Facebook, Twitter or WliatsApp. 8% uses all the
three. There are 15% who uses only Facebook and Twitter, 20% who uses only Twitter and WliatsApp
and 20% who uses only Facebook and WliatsApp. Number of people that use only Facebook, only
Twitter and only WhatsApp are equal. If the survey was conducted on 1000 people, what is the ratio of
number of people that uses WhatsApp only to the people using either WhatsApp or Facebook or both?

D. - E. None of these
9

29. A triangle is formed by connecting three randomly chosen vertices of a hexagon. What is the probability
that at least one of the sides of the triangle is also a side of the hexagon?

A A
10
D. -
8
E. None of these

30. If a2 + 7a < 0, then which one of the following could be the value of a?
A. -3 B. 0 C. 1 D. 2 E. 3

31. If the numerator of a certain fraction is increased by 2 and the denominator is increased by 1, then the

resulting fraction is equal to the . If however the numerator is increased by 1, then denominator is

decreased by 2, then the resulting fraction is equal to —3 . Find the original fraction
C. - D. - E. None of these
7 7 5

32. Arif & Pulok begin together to write out a booklet containing 817 lines. Arif starts with first line, writing
at rate of 200 lines an how and Pulok starts with the last line. He writes line 817 and so on backwards
proceeding at the rate of 150 lines an how. At what line will they meet?
A. 466“’ line B. 465th line C. 467*’ line D. 468,hline E. None of these

N-2
|x-2| =
33. If 1, then which of the following must be true?

I. |x| >2 n. x2 < i III. x3 > i


A. I only B. Ill only C. I and II only D. I and III only E. None of these

34. On mixing two classes A and B of students having average marks 25 and 40 respectively, the overall
average obtained is 30. Find the ratio of the students in classes A and B.
A. 2:1 B. 5:8 C. 5:6 D. 3:4 E. None of these

35. A boy on being asked 12 of a certain fraction had made the mistake of dividing the fraction by 12 and so
14 14

got an answer that exceeded the conect answer by _2_ . The correct fraction is?
65

A. 12 B. 12 C. 12 D. 2 E. None of these
15 65 45 7

36. Shahadat and Omor working together can complete a work in 24 days. If for the last 6 days, Shahadat alone
does the work, then it is completed in 26 days. How long will Omor take to complete the work alone?
A. 72 B. 73 C.49 D. 62 E. None of these
Page: 5
37. The price of an article reduces to Tk. 576 after two successive discounts. The markup is 80% above the
cost price of Tk. 5 00.What is the new profit percentage if instead of two successive discount the markup
price was further increased successively two times by the same percentage?
A. 259.2% B. 59.2% C. 159.2% D. 196.5% E. None of these

38. Machine A and machine B are each used to manufacture 660 sprockets. It takes machine A 10 hours
longer to produce 660 sprockets than machine B. Machine B produces 10% more sprockets per hour
than machine A. How many sprockets per hour do machine A produce?
A. 6 B. 6.6 C. 60 D. 100 E. 110

39. Six bells commence tolling together and toll intervals of 2, 4, 6, 8, 10 and 12 seconds respectively, hr 30
minutes, how many times do they toll together?
A. 4 B. 10 C. 15 D. 16 E. None of these

40. If 4 < , which of the following must be true?

I. 5 < x n. |x+3| > 2 III. - (x+5) is positive


A. II only B. Ill only C. I and II only D. II and III only E. None of these

41. The radius of each circle is ‘a’. Then the area of the shaded portion is what?
A. a2(V3
D. 3 (a —
— B. a (na2 - \/3)

E. None of these
C.V3^a2—
42. From the figure, which of the following must be true?
I. x + y = 90 II. x is 35 units greater than y III. x is 35 units less than y
A. I only B. II only C. Ill only
D. I and II only E. I and III only

43. Maliha, Shanta and Rahnuma shared a box of cookies. Maliha ate - of the cookies, Shanta ate one half
8
and Rahnuma ate 150 more cookies than Maliha. If no cookies remain in the box, how many cookies
were in the box?
A. 1200 B. 800 C. 600 D. 550 E. 400

44. hr a river, the ratio of the speed of stream and speed of a boat in still water is 2 : 5 Again, ratio of the
speed of stream and speed of an another boat in still water is 3 : 4. What is the ratio of the speeds of the
first boat to the second boat in still water?
A. 10:7 B. 15:8 C. 4:3 D. 5:4 E. None of these

45. Tazul and Sadib undertake to do a piece of work for Tk. 800. Tazul alone can do it in 12 days, while
Sadib alone can do it in 16 days. With the assistance of Abir, they finish it in 6 days. How should the
money (Tk.) be divided into Tazul, Sadib and Abir?
A. 100, 300, 400 B. 200, 200, 400 C. 400, 300, 100 D. 300, 400, 100 E. None of these
Page: 6
46. Mukit buys a single apple for Tk. 25. If he were to buy a dozen apples, he would have to pay a total
amount of Tk. 250. What would be the approximate % discount he would get on buying a dozen apples?
A. 32 B. 20 C. 12 D. 17 E. None of these

47. Tire ratio of incomes of Ali and Anik is 5: 4 and the ratio of their expenditure is 4: 3. If Ali saves tk. 500
per month, what is the income of Anik?
A. tk. 2,500 B. tk. 2,000 C. tk. 1,500 D. tk. 1,000 E. None of these

48. hi the figure CD = 2 AB and AB is parallel to CD. If the area of ABE is 3, what is the A^ 76
areaofCED?
A. 2 B 6 C. 9
D. 12 E. None of these i?

49. If the length of each side of an equilateral triangle was increased by 50%, what will be the increase in the area?
A. 75% B. 225% C. 150% D. 125% E. 75%

50. A bus is traveling with 60 passengers. When it arrives at a stop, x number of passengers get off and 8 get
on. At the next stop, one third of the passengers on board get off and 5 get on. There are now 37
passengers on the bus. Find x.
A. 15 B. 20 C. 25 D. 30 E. None of these

51. Mixhire X is 40 percent peanut and 60 percent hazel nut by weight; mixhire Y is 25 percent peanut and
75 percent hazelnut. If a mixture of X and Y contains 30 percent peanut, what percent of the weight of
this mixhu e is X?
A. 10% B. 33.33% C.40% D. 50% E. 66.66%

52. A pipe can fill a cistern in 12 minutes and another pipe can fill it in 15 minutes, but a third pipe can
empty it in 6 minutes. The first two pipes are kept open for 5 minutes in the beginning and then third
pipe is also opened, hr what time is the cistern emptied?
A. 30 min B. 33 min C. 37.5 min D. 45 min E. None of these

53. A bus travels from A to B at a constant speed, at which it should reach B in x minutes. After 30 km, due
to an engine malfunction, the speed of the bus decreases to - of the original speed. Because of that, the
bus reaches B in x + 45 minutes, instead of intended x minutes. Had the same malfunction happened
after the bus travelled 48 km, it would have reached B, in x + 36 minutes, instead of intended x minutes.
What is the distance between A and B?
A. 120 km B. 90 km C. 60 km D. 40 km E. None of these

54. Ashique is painting a rectangular room whose dimensions are given by a, b and c meters. Ashique takes
8 hours to paint a wall with dimensions a and c. He takes 4 hours to paint a wall with dimensions b and c
and 12 horns to paint the ceiling with dimensions a and b. If Ashique works at a constant rate and a = 6,
then what is die volume of the room?
A. 18 m3 B. 24 m3 C. 30 m3 D. 36 m3 E. None of these

55. A train P start from Parbatipur at 5:00 pm reached Sayedpur at 6:00 pm. Another train Q starts from
Sayedpur at 5:00 pm and reaches Parbatipur at 6:30 pm. At what time, two trains will cross each other?
A. 5:36 pm B. 4 : 36 pm C. 6:10 pm D. 6:15 pm E. None of these
Page: 7
Section 3: Analytical Ability 20x1=20 marks
Instruction: Solve each of the following problems and mark the correct answers on your Answer Sheet.
Questions 56 to 58 are based on the following information:
A. Statement (i) alone is sufficient but statement (ii) alone not sufficient to answer the question asked.
B. Statement (ii) alone is sufficient but statement (i) alone not sufficient to answer the question asked.
C. Both statement (i), (ii) together are sufficient to answer the question but neither statement is sufficient alone.
D. Each statement alone is sufficient to answer the question.
E. Statement (i) & (ii) are not sufficient to answer the question asked and additional data is needed to
answer the statements.

56. A number consist of two digits whose sum is 10 if the digits of the number are reversed then the number
decreased by 36. Which of the following is correct?
i. The number is divisible by a composite number. ii. The number is a multiple of a prime number.

57. A team of 2 is to be selected from a group of 5. If the number of males in the group is n, then what's the
value of n?
i. The number of males in the group is one more than the number of females
.9
ii. The probability the team consists of at least one male is —
..
58. If yz not equal to zero is 0 <y < 1? i. y< —y1 ii. y = z2

59. Any driver involved in an accident leading to personal injury or property damage exceeding S500 is
legally required to report the accident to the department of motor vehicles, unless the driver is incapable
of doing so. Siam is not required to report the accident in which he was involved as a driver.
Which one of the following can be properly inferred from the statements above?
A. If Siam is incapable of reporting the accident, then the accident did not lead to property damage
exceeding S500
B. If Siam's car was damaged in excess of S500 in the accident, then he is incapable of reporting the
accident to the department of motor vehicles
C. Someone other than Siam is legally required to report the accident to the department of motor vehicles
D. If Siam is incapable of reporting the accident to the department of motor vehicles, then he was
injured in the accident
E. Either no one was injured in the accident or the accident did not lead to property damage exceeding $500

60. In the United States, injuries to passengers involved in automobile accidents are typically more severe than in
Europe, where laws require a different kind of safety belt. It is clear from this that the United States needs to
adopt more stringent standards for safety belt design to protect automobile passengers better.
Each of the following, if true, weakens the argument above EXCEPT:
A. Europeans are more likely to wear safety belts than are people in the United States.
B. Unlike United States drivers, European drivers receive training in how best to react in the event of an
accident to minimize injuries to themselves and to their passengers.
C. Cars built for the European market tend to have more sturdy construction than do cars built for the
United States market.
D. Automobile passengers in the United States have a greater statistical chance of being involved in an
accident than do passengers in Europe.
E. States that have recently begun requiring the European safety belt have experienced no reduction in
the average severity of injuries suffered by passengers in automobile accidents.
Page: 8
Questions 61 to 65 are based on the following information:
A woman plans to plant exactly six kind of trees: orange, apple, mango, banana, lemon, and date. She
places six pots side by side in a straight line and numbers the pots consecutively from 1 to 6, left to right.
She will plant only one kind of tree in each pot. Tire arrangement of the trees is subject to the following
conditions:
Orange must be planted in some pot to the left of banana.
Lemon must be planted in some pot to the left of date.
Apple cannot be planted in pot 1.
Mango must be planted next to orange;

61. Which of the following arrangements of trees from pot 1 through 6, respectively, conforms to the
conditions above?
A. Date, orange, mango, lemon, banana, apple B. Apple, lemon, date, mango, orange, banana
C. Lemon, apple, date, banana, mango, orange D. Orange, banana, lemon, date, apple, mango
E. Mango, orange, lemon, apple, banana, date

62. If apple is planted in some pot to the right of banana, which of the following must be true?
A. Apple is planted in some pot to the right of orange.
B. Apple is planted in some pot to the right of lemon.
C. Apple is planted in some pot to the right of date.
D. Banana is planted in some pot to the left of lemon.
E. Banana is planted in some pot to the left of mango.

63. If date is planted in some pot to the left of orange, which of the following must be tme?
A. Date is planted in some pot to the left of apple.
B. Date is planted in some pot to the left of mango.
C. Orange is planted in some pot to the left of apple.
D. Orange is planted in some pot to the left of mango.
E. Orange is planted in some pot to the left of lemon.

64. If banana is planted in some pot to the left of lemon, lemon could be planted in which of the following pots?
A. 1 B. 2 C. 3 D. 4 E. 6

65. If orange is planted next to date, which of the following must be true?
A. Lemon is planted in pot 1 B. Apple is planted in pot 2
C. Mango is planted in pot 3 D. Orange is planted in pot 4 E. Date is planted in pot 5

Questions 66 to 70 are based on the following information:


All mails in a large province are transmitted by means of a mail network that connects eight cities F, —
G, H, I, J, K, L, and M. All of the connections are two-way, so that mails can be sent back and forth
between any two connected cities. Any given mail enters, and leaves a given city at most once. Tire
cities are connected only in the following way:
F is connected to G and to I. H is connected to G and to I.
J is connected to H and to L. K is connected to H, to L, and to M.
Page: 9
66. The path followed by mails sent from L to M must include which of the following cities?
A. F B. G C. H D. J E. K

67. Mails sent from F to J can go along any of the following paths EXCEPT
A. F to G to H to J B. F to I to H to J C. F to G to H to I to J
D. F to G to H to K to L to J E. F to I to H to K to L to J

68. Which of tire following specifies in its entirety a sequence, from first to last, of cities through which a
mail from M to I can pass?
A.M, F, I B.M, K, F, I C. M,K,L, I D. M, K, H, G, I E. M, K, L, J, H, I

69. What is tire minimum number of cities, excluding the originating and destination cities, through which a
mail from I to K must pass?
A. One B. Two C. Three D. Four E. Five

70. Disconnecting which of the following cities from every city to which it is connected would leave exactly
one of the other cities unconnected to the rest of the mail network?
A.F B. G C. J D. K E.M

Questions 71 to 75 are based on the following information:


Seven pieces of table lamps with different shades- A, B, C, D, X, Y & Z- are to be arranged along one
wall of a show room for display. Each lamp must be placed along the wall at one of seven electrical
outlets, numbered one through seven consecutively, according to the following conditions:
A must be placed at an outlet adjacent to the outlet at which X is placed.
B must be placed at an outlet that is adjacent to an outlet at which either D or Y is placed.
Z cannot be placed at an outlet adjacent to the outlet at which C is placed.
C must be placed at outlet number two.

71. If Y is placed at outlet five, D must be placed at outlet


A. One B. Three C. Four D. Six E. Seven

72. If B is placed at outlet seven, which o the following must be true?


A. Y is placed at outlet six B. Z is placed at outlet five C. X is placed at outlet four
D. A is placed at outlet three E. D is placed at outlet one

73. Which of the following is an acceptable arrangement of lamps at outlets one through seven?
A. A, X, B, Y, Z, D, C B.B, D,A,X,C,Y,Z C. X, C, A, Z, D, B, Y
D. Y, C, B, D, Z, A, X E. Y, C, Z, D, B, A, X

74. If A is placed at outlet seven, which of the following must be true?


A. X is placed at outlet five B. B is placed at outlet three C. Z is placed at outlet five
D. D is placed at outlet four E. Y is placed at outlet one

75. Z can be placed at which of the following outlets?


A. One B. Three C. Four D. Six E. Seven
Page: 10
Part II: Writing Ability [25 marks]
1 . Write a paragraph using the following words. You must give an appropriate title and CAN NOT change
tire formation of the following words. You must underline the following words when you use. 10
Deforestation contamination biodiversity agricultural land fossil fuels
fertilizer Poverty population respiratory diseases underground

2. ‘It is necessary to implement carbon taxes to combat climate change’- Do you support the statement. If
you do so, show your logics. 15

Page: 11
“In the name of .ACOih, the most beneficent, the most merciful”
CAPSTONE
Contact: 01972 277 866, 016 3031 3031 Web: www.capstonebd.com facebook.com/capstonebd

Mock Test for IBA (DU) MBA Admission

INSTRUCTION: Please read carefully! Test Booklet No: 0003


1. The test consists of two parts: I. Multiple Choice Questions (MCQ) and IL Written Ability.
This part to be filled in
2. Total duration of the test will be 120 minutes (90 for Part-I &
30 minutes for Part II). The answer Sheet (For MCQs) will be by the Candidate
taken away after 90 minutes.
Admit No
3. On this Answer Sheet, write your Admit Number and Test
Booklet Number in the grids and your Name and Signature in Name
the boxes provided using a pen. Darken the corresponding
number boxes below the grids using a pen.

4. On this Test Booklet write your Admit Number and Name in the spaces provided using a pen.

5. Give all your answers in the Answer Sheet by completely darkening the lettered box, which represents
the correct answer to that the letter cannot be seen USE Pen ONLY.
® @
6. Do rough work anywhere on the rest booklet,
This part to be filled in by the invigilator
except in the spaces provided for written answers.
Do not use the answer sheet or extra paper for Please verify that
rough work.
Candidate’s Name & Admit Number matches the
7. For each MCQ, 25% of assigned marks will information on the admit card.
be deducted for each wrong answer. Each
unanswered question will earn zero marks. Candidate’s appearances resembles the photograph
on the Admit Card.
8. Candidates are required to obtain a minimum Admit No
acceptable mark in each section and in each Signature of the Invigilator
part.

9. Switch off your mobile phones.

10. Your script will not be examined if you adopt any unfair means.

11. Do not use a calculator or any other electronic devices.

12. Remain seated silently at the end until you are asked to go.

|STOP! DO NOT TURN THIS PAGE UNTIL YOU ARE ASKED TO DO SO.|
For Office Use Only

I 1 I 2 I ^~~l
Part I: Multiple Choice Questions [70 marks]
Section 1: Analytical Ability 15x1=15 marks
Instruction: Solve each of the following problems and mark the correct answer on your Answer Sheet.
Questions 1, 2 are based on the following information:
A. Statement (i) alone is sufficient but statement (ii) alone not sufficient to answer the question asked.
B. Statement (ii) alone is sufficient but statement (i) alone not sufficient to answer the question asked.
C. Both statement (i), (ii) together are sufficient to answer the question but neither statement is sufficient alone.
D. Each statement alone is sufficient to answer the question.
E. Statement (i) & (ii) are not sufficient to answer the question asked and additional data is needed to
answer the statements.

1 . hi a single row of yellow, green and red colored tiles, eveiy red tile is preceded immediately by a yellow
tile and eveiy yellow tile is preceded immediately by a green tile. What color is the 24th tile in the row?
i. The 18th tile in the row is not yellow.
ii. Tire 19th tile in the row is not green.

2. When positive integer n is divided by 7, the remainder is 1 . If n is less than 50, what is the value of n?
i. When n is divided by 9, the remainder is 7.
ii. When n is divided by 4, the remainder is 3.

3 . If A + B means A is the brother of B; A - B means A is the sister of B and A x B means A is the father of
B. Which of the following means that C is the son of M?
A.M-NxC + F B.F-C + NxM C.N + M-FxC
D. M xN - C + F E. None of these

4. Some good cooks are gourmet cooks who pride themselves on always using extravagantly rich
ingredients in elaborate recipes. Some good cooks can be characterized as fast-food cooks. They may
use rich ingredients as long as the recipes are easy to follow and take little time. Other good cooks are
health food enthusiasts, who are concerned primarily with the nutritional value of food. But even though
not all good cooks are big eaters, they all enjoy preparing and serving food.
If the information in the passage is true, which one of the following CANNOT be true?
A. Most good cooks do not use extravagantly rich ingr edients.
B. Everyone who enjoys preparing and serving food is a good cook.
C. More good cooks who use extravagantly rich ingr edients are big eaters than are good cooks who do
not use such ingredients.
D. There are fewer good cooks who enjoy serving and preparing food than there are good cooks who
are big eaters.
E. Gourmet cooks, fast-food cooks, and cooks who are health food enthusiasts are all big eaters.
Page: 1
5. Plankton generally thrive in areas of the ocean with sufficient concentrations of certain nitrogen
compounds near the surface where plankton live. Nevertheless, some areas, though rich in these nitrogen
compounds, have few plankton. These areas have particularly low concentrations of iron, and
oceanographers hypothesize that this shortage of iron prevents plankton from thriving. However, an
experimental release of iron compounds into one such area failed to produce a thriving plankton
population, even though local iron concentrations increased immediately.
Which of the following, if true, argues most strongly against concluding, on the basis of the
information above, that the oceanographers hypothesis is false?
A. Not all of the nitrogen compounds that are sometimes found in relatively high concentrations in the
oceans are nutrients for plankton.
B. Certain areas of the ocean support an abundance of plankton despite having particularly low
concentrations of iron.
C. Tire release of the iron compounds did not increase the supply of nitrogen compounds in the area.
D. A few days after the iron compounds were released, ocean currents displaced the iron-rich water
from the surface.
E. The iron compounds released into the area occur naturally in areas of the ocean where plankton thrive.

Questions 6 to 10 are based on the following information:


A doctor has prescribed a meal program for a patient. Choosing from meals F, G, H, I, J, K, L, and M,
the patient must take a routine of exactly five different meals each day. hi any day routine, except the
first, exactly three of the meals must be ones that were included in the routine done on the previous day,
and any permissible routine must also satisfy the following conditions:
If F is in a routine, L cannot be done in that routine.
If G is in a routine, J must be one of the meals done after G in that routine.
If H is in a routine, L must be one of the meals done after H in that routine.
The fifth meal of any routine must be either I or K.

6. Which of the following could be the routine for the first day of the progr am?
A.F,H,L,I,K B.G, I,H,L, K C.J,K,H,L,I D.K, G,I, J,M E. L, G, H, J. I

7. If one day's routine is F, G, M, J, K, each of the following could be the next day's routine EXCEPT-
A. G, H, L, J,K B.G, J, L, M, I C. M, H, L, J. K D. M, J. K, L, I E. M, J, I, F, K

8. Which of the following is true for any permissible routine?


A. F cannot be done third. B. G cannot be done third. C. J cannot be done third.
D. H cannot be done fourth. E. K cannot be done fourth.

9. If the patient chooses H and M for the first day's routine, which of the following could be the other three
meals chosen?
A. F, J, K B.G, I, L C. G, J, L D. J, I, K E. J, I, L

10. If H is the third meal in a routine, which of the following CANNOT be the second meal in that routine?
A. G B. I C. J D.K E. M
Page: 2
Questions 11 to 15 are based on the following information:
The bar graph given below shows the sales of books (in thousand numbers) from six branches of a
publishing company during two consecutive years 2000 and 2001.
Sales of Books (in thousand numbers) from Six Branches-
Bl, B2, B3, B4, B5 and B6 of a publishing Company in 2000 and 2001.

11. What is the ratio of the total sales of branch B2 for both years to the total sales of branch B4 for both years?
A. 2:3 B. 3:5 C. 4:5 D. 7:9 E. None of these

12. Total sales of branch B6 for both the years is what percent of the total sales of branch B3 for both the year's?
A. 68.54% B. 71.11% C. 73.17% D. 75.55% E. 88.67%

13. What percent of the average sales of branches Bl, B2 and B3 in 2001 is the average sales of branches
B1,B3 and B6 in 2000?
A. 75% B. 77.5% C. 82.5% D. 87.5% E. 51.23%

14. What is the average sale of all the branches (in thousand numbers) for the year 2000?
A. 73 B.80 C. 83 D. 88 E. None of these

15. Total sales of branches B 1 , B3 and B5 together for both the years (in thousand numbers) is?
A. 250 B.310 C.435 D. 560 E. None of these

Section 2: Mathematics 25x1=25 marks


Instructions: Solve each of the following problems and mark the correct answer on your Answer Sheet.
DO NOT USE A CALCULATOR. Figures are not drawn to scale.
16. p and q are integers. If p is divided by 2, the remainder is 1; and if q is divided by 6, the remainder is 1.
Which of the following must be true?
I. pq + 1 is even. II. is an integer. III. pq is a multiple of 12.
A. I only B. II only C. Ill only D. I and II only E. I and III only
Page: 3
17. Avik is twice as efficient as Tanvir and Tanvir is thrice as efficient as Shahadat. If all three together can
complete a work in 10 days, find the difference in the number of days taken to complete the work when
Avik and Tanvir work together and when Tanvir and Shahadat work together.
A. 25 B —
IP'S
6
C.—
7
IP'S
D. —
IP'S
8
E. None of these

18. What is the largest integer n such that 2n is a factor of 208?


A. 1 C. 4 D. 8 E. 16

1 9. A car manufactur er tripled the price of a car and by this quadrupled percentage of profit on it. What was
the original profit percentage on the car?
A. 33.33% B. 66.66% C. 100% D. 200% E. None of these

20. There is a circular track with a circumference of 500 meters; Foyez & Ababil are running anticlockwise
around the track. Foyez runs 1000 meters every five minutes and Ababil runs 1000 meters every six
minutes. If Foyez and Ababil start opposite one another on the circular track, and run in the same
direction how many minutes must Foyez run in order to pass Ababil and catch him again?
A. 7.5 B. 22.5 C. 750 D. 7.5n E. 45n

21. How many integers in the set of all integers from 10 to 150 (all inclusive) are not the square of an integer?
A. 101 B. Ill C. 121 D. 132 E. None of these

22. A clock is started at 6 O'clock in the morning. Through how many degrees will the hour hand rotate
when the clock shows 11 O'clock in the morning?
A. 140° B. 180° C. 150° D. 160° E. None of these

23. Out of 100 families in the neighborhood, 50 have radios, 75 have TVs and 25 have VCR. Only 10
families have all three and each VCR owner also has a TV. If some families have radio only, how many
have only TV?
A. 30 B.35 C.40 D. 45 E. None of these

24. hi an election only two candidates contested. 20% of the voters did not vote and 120 voters were
declared as invalid. The winner got 200 votes more than his opponent, thus he secured 41% votes of the
total voters on the vote list. Percentage of votes of the defeated candidate out of total votes casted is?
A. 36% B. 41% C.42% D. 45% E. 48.25%

25. Pial, Tayef and Fahini had lunch together. Fahim’s meal cost 50% more than Tayef s meal and Pial’s
meal cost|as much as Fahim’s meal. If Tayef paid tk. 1,000 for his meal, what was the total amount
they paid for lunch?
A. 2833 B. 3000 C. 3500 D. 3750 E. None of these

26. A pet shop runs a sale on goldfish and platys. If Alimul buys 13 goldfish and 12 platys for Tk.5.60, and
Avik buys 14 goldfish and 21 platys for Tk.8.05, what is the ratio of the price of goldfish to platys?
A. 0.8 B.0.75 C.-9 D. -7 E. None of these
Page: 4
27. If m and 11 are the roots of x2-7|x|-18=0, then what is the value of | m - n | ?
A. 0 B. 15 C. 18 D. 9 E. None of these

28. 100 people are attending a newspaper conference. 45 of diem are writers and more than 38 are editors.
Of the people at the conference, x are both writers and editors and 2x are neither. What is die largest
possible number of people who are both writers and editors?
A. 6 B. 16 C. 17 D. 33 E. None of these

29. There are 8 teams in a certain league and each plays the other team exactly once. If each game is played
by two teams, what is the total number of game played?
A. 15 B. 28 C. 16 D. 58 E.64

30. For how many integer values of x, | x - 6 1>| 3x + 6 1 ?


A. 1 B. 3 C. 5 D. 7 E. None of these

3 1 . The average age of two boys and their father is greater than average age of those two boys and their

1
mother by 3 yr. The average age of the four is 19 yr. If the average age of the two boys be 5 , then find
the age of the father and mother.
A. 37 yr & 28 yr B. 47 yr & 38 yr C. 50 yr & 41 yr D. 35 yr & 32 yr E. None of these

32. 21 mango trees, 42 apple trees and 56 orange trees have to be planted in rows such that each row
contains the same number of trees in one variety only. Minimum number of rows in which the above
trees may be planted is?
A. 15 B. 17 C. 3 D. 20 E. None of these

33. If x2 > y > z2 , then which of die following could be true?


I. x>yandy = z II. y>x>z III. y>z>x
A. Only II B. I and II C. II and III D. All the three E. None of these

34. Sayeda has a wheat business. She purchases wheat from a local wholesaler at a particular cost per pound.
The price of the wheat at her stores is S3 per pound. Her faulty spring balance reads 0.9 pounds for a
pound. Also, in the festival season, she gives a 10% discount on the wheat. She found diat she made
neither a profit nor a loss in the festival season. At what price did Sayeda purchase the wheat from the
wholesaler?
A. 2.43 B. 2.5 C. 2.7 D. 3 E. 3.3

35. Alvie must answer 7 out of 10 questions on an examination. If at least 3 of the first 5 questions must be
answered, how many possible combinations of 7 questions are available to Alvie?
A. 50 B. 80 C. 110 D. 220 E. None of these

36. The current of a stream runs at 1 km/hr. A motor boat goes 35 km upstream and back again to the
starting point in 12 horns. The speed (km hr) of motor boat in still water is?
A. 6 B.7 C. 8 D. 8.5 E. None of these
Page: 5
37. hi the figure, BD = 2DC and AD = DB, What is the value of AB?
A. 7(BD2 + DC2) B . V3B D
C. 3BC + DC D. 2BC
E. None of these

38. A certain fruit stand sold apples for tk. 7 each and bananas for tk. 5 each. If a customer purchased both
apples and bananas from the stand for total tk. 63, what is the total number of fruits (apples and bananas)
the customer bought?
A. 10 B. 11 C. 12 D. 13 E. 14

39. hi a subway station, there is a long escalator. Two brothers have to get to a baseball game and are in a
hurry, and so they run up the moving steps, adding their speed to that of the escalator. The taller boy
climbs three times as quickly as his little brother, and while he runs up he counts 75 steps. The little one
counts only 50. How many steps does the visible pail of this escalator have?
A. 50 B.75 C. 100 D. 150 E. None of these

40. One day, Mr. Niaz started 30 minutes late from home and reached his office 50 minutes late, while
driving 25% slower than his usual speed. How much time in minutes does Niaz usually take to reach his
office from home?
A. 40 B.20 C. 80 D. 60 E. None of these

Section 3: Language & Communication 30x1=30 marks


Questions 41 to 45 are based on the following instruction:
Instruction: Each sentence below has one or two blanks, each blank indicating that something has been omitted.
Beneath the sentence are five words or sets of words labeled A thr ough E. Choose the word or set of
words that, when inserted in the sentence, best fits the meaning of the sentence as a whole.

41. Our youth and others have to drugs not only because their friends social pressures but
also because escape with dings is so complete and swift.
A. clung, put B. tinned, enforce C. given, exhibit D. moved, exert E. resorted, apply

42. But even Mr. Saumik, one of the most public servants of our time, is at a loss to convey in words
the size, the imaginative engineering that built this contribution to the welfare of family and
industry.
A. tongue-tied, miracle B. dedicated, oppositeness C. unappreciated, technique
D. public-spirited, skill E. articulate, ingenuity

43. Among the younger people there are complaints that the sight of ex-Nazis flourishing recommends
youth, that it instills, instead of needed moral values, the dubious percept that is the
best policy.
A. precepts, intolerance B. desperation, dishonesty C. emulation, honesty
D. cynicism, expediency E. automatation, simultaneity
Page: 6
44. The seminar was organized by the Embassy of Bangladesh in with the ministry of foreign
affairs of Bangladesh.
A. addition B. venturing C. collaboration D. participation E. None of these

45. But if you disregard the law of Empowerment, Your potential will be .
A. destroyed B. gone C. limited D. ended E. feigned

Questions 46 to 48 are based on the following instruction:


Instruction: Choose the conect sentence from the given options.

46. A. It is a pleasure to see an alligator basking in the sunshine on a river bank as long as 90 feet.
B. It is a pleasure to see an alligator as long as 90 feet basking in the sunshine on a liver.
C. It is a pleasure to see an alligator basking in the sunshine as long as 90 feet on a liver bank.
D. It is a pleasure to see an alligator basking as long as 90 feet in the sunshine on a river bank.
E. All are wrong.

47. A. The report was useless to them because there was no needed information.
B. Since the report lacked needed information, it would have not been useful to them.
C. Since the report did not contain the needed information, it was not real usefill to them.
D. Being that the report lacked the needed information, they could not use it.
E. Since the report lacked needed information, it was of no use to them.

48. A. I have read such a lot about him that I am looking forward to seeing him veiy much.
B. I am reading such a lot about him that I will be looking forward to seeing him veiy much.
C. Having read such a lot about him that I am looking forward to seeing veiy much.
D. I had read such a lot about him that I am looking foiward to seeing him very much.
E. With regard to reading such a lot about him that I am looking foiward to seeing him veiy much.

Questions 49 to 52 are based on the following comprehension:


Instruction: Read the following passage and answer the accompanying questions, base your answer on what
is stated or implied in the passage. Mark your answers on the Answer Sheet.
Diamonds, an occasional component of rare igneous called lamproites and kimberlites, have never been
dated satisfactorily. However, some diamonds contain minute inclusions of silicate minerals, commonly
olivine, pyroxene, and garnet. These minerals can be dated by radioactive decay techniques because of
the veiy small quantities of radioactive trace elements they, in turn, contain. Usually, it is possible to
conclude that the inclusions are older than their diamond hosts, but with little indication of the time
interval involved. Sometimes, however, the crystal form of the silicate inclusions is observed to
resemble more closely the internal structure of diamond than that of other silicate minerals. It is not
known how rare this resemblance is, or whether it is most often seen in inclusions of silicates such as
garnet, whose crystallography is generally somewhat similar to that of diamond; but when present, the
resemblance is regarded as compelling evidence that the diamonds and inclusions are truly co-genetic.
Page: 7
49. The author implies that silicate inclusions were most often formed
A. with small diamonds inside of them
B. with trace elements derived from their host minerals
C. by the radioactive decay of rare igneous rocks
D. at an earlier period than were their host minerals
E. from the crystallization of rare igneous material

50. According to the passage, the age of silicate minerals included in diamonds can be determined due to a
feature of the
A. trace elements in the diamond hosts
B. trace elements in the rock surrounding the diamonds
C. trace elements in the silicate minerals
D. silicate minerals’ crystal structure
E. host diamonds’ crystal structure

51 . Tire author states that which of the following generally has a crystal structure similar to that of diamond?
A. Lamproite B. Kimberlite C. Olivine D. Pyroxene E. Garnet

52. Tire main purpose of the passage is to


A. explain why it has not been possible to determine the age of diamonds
B. explain how it might be possible to date some diamonds
C. compare two alternative approaches to determining the age of diamonds
D. compare a method of dating diamonds with a method used to date certain silicate minerals
E. compare the age of diamonds with that of certain silicate minerals contained within them

Questions 53 to 57 are based on the following instruction:


Instruction: Each question below consists of five options. You have to find out the incorrect usage of
idioms from the first four options. If all the four options are correct, choose option E.
53. A. It took me three hours to wade through this extremely long report.
B. He was on the carpet last week for being late for work three times.
C. I like that new procedure and I have decided to jump on the bandwagon.
D. Tire habitual cheat finally saw the light.
E. All of these are correct.

54. A. We saw the boss at the bar but we gave him a wide berth.
B. It may appear that they're getting ahead by cheating, but they'll get their just deserts in the end.
C. Everyone should try to avoid poetic justice.
D. At last, the Secretary took the chair.
E. All of these are correct.

55. A. Tire spokesman took an early bath after a series of embarrassing and incorrect statements.
B. All the football lovers around the globe want the world cup final to be a nail biter.
C. His questions covered' a lot of territory.
D. After his failure, the chairman got his day in the sun.
E. All of these are correct.
Page: 8
56. A. His ship sank but he escaped with the skin of his teeth.
B. Income tax is a real cash .cow for the government.
C. Tire bombing was a wake-up call to strengthen domestic security.
D. An eager beaver is hated by all the organisms of its class.
E. All of these are correct.

57. A. It is our labor of love and a huge reward is anticipated hence.


B. Tire police urged zero tolerance for the eve teasers.
C. Tire New Year party is going in frill swing.
D. I need your help but you just turned your back on me.
E. All of these are correct.

Questions 58 & 59 are based on the following instruction:


Instruction: Identify the sentence(s) that is/are showing the conect use of the underlined word.

58. i. The auctioneer was describing the antique in his hand to induce the audience to bid for it.
ii. Some of the things he inherited can actually be called antiques!
iii. It was absurd how they antiquated the chosen items to increase their value.
A. both i and ii B. both i and iii C. both ii and iii D. i only E. all three Sentences

59. i. There is a manifest difference between the students of this institution and those studying elsewhere.
h. She manifested truthfully that she lacked any interest in her studies
iii. After the accident, the police asked for the passenger manifest of the ship.
A. i only B. ii only C. both i and iii D. both i and ii E. all three sentences

Questions 60 & 61 are based on the following instruction:


Instruction: Choose the word that is most nearly SIMILAR in meaning to the given word.

60. As the job fair neared to an end, the recent college gr aduate became ever more ingratiating, desperately
hying to befriend prospective employers he had earlier not even deigned to give so much as a cursory
glance.
A. fawning B. withdrawn C. volatile D. vociferous E. direct

61. Possessed of an insatiable sweet tooth, Jim enjoyed all kinds of candy, but he had a special affinity for
gumdrops, his absolute favorite.
A. container B. odium C. nature D. disregard E. predilection

Questions 62 & 63 are based on the following instruction:


Instruction: Choose the right prepositions.
62. The struggle independence and the subsequent partitions 1947 and 1971 were momentous events
which came tenibly high costs.
A. of, in, into B. for, at, by C. for, in, at D. through, for, by E. None of these
Page: 9
63. There were genuine concerns a section of Burmese nationalists that their country's separation
India was a ploy the part of the colonial administration to halt the advance of representative
government in Burma.
A. between, of, into B. among, to, by C. among, for, onto
D. among, from, on E. None of these

Questions 64 & 65 are based on the following instruction:


Instruction: Tire underlined pail of the given sentence may contain error. Select tire choice that best
replaces the underlined part keeping the original meaning intact. If you think that the original
sentence is the best choice then select A.
64. Tire controversial themes, which resonate with recent political events, explain why the book is selling at
such a feverish pace.
A. explain why the book is selling at such a feverish pace
B. explains the feverish pace of the book
C. explain the reason for the pace of the book's feverish sales
D. explains why the book’s selling pace is so feverish
E. is why the book is selling well

65. One of the best features of the journalist’s lifestyle is von never know what's next.
A. you never know what’s next
B. it’s so unpredictable
C. that you never know what’s next
D. one can never predict what’s next
E. its unpredictability

Questions 66 to 70 are based on the following instruction:


Instruction: Underlined parts of the following sentences may contain error(s) in grammar, diction, usage or
idiom. Choose the underlined portion that contains such error(s). If there is no error then select E.
66. Reading (A) people is perhaps (B) the most important intuitive skill (C) leaders can possess (D).
No error (E).

67. For that reason, they are continually (A) aware on (B) what (C) they have at their (D) disposal.
No error (E).

68. Everything that (A) happens around us does (B) so (C) in the context (D) of a bigger picture.
No error (E).

69. An internet shutdown may not seem (A) like an act of violence, but when (B) bullets are fired
protesters (C) and nobody is (D) able to document it, brutality is able to flourish. No error (E).

70. Tire collective (A) firry pouring out on to the streets is a (B) result of decades of (C) oppression against
(D) women in Iran. No error (E).
Page: 10
Part II: Writing Ability [30marks]
1. Some say that central bank should adopt crypto currency now as it will be the future while some other
differ the argument on the point that it will crash the existing economy. What is your standpoint? Justify.
10
2. Construct writing with the following words and give a suitable title. 10
Port Cooperation Project Stability Destruction
Estimate Indication Resources Knowledge Concrete

3. Describe the challenges of an immigrant family in the developed countries. 10

Page: 11
“In the name of .ACOih, the most beneficent, the most merciful”
CAPSTONE
Contact: 01972 277 866, 016 3031 3031 Web: www.capstonebd.com facebook.com/capstonebd

Mock Test for IBA (DU) MBA Admission

INSTRUCTION: Please read carefully! Test Booklet No: 0004


1. The test consists of two parts: I. Multiple Choice Questions (MCQ) and IL Written Ability.
This part to be filled in
2. Total duration of the test will be 120 minutes (90 for Part-I &
30 minutes for Part II). The answer Sheet (For MCQs) will be by the Candidate
taken away after 90 minutes.
Admit No
3. On this Answer Sheet, write your Admit Number and Test Name
Booklet Number in the grids and your Name and Signature in
the boxes provided using a pen. Darken the corresponding
number boxes below the grids using a pen.

4. On this Test Booklet write your Admit Number and Name in the spaces provided using a pen.

5. Give all your answers in the Answer Sheet by completely darkening the lettered box, which represents
the correct answer to that the letter cannot be seen USE Pen ONLY.
® @
6. Do rough work anywhere on the rest booklet,
This part to be filled in by the invigilator
except in the spaces provided for written answers.
Do not use the answer sheet or extra paper for Please verify that
rough work. Candidate’s Name & Admit Number matches the
information on the admit card.
7. For each MCQ, 25% of assigned marks will Candidate’s appearances resembles the photograph
be deducted for each wrong answer. Each on the Admit Card.
unanswered question will earn zero marks.
Admit No
8. Candidates are required to obtain a minimum Signature of the Invigilator
acceptable mark in each section and in each
part.

9. Switch off your mobile phones.

10. Your script will not be examined if you adopt any unfair means.

11. Do not use a calculator or any other electronic devices.

12. Remain seated silently at the end until you are asked to go.

|STOP! DO NOT TURN THIS PAGE UNTIL YOU ARE ASKED TO DO SO.|
For Office Use Only
1 2 | 3
Part I: Multiple Choice Questions [70 marks]
Section 1: Language & Communication 25x1=25 marks
Questions 1 to 3 are based on the following instruction:
Instruction: Read the following two passages and answer the accompanying questions, base your answer on
what is stated or implied in the passage. Mark your answers on the Answer Sheet.
Passage A
hr January 1995 a vast section of ice broke off the Larsen ice shelf in Antarctica. While this occurrence,
the direct result of a regional wanning trend that began in the 1940s, may be the most spectacular
manifestation yet of serious climate changes occurring on the planet as a consequence of atmospheric
heating, other symptoms—more intense storms, prolonged droughts, extended heat waves, and record
flooding—have been emerging around the world for several years.
According to scientific estimates, furthermore, sea-level rise resulting from global wanning will reach 3
feet (1 meter) within the next century. Such a rise could submerge vast coastal areas, with potentially
irreversible consequences.
Late in 1995 the Intergovernmental Panel on Climate Change (IPCC) reported that it had detected the
“fingerprint” of human activity as a contributor to the wanning of the earth's atmosphere. Furthennore,
panel scientists attributed such wanning directly to the increasing quantities of carbon dioxide released by
our binning of fossil fuels. The IPCC report thus clearly identifies a pattern of climatic response to human
activities in the climatological record, thereby establishing without doubt that global warming can no longer
be attributed solely to natural climate variability.
Passage B
Over the past two decades, an extreme view of global wanning has developed. While it contains some
facts, this view also contains exaggerations and misstatements, and has sometimes resulted in unreasonable
environmental policies.
According to this view, global warming will cause the polar ice to melt, raising global sea levels,
flooding entire regions, destroying crops, and displacing millions of people. However, there is still a great
deal of uncertainty regarding a potential rise in sea levels. Certainly, if the earth warms, sea levels will rise
as the water heats up and expands. If the polar ice caps melt, more water will be added to the oceans, raising
sea levels even further. There is some evidence that melting has occurred; however, there is also evidence
that the Antarctic ice sheets are growing, hr fact, it is possible that a warmer sea surface temperature will
cause more water to evaporate, and when wind carries the moisture-laden air over the land, it will precipitate
out as snow, causing the ice sheets to grow. Certainly, we need to have better knowledge about the
hydrological cycle before predicting dire consequences as a result of recent increases in global temperatures.
Uris view also exaggerates the impact that human activity has on the planet. While human activity may
be a factor in global wanning, natural events appear to be far more important. Ure 1991 eruption of Mount
Pinatubo in the Philippines, for example, caused a decrease in the average global temperature, while El
Nino, a periodic perturbation in the ocean’s temperature and circulation, causes extreme global climatic
events, including droughts and major flooding. Of even greater importance to the earth's climate are
variations in the sun's radiation and in the earth’s orbit. Climate variability has always existed and will
continue to do so, regardless of human intervention.
Paae: 1
1 . Which one of the following questions is central to both passages?
A. How has an increase in the burning of fossil fuels raised the earth's temperature?
B. To what extent can global wanning be attributed to human activity?
C. What steps should be taken to reduce the rate of global wanning?
D. What kinds of human activities increase the amount of carbon dioxide in the atmosphere?
E. To what extent is global warming caused by variations in the sun's radiation and the earth's orbit?

2. Which one of the following is mentioned in passage B but not in passage A as a possible consequence of
global warming?
A. an increase in the size of the Antarctic ice sheet
B. a decrease in the amount of snowfall
C. a falling of ocean sea levels
D. an increase in the severity of heat waves
E. an increase in the frequency of major flooding
3. The author of passage B would be most likely to make which one of the following criticisms about the
predictions cited in passage A concerning a rise in sea level?
A. These predictions incorrectly posit a causal relationship between the wanning of the earth and rising
sea levels.
B. These predictions are supported only by inconclusive evidence that some melting of the polar ice
caps has occurred.
C. These predictions exaggerate the degree to which global temperatures have increased in recent
decades.
D. These predictions rely on an inadequate understanding of the hydrological cycle.
E. These predictions assume a continuing increase in global temperahues that may not occur.

Questions 4 to 8 are based on the following instruction:


Instruction: hi questions given below out of five alternatives, choose the one which expresses the same
meaning as the underlined phrase/clause.
4. After visiting her grandparents, Bashundhara city, and watching the movies at Cineplex, Jasmine was
bending over backwards to entertain her nephews.
A. Jasmine was doing very little to entertain her nephews.
B. Jasmine was making small efforts to entertain her nephews.
C. Jasmine was hying very hard to entertain her nephews.
D. Jasmine was not hying at all to entertain her nephews.
E. None of the above
5. Sarwar would have been playing cricket rurtil the cows came home if it hadn't been for Natasha
dragging him home for dinner.
A. Sarwar was just about to stop playing cricket.
B. Sarwar didn't even want to play cr icket to begin with.
C. Sarwar prefers nature to athletics.
D. Sarwar would have continued playing cricket for a long time.
E. None of the above

6. Shourov wanted to fix his cousin's computer, but he was already having problems setting up his Aunt's
Wi-Fi network and he didn't want to open a whole new can of worms.
A. Shourov was sick of spending his time helping his family.
B. Shourov wanted to go fishing instead of working on computers.
C. Shourov was having difficulties untangling the computer wires.
D. Shourov was not ready to begin working on a complicated new problem.
E. None of the above
Paae: 2
7. After losing the match against Tottenham, Zidane took Ronaldo to task.
A. rebuked Ronaldo B. dismissed Ronaldo C. promoted Ronaldo
D. praised Ronaldo E. None of the above

8. Tasnia expected Shahed to sob uncontrollably when she broke up with him; however, Shahed kept a
stiff upper lip.
A. Shahed cried even more than Tasnia had expected.
B. Shahed cried about as much as Tasnia had expected.
C. Shahed cried a little less than Tasnia had expected.
D. Shahed did not cry E. None of the above

Questions 9 to 13 are based on the following comprehension:


Instruction: Read the following passage and answer the accompanying questions, base your answer on
what is stated or implied in the passage. Mark your answers on the Answer Sheet.
Ocean water plays an indispensable role in supporting life. Tire great ocean basins hold about 300
million cubic miles of water. From this vast amount, about 80,000 cubic miles of water are sucked into the
atmosphere each year by evaporation and returned by precipitation and drainage to the ocean. More than
24,000 cubic miles of rain descend annually upon the continents. This vast amount is required to replenish
the lakes and streams, spring and water tables on which all flora and fauna are dependent. Thus, the
hydrosphere permits organic existence.
The hydrosphere has strange characteristic because water has properties unlike those of any other liquid.
One anomaly is that water upon freezing expands by about 9 percent, whereas most liquids contract on
cooling. For this reason, ice floats on water bodies instead of sinking to tire bottom. If the ice sank, the
hydrosphere would soon be frozen solidly, except for a thin layer of surface melt water during the summer
season. Tims, all aquatic life would be destroyed and the interchange of warm and cold currents, which
moderates climate, would be notably absent.
Another outstanding characteristic of water is that water has a heat capacity which is the highest of all
liquids and solids except ammonia. This characteristic enables the oceans to absorb and store vast quantities
of heat, thereby often preventing climatic extremes, hi addition, water dissolves more substances than any
other liquid. It is this characteristic which helps make oceans a great storehouse for minerals which have
been washed down from the continents, hi several areas of the world these minerals are being commercially
exploited. Solar evaporation of salt is widely practiced, potash is extracted from the Dead Sea, and
magnesium is produced from sea water along the American Gulf Coast.

9. The author’s main purpose in this passage is to-


A. describe the properties and use of water B. illustrate the importance of conserving water
C. explain how water is used in commerce and industry D. reveal the extent of the earth’s ocean masses
E. compare water with other liquids.

10. According to the passage, fish can survive in the oceans because
A. they do not need oxygen B. ice floats
C. evaporation and condensation create a water cycle D. there are currents in the oceans
E. water absorbs heat
11 . Which of the following characteristics of water does the author mention in the passage?
I. Water expands when it is frozen II. Water is a good solvent III. Water can absorb heat
A. I only B. II only C. I and II only D. II and III only E. I, II and III

12. According to the passage, the hydrosphere is NOT


A. responsible for all forms of life B. able to modify weather
C. a source of a natural resources D. in danger of freezing over
E. the part of the earth covered by water
Page: 3
13. The author’s tone in the passage can best be described as
A. dogmatic B. dispassionate C. speculative D. biased E. hortatory

Questions 14 to 18 are based on the following instruction:


Instruction: Tire underlined part of the given sentence may contain error. Select tire choice that best
replaces the underlined part keeping the original meaning intact. If you think that the original
sentence is the best choice then select A.
14. Tire representatives shook hand with each other in the meeting yesterday.
A. shook hand with B. shakes hand with C. shaked hand with
D. shook hands with E. shake hand with

15. A lot of people worry unnecessarily about the past. I have never have any regrets about my past.
A. have never have B. had never C. had not
D. have never E. have never had

16. The old brain is called the reptilian brain. It does not know passion, but only stolid obedience to its own
genetic dictates.
A. After the old brain is called the reptilian brain, it does not know passion, but only stolid obedience to
its own genetic dictates.
B. Tire old brain, called the reptilian brain, does not know passion, but only stolid obedience to its own
genetic dictates.
C. Tire old brain is called the reptilian brain, whereupon it does not know passion, but only stolid
obedience to its own genetic dictates.
D. Unless the old brain, called the reptilian brain, does not know passion, only stolid obedrence to its
own genetic dictates.
E. Correct as is

17. Because of the need for accuracy, all employees must diligently review then work at the end of every day.
A. all employees must diligently review their work at the end of every day
B. all employees who work here must be diligent and careful to review their work at the end of every day
C. employees must be diligently reviewing and checking their work at the end of every day
D. workers and employees must diligently review their work at the end of every day
E. all employees must diligently review and assess their work daily, every day

1 8. When making a chocolate torte, only the best ingredients should be used.
A. only the best ingr edients should be used. B. one should use only the best ingredients.
C. the best ingr edients only should be used. D. one should have used only the best ingredients.
E. using only the best ingredients is essential.

Questions 19 & 20 are based on the following instruction:


Instruction: Choose the appropriate preposition
Amjad Yousef, a major in one of Syria’s most feared intelligence units, is operating from the Kafr Sousa
base, where he has been (19) most of the past six months since the Guardian revealed his role
(20) shooting dead dozens of people across a death pit in Tadamon, a suburb of the Syrian capital
in 2013.
19. A. in B. for C. of D.by E. to
20. A. in B. for C. of D.by E. to
Page: 4
Questions 21 to 23 are based on the following instruction:
Instruction: Underlined parts of the following sentences may contain error(s) in grammar, diction, usage or
idiom. Choose the underlined portion that contains such error(s). If there is no error then select E.
21. Many nurses said they were not told (A) about the training repayment requirement before
beginning (B) work, and that classroom instruction often repeated (C) what they learned in
(D) school. No error (E).

22. Tire more we can define and gain confidence in (A) our sense of self, the more (B) our identity
becomes (C) our foundation for growth and innovation in whom (D) we are. No error (E).

23. It is well-known that Serb snipers in the hills surround (A) the city would arbitrarily shoot (B)
residents on the streets below, and that (C) select Serb allies were invited to fire (D) some shots of
their own. No error (E).

Questions 24 & 25 are based on the following instruction:


Instruction: Identify the sentence(s) that is/are showing the correct use of the underlined word.
24. I. Any opposition to the rules is intolerable.
II. His boss was intolerant of his tardiness.
III. Unwilling to accept views of the others, Mike's intolerances have crossed its limit.
A. II only B. I & III only C. I & II Only D. All are conect E. None of these

25. 1. Vitalness is the key for the success of this program.


II. His intense vitality was easily observable.
III. Tazul approached for the solution with inevitable vitality.
A. I & II only B. II only C. Ill only D. All are conect E. None of these

Section 2: Mathematics 30x1=30 marks


Instructions: Solve each of the following problems and mark the conect answer on your Answer Sheet.
DO NOT USE A CALCULATOR. Figures are not drawn to scale.
26. A ship develops a leak 12 km from the shore. Despite the leak, the ship is able to move towards the
shore at a speed of 8 km/hr. However, the ship can stay afloat only for 20 minutes. If a rescue vessel
were to leave from the shore towards the ship and it takes 4 minutes to evacuate the crew and passengers
of the ship, what should be the minimum speed of the rescue vessel in order to be able to successfully
rescue the people aboard the ship?
A. 53 km hr B. 37 km/hr C. 28 km/hr D. 44 km/hr E. None of these

27. Kaiser writes all the numbers from 100 to 999. The number of zeroes that he uses is 'a', the number of 5's
that he uses is 'b' and the number of 8's he uses is 'c'. What is the value of b + c - a ?
A. 280 B. 380 C. 180 D. 80 E. Cannot be detennined

28. hi the figure, ABCD is a rectangle and points E, F, G and H are midpoints E
A B
of its sides. What is the ratio of the area of the shaded region to the area of
the un-shaded region in the rectangle? H F
A. 1:1 B. 1:2 C. 2:1 D G C
D. 1:3 E. 3:1
Page: 5
29. In a certain class consisting of 36 students, some boys and some girls, exactly —3 of the boys and exactly —4 of
the girls walk to school. What is the greatest possible number of students in this class who walk to school?
A. 9 B. 10 C. 11 D. 12 E. None of these

30. Tire price of a consumer good increased by p% during 2012 and decreased by 12% during 2013. If no
other change took place in the price of the good and the price of the good at the end of 2013 was 10%
higher than the price at the beginning of 2012, what was the value of p?
A. -2% B. 2% C. 22% D. 25% E. None of these

31. If x, y, z > 0 and x > y > z which of the following could be true?
I. 7x > 7y > 7z II. 7z > y y > 7x III. 7x > Vz > Jy
A. Only I B. Only II C. I and II D. I and III E. None of these

32. At Supersonic Corporation, the time required for a machine to complete a job is determined by the
formula: t = 7w + 7(w-l) where w is the weight of the machine in pounds and t is the horns required
to complete the job. If machine A weighs 8 pounds, and machine B weighs 7 pounds, how many hours
will it take the two machines to finish one job if they work together?

A B. 0.5(78 + 76) C. 0.33(6 - 73) D. 3(73+72) E. None of these


7-73
33. If a right-angled isosceles triangle is inscribed in a circle, what is the ratio of the area of the circle to the
area of the triangle?
A.-
4
B-
2
C -2 D. 2tt E. ti

34. Sakib the barber shaved 40% of his customers and gave a haircut to 80% of his customers. He charged
Tk. 7 for a shave and Tk. 5 for a haircut. If 20% of customers who opted for a shave also had a hair-cut,
what were Sakib’s earnings if he had 75 customers (in Tk.)?
A. 410 B. 1020 C. 510 D. 980 E. None of these

35. If each day a tree loses half its remaining leaves, what fraction of its leaves will it have lost after 5 days?
A.—
32
B.—
16
C. 10 — D. 16— E. 32—
36. A question paper consists of two sections having respectively 3 and 5 questions. The following note is
given on the paper. “It is not necessary to attempt all the questions”. One question from each section is
compulsory, hi how many ways, a candidate can select the questions?
A. 38 B.320 C. 256 D. 217 E. None of these
2
37. At a school, the teacher-student ratio is 1:9. If -rd of the students is female and one-quarter of the
teachers are female, what fraction of the total students and teachers are female?
A.—
24
B-8 C. 56 — D. 12— E. None of these

38. The average weight of three men A, B and C is 84 kg. Another man D joins the group and the new
average becomes 80 kg if another man E, whose weight is 3kg more than that of D, replace A, then the
average weight of B, C, D and E becomes 79 kg. The weight of A is-
A. 70kg B. 72kg C. 75kg D. 80kg E. None of these
Paae: 6
39. Kishore’s wife’s age is 24 and his Son’s age is 12 as on today. On same day Kishore invented a Time
Machine through which he travelled to ancient time. He stayed in ancient time for 6 Years then he
returned back. But by Surprise, there is no change in Kishore’s age after his return. If the difference
between the average age of the family before and after is only 4 then Kishore’s age is?
A. 33 B. 39 C. 40 D. 41 E. Cannot be determined

40. 100 grams Rosewater is 99% water by weight. After some of the water evaporates, the rosewater is now
98% water by weight. What is the new weight of the rosewater, in grams?
A. 2 B.50 C. 92 D. 96 E. None of these

41. How many positive even integers are factors of 723?


A. 9 B. 10 C. 31 D. 63 E. None of these

42. A can do a piece of work in 21 days. B is 50% more efficient than A. C is twice efficient than B. A
started the work alone and worked for some days and left the work. Then B and C joined together and
completed the work in 2 days. Then how many days did A work alone?
A. 7 days B. 12 days C. 14 days D. 21 days E. None of these

43. Two alloys A and B are composed of two basic elements. The ratios of the compositions of the two basic
elements in the two alloys are 5:3 and 1:2, respectively. A new alloy X is formed by mixing the two
alloys A and B in the ratio 4:3. What is the ratio of the composition of the two basic elements in alloy X?
A. 1:1 B.2:3 C. 5: 2 D. 4: 3 E. 7: 9

44. One hundred business people gather at a meeting. 85 of them cany a cellular phone, 80 of them have a
beeper, 75 of them speak at least two languages and 70 of them wear a suit. Therefore, a certain number
of them have all of the above: a cell phone AND a beeper AND speak at least two languages AND wear
a suit. Out of these 100 businesspeople, what is die least possible number who has all of die above?
A. 10 B.20 C. 25 D. 30 E. None of these

45. hi figure, AB = AC, D is a point on AC and E on AB such that AD = ED = EC = BC.


Then zA:zB-
A. 1:2 B. 2:1 C. 3:1
D. 1:3 E. None of these

46. There are five hobby clubs in a college viz. photography, yachting, chess, electronics and gardening. The
gardening group meets every second day, the electronics group meets every third day, the chess group
meets every fourth day, the yachting group meets every fifth day and the photography group meets every
sixth day. How many times all the five groups do meets on the same day within 180 days?
A. 3 B. 5 C. 10 D. 18 E. None of these

47. If n = 1 + x where x is a product of four consecutive positive integers, then which of the following is true?
i. n is odd ii. n is prime iii. n is a perfect square
A. i and iii only B. i and ii only C. i only D. iii only E. ii and iii only


48. x and y are positive integers. If — + < 2 , which of the following must be true?
x y
A. x + y > 4 B. xy > 1 D. (x - y)2 > 0 E. None of these
y x

Paae: 7
49. A store was selling beans at a profit of 20%, bnt the weighing machine was broken and showed 950
giains for 1 kilogr am. What is the store's actual profit percentage?
A. 10% B. 14% C. 16% D. 19% E. None of these

50. Omor writes the numbers 1,2, 3, , 99, 100. He marks out 1, skips the next number 2, marks out 3,
and continues skipping and marking out the next number to the end of the list. Then he goes back to the
start of his list, marks out the first remaining number 2, skips the next number 4, marks out 6, skips 8,
marks out 10, and so on to the end. Omor continues in this manner until only one number remains. What
is that number?
A. 13 B.32 C. 56 D. 64 E. None of these

51. Reza has five tickets of a lucky draw for which a total of 12 tickets were sold and exactly six prizes are
to be given. The probability that the per son will win at least one prize is?
A.
A

61
32
B.
TA
—31
132
C.
O

131
132
D.
TA

11
12
E.
T^ AT xl
None off these

52. Foyez and Alvie cross the lake in a straight line with the help of a one-seat boat. Each can row the boat
at a speed of 7 km hour and swim at a speed of 2 km/hour. They start from the same point, where Foyez
starts by rowing while Alvie swims. After some time Foyez got out of his boat and started swimming by
leaving the boat in the position he started swimming. Once Alvie reaches the position of the boat, he will
begin rowing. If they arrived on the other side of the lake at the same time which is 90 minutes after they
started, the length of time the boat was empty would be?
A. 35 minutes B. 40 minutes C. 45 minutes D. 50 minutes E. None of these

Questions 53 to 55 are based on the following information:


A. Statement (i) alone is sufficient but statement (ii) alone not sufficient to answer the question asked.
B. Statement (ii) alone is sufficient but statement (i) alone not sufficient to answer the question asked.
C. Both statement (i), (ii) together are sufficient to answer the question but neither statement is sufficient alone.
D. Each statement alone is sufficient to answer the question.
E. Statement (i) & (ii) are not sufficient to answer the question asked and additional data is needed to
answer the statements.

53. If k is an integer, what is the value of k?


i. (k-l)(k-3)<0 ii. (k- 5)(k - l)(k- 3)(k- 6) < 0

54. If the cost of House L increased by 20% over the same time that the cost of House M decreased by 20%,
then the ratio of the new price of House M to the new price of House L is what percent of the original
cost of House M ?
i. If House L had increased by 30%, then the increased price of House L would have been equal to the
original price of House M.
ii. The price of House L is $300,000.

55. For a sale, the original retail price of a particular shirt and the original retail price of a particular hat were
both reduced by 20 percent. The sale price of the hat is how many dollars more than the sale price of the
shirt?
i. The original retail price of the hat was 10 dollars more than the original retail price of the shirt.
ii. The original retail price of the hat was 50 percent greater than the original retail price of the shirt.
Paae: 8
Section 3: Analytical Ability 15x1=15 marks
Instruction: Solve each of the following problems and mark the correct answer on your Answer Sheet.
56. Advocates insist that health savings accounts are an efficient method to reduce medical expenses.
However, widespread adoption of these accounts will soon undermine the public’s health. One reason
for this is that most people will be reluctant to deplete their accounts to pay for regular preventive
examinations, so that in many cases a serious illness will go undetected until it is far advanced. Another
reason is that poor people, who will not be able to afford health savings accounts, will no longer receive
vaccinations against infectious diseases.
The statements above, if true, most support which of the following?
A. Wealthy individuals will not be affected negatively by health savings accounts.
B. Private health instuance will no longer be available.
C. Most diseases are detected din ing regular preventive examinations.
D. Some people without health savings accounts are likely to contract infectious diseases.
E. The causal relationship between an individual's health and that person’s medical care has been
adequately documented.

57. Newspaper editors should not allow reporters to write the headlines for their own stories. The reason for
this is that, while the headlines that reporters themselves write are often clever, what typically makes
them clever is that they allude to little-known information that is familiar to the reporter but that never
appears explicitly in the story itself.
Which of the following, if true, most strengthens the argument?
A. Tire reporter who writes a story is usually better placed than the reporters' editor is to judge what the
story's most newsworthy features are.
B. To write a headline that is clever, a person must have sufficient understanding of tire story that the
headline accompanies.
C. Most reporters rarely bother to find out how other reporters have written stories and headlines about
the same events that they themselves have covered.
D. For virtually any story that a reporter writes, there are at least a few people who know more about the
story's subject matter than does the reporter.
E. The kind of headlines that newspaper editors want are those that anyone who has read a reporter's
story in its entirety will recognize as clever.

58. Some theorists and critics insist that no aesthetic evaluation of a work of art is sound if it is based even
in part on data about the cultural background of the artist. This opinion is clearly false. The only sound
aesthetic evaluations of artists' works are those that take into account factors such as the era and the
place of the artists' births, their upbringing and education, and the values of their societies- in sum, those
factors that are part of their cultural background.
The above argument is most vulnerable to which of the following objections?
A. Tire argument presupposes the conclusion for which it purports to provide evidence.
B. Tire argument cites evidence that undermines rather than supports the conclusion.
C. Tire argument draws its conclusion by means of an equivocal interpretation of key terms.
D. The argument assumes that the production of an effect is evidence of an intention to produce that effect.
E. The argument assumes that evaluative disputes can be resolved by citing factual evidence.
Paae: 9
Questions 59 to 63 are based on the following information:
A group of eight people is going camping in Van 1, Van 2, and Van 3. The group consists of two
women- V and S- and six girls- F, G, H, I, J, and T. Van assignments will be made as follows:
There will be no more than three persons in a Van.
V will be in Van I.
V will not be in a Van with her daughter I. and S will not be in a Van with her daughter J.
F, G, and H, who are close friends, will be in a Van together

59. Which of the following can be in Van 1?


A.F B. G C. H D. I E.S

60. If F is in Van 2, which of the following must be true?


A. His in Van 3 B. I is in Van 3 C. J is in Van 2 D. T is in Van I E.S is in Van 3

61. If the two women are together in a Van, which of the following is a pair of girls who must be together in a
Van?
A. F and J B. G and T C. H and I D. I and J E. J and T

62. If S is in Van 2, which of the following must be in the same Van as V?


A.F B. G C. I D. J E. T

63. If G is in Van 3 and the two women are not in the same Van as each other, the people in Van 2 can be-
A. Hand T only B. S and T only C. F, H, and I D. I, J, andS E. I, T, and S

Questions 64 to 67 are based on the following information:


Each of six automated jobs, numbered 1 through 6, takes one full month to complete. No time elapses
between the completion of any of the six jobs and the beginning of another job. The group of six job,
must be completed in the shortest possible time period, subject only to the following restrictions:
Jobs 1 and 2 must both be completed before any of the other jobs can be begun.
Job 3 must be completed before job 4 can be begun.
At any one time, no more than one job can be performed, except that jobs 4 and 5 can be performed
concurrently.
64. Which of the following jobs could be the second job performed?
A. 2 B.3 C. 4 D. 5 E. 6

65. The shortest possible time period in which the group of six jobs can be completed is
A. Two months B. Three months C. Four months D. Five months E. Six months

66. Which of the following CANNOT be true of any acceptable ordering of jobs?
A. Job 1 is performed before job 2. B. Job 3 is performed before job 6.
C. Job 4 is performed before job 6. D. Job 5 is performed before job 3.
E. Job 6 is performed before job 3.

67. If job 6 is performed as early in the order of jobs as is permissible, then job 6 is performed
A. first B. second C. third D. fourth E. fifth
Page: 10
Questions 68 to 70 are based on the following information:
The following pie-chart shows the percentage distribution of the expenditure incurred in publishing a
book. Study the pie-chart and the answer the questions based on it.

Various Expenditures (in percentage) Incurred in Publishing a Book

Promotion Cost, Printing Cost, 20%]


10%

Transportation
| Royalty Cost, 15% f Cost, 10%

| Binding Cost, 20% Paper Cost, 25% |

68. If for a certain quantity of books, the publisher has to pay Tk. 30,600 as printing cost, and then what will
be amount of royalty to be paid for these books?
A. Tk. 19,450 B.Tk. 21,200 C. Tk. 22,950 D. Tk. 26,150 E. None of these

69. What is the central angle of the sector corresponding to the expenditure incurred on Royalty?
A. 15° B. 24° C. 54° D. 48° E. None of these

70. The price of the book is marked 20% above the C.P. If the marked price of the book is Tk. 180, then
what is the cost of the paper used in a single copy of the book?
A. Tk. 36 B.Tk. 37.5 C. Tk. 42 D. Tk. 44.25 E. None of these

Part II: Writing Ability [30 marks]


1 . Prepare a thematic write-up using the following words/phrases. You may use one word/phrase more than
once. You may also change the form of the given word/phrases as and when used. Give an appropriate
title of your write-up. To maintain coherence, you may write some sentence without using the
words/phrases given in the list. 10
Corruption Depravity Selfish interest Afflict Inhibit
Adoption Compel Penalties Vicious Professionalism

2. Explain "Hate is not the answer". 10

3. Write about the last historical place you have visited. 10

Page: 11
“In the name of ACCah, the most Beneficent, the most merciful”
CAPSTONE
Contact: 01972 277 866, 016 3031 3031 Web: www.capstonebd.com facebook.com/capstonebd

Mock Test for IBA (DU) MBA Admission

INSTRUCTION: Please read carefully! Test Booklet No: 0005


1. The test consists of two parts: I. Multiple Choice Questions (MCQ) and IL Written Ability.
This part to be filled in
2. Total duration of the test will be 120 minutes (90 for Part-I &
30 minutes for Part II). The answer Sheet (For MCQs) will be by the Candidate
taken away after 90 minutes.
Admit No
3. On this Answer Sheet, write your Admit Number and Test Name
Booklet Number in the grids and your Name and Signature in
the boxes provided using a pen. Darken the corresponding
number boxes below the grids using a pen.

4. On this Test Booklet write your Admit Number and Name in the spaces provided using a pen.

5. Give all your answers in the Answer Sheet by completely darkening the lettered box, which represents

•©
the correct answer to that the letter cannot be seen USE Pen ONLY.
@@ @
6. Do rough work anywhere on the rest booklet,
This part to be filled in by the invigilator
except in the spaces provided for written answers.
Please verify that
Do not use the answer sheet or extra paper for
rough work. Candidate’s Name & Admit Number matches the
information on the admit card.
7. For each MCQ, 25% of assigned marks will Candidate’s appearances resembles the photograph
be deducted for each wrong answer. Each on the Admit Card.
unanswered question will earn zero marks.
Admit No
8. Candidates are required to obtain a minimum Signature of the Invigilator
acceptable mark in each section and in each
part.

9. Switch off your mobile phones.

10. Your script will not be examined if you adopt any unfair means.

11. Do not use a calculator or any other electronic devices.

12. Remain seated silently at the end until you are asked to go.

|sTOP! DO NOT TURN THIS PAGE UNTIL YOU ARE ASKED TO DO SO.|
For Office Use Only

Page: 1
Part I: Multiple Choice Questions [70 marks]
Section 1: Analytical Ability 15x1=15 marks
Instruction: Solve each of the following problems and mark the correct answer on your Answer Sheet.
Questions 1 to 3 are based on the following information:
A. Statement (i) alone is sufficient but statement (ii) alone not sufficient to answer the question asked.
B. Statement (ii) alone is sufficient but statement (i) alone not sufficient to answer the question asked.
C. Both statement (i), (ii) together are sufficient to answer the question but neither statement is sufficient alone.
D. Each statement alone is sufficient to answer the question.
E. Statement (i) & (ii) are not sufficient to answer the question asked and additional data is needed to
answer the statements.
1 . If aa and bb are positive integers, what is the greatest common divisor of aa and bb?
i. a + 3b -61 —
ii. 5a b = 1

2. If two students are chosen at random with replacement from a certain class, what is the probability that
two male students or two female students are selected?
i. There are 49 male students in the class.
.. ...
ii. The probability of selecting one male and one female student is
.21

3. A candy wholesaler needs to quickly sell some candy bars that are nearing their expiration date, so he
reduced the price of the candy bars. By what percent did he reduce the price of the candy bars?
i. The price of a candy bar was reduced by 36 cents.
ii. If a candy retailer purchases a case of 144 candy bars from the wholesaler, she will save S51.84 as a
result of the price reduction.

4. Women who are married sleep more soundly than women who have never married or lost a partner,
according to research from an eight-year study. Furthermore, it is well established that sleeping less than
six hours a night makes you 12% more likely to die prematurely than someone who sleeps up to eight
horns. Therefore, getting married increases a woman's life expectancy.
Which of the following, if true, does NOT strengthen the argument above?
A. Populations with lower odds of premature death have higher life expectancies.
B. Getting married does not lead to oversleep.
C. Marriage may introduce lifespan-shortening factors into a woman's life, but these factors tend to be
collectively of less consequence than the benefit of sounder sleep.
D. Not sleeping soundly is the key reason women who cannot sleep at least eight horns a night fail to do so.
E. Tire psychology of women who many is, on average, more conducive to sound sleep than that of
women who have never married or lost a partner.

5. Demographic experts predict that the global human population will reach its peak sometime in the
middle of this century, after which it will begin to decline. Population growth is driven primarily by high
birth rates in developing countries. It is known that when women have access to education and economic
opportunities, they choose to have fewer children. Therefore, these experts propose expanding
educational and economic opportunities to women in developing countries to bring about an earlier and
smaller population peak.
Page: 2
Which of the following, if true, provides the strongest grounds to doubt that the experts' proposal,
if adopted, will achieve its aim?
A. The demographic experts proposing expanding opportunities for women in developing countries
made the same recommendations over twenty years ago.
B. Tire experts' prediction is based on realistic assessments of the educational and economic
opportunities that can be made available to women in developing countries before that time.
C. Many women in INDUSTRIALIZED countries will continue to have four or more children despite
access to a variety of educational and economic opportunities.
D. Tire demogr aphic experts fail to explain why an earlier and smaller population peak is preferable to a
later and larger peak.
E. Expanding opportunities to women in developing countries is generally considered a positive
outcome regardless of its impact on population levels.

Questions 6 to 10 are based on the following information:



A professional cricket coach has agreed to give lessons to seven talented cricket players Fahid, Gohar,
Hamid, Israr, Junaid, Kashif, and Liaqrrat- and for this purpose has set aside an hour each weekday,
Monday through Friday. Tire week's schedule of lessons must meet the following conditions:
Each player must be scheduled for exactly one of the five days.
Exactly two of the lessons must each be for two players, but those two joint lessons cannot be on
consecutive days.
Fahid's lesson must be on a day earlier in the week than Gohar's lesson.
Hamid must be scheduled to be given his lesson by himself.
Israr's lesson must be on the day immediately before the day of Junaid's lesson.

6. Which of the following is an acceptable weekly schedule of lessons?


Monday Tuesday Wednesday Thursday Friday
A Fahid Junaid, Kashif Israr, Liaquat Hamid Gohar
B Hamid Fahid, Gohor Israr Junaid, Kashif Liaquat
C Israr Junaid Fahid, Hamid Gohar Kashif, Liaquat
D Kashif, Liaquat Hamid Israr Fahid, Junaid Gohar
E Liaquat Junaid, Kashif Hamid Fahid, Israr Gohar

7. If Hamid's lesson is scheduled for Wednesday, Israr's lesson must be scheduled for either
A. Monday or Tuesday B. Monday or Thursday C. Monday or Friday
D. Tuesday or Thursday E. Tuesday or Friday

8. Tire latest day in the week on which Fahid and Israr could be given a joint lesson is
A. Monday B. Tuesday C. Wednesday D. Thursday E. Friday

9. If a joint lesson for Gohar and Junaid is scheduled earlier in the week than a joint lesson for Kashif and
Liaquat, for which day must Hamid's lesson be scheduled?
A. Monday B. Tuesday C. Wednesday D. Thursday E. Friday

10. If tire two joint lessons are scheduled for as early in the week as possible, and Israr's and Junaid's lessons
for as late in the week as possible, Liaquat could be given his lesson
A. with Gohar B. with Israr C. with Junaid D. with Kashif E. by himself
Page: 3
Questions 11 to 15 are based on the following information:
The members of the Exam Control Committee and the members of the Administrative committee are to
be selected from exactly six qualified candidates. Tire six candidates are L, M, N, O, P, and Q. The
following rules apply:
Each Committee must have exactly three members.
Tire two Committees must have at least one member in common.
L cannot be on a Committee with O.
If O is selected for a Committee, P must also be selected for that Committee.
11. If the members of the Exam Control Committee are selected first, which of the following could be those selected?
A. L, M, and O B.L,O, and Q C. M, N, and O D. M, O, andP E. N, O, and Q

12. If the two Committees have parallel terms of office, which of the following could be selected as the
members of the Exam Control Committee and as the members of the Administrative Committee,
respectively, for one such term of office?
A. L, M. and N; O, P, and Q B. L, N, and P; M, O, and Q C. L, O, and P; L, O. and Q
D. M, N, and P; M, N, and O E. N, O, and P; O, P, and Q

1 3. If the members of the Exam Control Committee are M, N, and Q, and if the Administrative Committee is
to have as many members in common with the Exam Control Committee as the rules allow, tire
Administrative C ornmittee must consist of.
A. L, M, andN B.M,N, and Q C. M, O, and Q D.N,P,andQ E. O, P, and Q

14. If L, M, and N make up the Exam Control Committee, and N, P, and Q make up the Administrative
Committee, which of these Committee members could yield his or her place on a Committee to O
without necessitating any other membership changes?
A.L B. M C. N D. P E. Q

15. If L and O are each selected for a Committee, and only Q is selected for both Committees, which of the
following must be true?
A. M is selected for the same Committee as N B. N is selected for the same Committee as P
C. N is selected for the same Committee as O D. L is selected for a different Committee than P
E. O is selected for a different Committee than P

Section 2: Language & Communication 30x1=30 marks


Questions 16 to 23 are based on the following instruction:
Instruction: Fill in the blanks with suitable w^ords.
16. Robin's game-winning goal in the closing minutes of the game his well-known ability to excel
under stressful conditions.
I. clarified II. solidified III. identified IV. epitomized V. exemplified VI. decried
A. I or III B. IVorV C. Only IV D. II or VI E. None of these

17. Dangerously high winds attempts to begin the space shuttle mission on schedule, delaying the
launch by nearly a wreek.
I. thwarted II. forfeited III. implemented IV. discharged V. redoubled VI. frustrated
A. Only I B. norm C. Ill or V Dior IV E. I or VI
Page: 4
18. Deliberately designed to be devoid of elaborate cawing or other , Kakoli furniture was known for
its .
A. customization ... uniqueness B. spareness ... starkness C. embellishment ... garnishes
D. ornamentation ... simplicity E. flamboyance ... flourishes

1 9. Tire scientific organization tire newspaper for prominently covering the predictions of a psychic
while to report on a major research conference.
A. Celebrated ... failing B. Promoted ... refusing C. Denounced ... neglecting
D. Spumed ... hastening E. Honored ... opting

20. Tire colors and patterns on butterflies' wings may seem merely , but they are actually the
survival of these insects, enabling them to attract mates and to hide from predators.
A. Artificial ... dependent on B. Unique ... unnecessary to C. Decorative ... instrumental in
D. Beautiful ... results of E. Unrelated ... precursors of

21 . Thought it is often exclusively Brazil, tire Amazon jungle actually pails of eight other
South American countries.
A. Protected by ... threatens B. Located in ... bypasses C. Limited to ... touches
D. Surrounded by ... borders E. Associated with ... covers

22. More valuable and comprehensive than any previously proposed theory of the phenomenon, Salarza's
research has the basis for all subsequent in their field.
A. undermine . . . advancements B. prepared ... debacles C. provided ... investigations
D. dissolved ... experiments E. reinforced .. . misconceptions

23. After several months of training, the yorurg spaniel was finally enough to be walked safely
without a leash.
A. eager ... unruly B. placid ... defiant C. clever ... helpful
D. boisterous ... docile E. vigilant ... convinced

Questions 24 to 26 are based on the following instruction:


Instruction: Fill in the gaps with suitable words
24. Susan was late for the meeting but she somehow managed to all that she had missed.
A. call on B. add to C. bring on D. catch up with E. bring up

25. Try listening to some music; it will surely help you to your thoughts about the upcoming exam results.
A. ease off B. figure out C. ease out D. ease up E. turn in

26. Many people in our small town are that big construction project because the closest store right
now is 10 miles away.
A. bearing up B. crying on C. crying up D. taking up E. showing up

27. Tire trouble with doing away with choice in schools is that there is simply no other mechanism that has
been shown to work for producing quality. Here ‘doing away with’ means-
A. scale up B. remove C. cope with D. delete E. understand
Page: 5
Questions 28 to 32 are based on the following instruction:
Instruction: The underlined part of the given sentence may contain error. Select the choice that best
replaces the underlined part keeping the original meaning intact. If you think that the original
sentence is the best choice then select A.
28. After carefully studying both of the articles, Dr. Rodriguez and Nurse Alba found that the only
difference between them were their titles.
A. them were their titles B. them were the titles C. the articles were the titles
D. the articles was that of the titles E. the articles was their titles

29. If every nation were completely self-sufficient and operated under a free-market economy, the world's
food supply will be governed solely by the economics of supply and demand.
A. will be B. would have been C. was to be D. was E. would be

30. Gold leaf is pure gold that is hammered so thin for it to take 300,000 units to make a stack one inch high.
A. for it to take B. so that it takes C. so it takes D. as for it to take E. that it takes

3 1 . Adult ladybugs often prey on agricultural or garden pests, causing many fruit growers to consider them
among the most beneficial insects.
A. causing many fruit growers B. therefore, many fruit growers C. this causes many fruit growers to
D. which cause many fruit growers to E. many fruit growers

32. Covering about 120 square miles, the New York City borough of Queens is almost as if you combine
Manhattan, the Bronx, and Staten Island.
A. if you combine Manhattan, the Bronx, and Staten Island
B. when Manhattan, the Bronx, and Staten Island are combined
C. Manhattan, the Bronx, and Staten Island combined
D. if Manhattan were to combine with the Bronx and Staten Island
E. combining Manhattan, the Bronx, and Staten Island

Questions 33 to 35 are based on the following instruction:


Instruction: Identify the sentence(s) that is/are showing the incorrect use of the underlined word.
33.1. Manif presented a persuasive argument for his salary increase.
II. Cameron persuasion was not enough to prevent Brexit.
III. They couldn't persuade their critics to see their point of view.
A. II only B. II & III only C. I & III only D. I only E. None of these

34. 1. Time and space are distorted when traveling at the speed of light.
II. Distortion of the image from a microscope can be caused by low light.
III. If others follow the same fate of Britain, then the EU may distort soon.
A. Ill only B. I & II only C. I & III only D. II only E. None of these

35. 1. In order to save themselves from boredom, US citizens visit many diversification areas.
II. Freud had many diverse interests in psychology.
III. Tire diversity of life forms on Earth makes zoology an interesting area of study.
A. I only B. II & III only C. I & II only D. I and III only E. None of these
Page: 6
Questions 36 to 40 are based on the following comprehension:
Instruction: Read the following passage and answer the accompanying questions, base your answer on
what is stated or implied in the passage. Mark your answers on the Answer Sheet.
Bangladesh is set to be one of the few countries in the Asia-Pacific region which suffer less than others
from the global economic slowdown that has come in die aftermath of the financial crisis. Bangladesh
achieved robust growth of 6.2 percent in 2008, down only slightly from 6.4 percent in 2007 despite back-to-
back floods and a devastating cyclone. With the global economic crisis deepening, growth of Bangladesh's
economy is expected to further ease to 5.5 percent in 2009, according to the Economic and Social Survey of
Asia and the Pacific-2008. This year’s issue of the flagship publication of ESCAP is titled “Addressing
Triple Threats to Development”. It analyses the three global crises which have converged to threaten
development in the Asia-Pacific region: the financial crisis, fuel and food prices, and climate change.
The survey provides a regional perspective as well country- specific analysis, outlining ways in which
economies in the region can move forward in unison towards a more inclusive and sustainable development
path. Inflation has been dr iven up in all the countries of South Asia, partly by unrelenting pressures from
higher international commodity prices, particularly the prices of oil, basic metals and selected food items.
Inflation in Bangladesh in 2008 rose to around 10 percent, up from 7.2 percent in 2007. With the fall in oil
and other commodity prices in international markets, inflation is expected to come down in 2009.
hr Bangladesh, the fiscal deficit as a percentage of gross domestic products increased fr om 3.7 percent in
2007 to 4.8 percent in 2008, despite healthy growth in government revenue. The pressure on the fiscal
balance increased due to large increase in spending on flood and cyclone relief work and an expansion of
subsidies following the rise in fuel, fertilizer and grain prices in international markets. Fiscal policy is
expected to remain expansionary to promote growth and employment, while subsidies on food and fertilizer
are expected to remain in place to contain inflation and boost the production of agr icultural crops.
The singe in prices of fuel oil, food and other commodities created severe problems for the external
balances of most countries in South Asia. However, in Bangladesh, despite an increasing trade deficit
propelled by higher imports, the current account balance showed a surplus resulting from robust growth in
export earnings and workers’ remittances. The inflow of workers’ remittances increased by one-third,
reaching nearly S8 billion in the fiscal year of 2008. If the global economic crisis persists for a longer
period, workers’ remittances my fall in coming years, causing difficulties for balance of payments.
Among long-term challenges, poverty remains a major problem for most countries in South Asia. Also,
economic and social inequalities remain widespread. Therefore, towards improving growth rate and
overcoming inequalities in these countries, the pro-poor growth sectors like agriculture, construction and
small and medium enterprises which generate employment should be supported.

36. Which of the following statements is not true in the context of economic performance of Bangladesh
during 2008?
A. Government revenue increased dining the year. B. Balance of Trade decreased din ing the year.
C. Balance of Payment increased during the year. D. Rate of inflation increased during the year.
E. Balance of Trade increased dining the year.
Page: 7
37. Which of the following, you think, should be most appropriate as the title of die passage?
A. Fallouts from Global Recession to be less severe for Bangladesh.
B. Global Financial Crisis and its Impact on Bangladesh.
C. Pro-poor Growth Sectors should be Supported in Bangladesh.
D. Trade Deficits go on Increasing in Bangladesh.
E. Government subsidies may contain inflation and boost production in Bangladesh.

38. Which of the following does not emerge as a logical forecast about the condition of Bangladesh
economy in 2009?
A. Bangladesh economy may grow at 5.5% in 2009.
B. The rate of inflation may come down in 2009, compared to that of the previous year.
C. Workers' remittances may fall in 2009 to less than S8 billion.
D. The balance of payment position of Bangladesh may be more unfavorable in 2009.
E. The rate of inflation may increase in 2009 as compared to that in 2008.

39. Howr did the economic growth rate change in 2008, compared to that of 2007?
A. It increased by 6.2% B. It decreased by 6.4% C. It decreased by 5.5%
D. It decreased by 0.2% E. It decreased by 3.3%

40. According to the ESCAP Publication which of the following was not identified as a major threat to
development in Asia-Pacific region, including Bangladesh?
A. Increased fuel prices B. Devastating floods and cyclones C. Global financial crisis
D. Changes in climatic conditions E. Increased food prices

Questions 41 to 45 are based on the following instruction:


Instruction: Underlined pails of the following sentences may contain error(s) in grammar, diction, usage or
idiom. Choose the underlined portion that contains such error(s). If there is no error then select E.
41. She (A) and other advocates believe that people in recovery could (B) play a vital role in ending (C) the
addiction epidemic, much as (D) the protest group Act Up did in the AIDS crisis. No error (E).

42. When we believe that “all we need is love,” then like Lennon, us more likely (A) to ignore fundamental
values (B) such as (C) respect, humility and commitment towards the people we care about (D). No
error (E).

43. Just (A) because you fall (B) in love with someone doesn’t (C) necessarily mean they’re a good (D)
partner for you to be with over the long term. No error (E).

44. The common cold is one of our most (A) indiscriminate diseases; It makes (B) no distinction between
(C) you and me (D), millionaires and paupers, or athletes and couch potatoes. No error (E).

45. The architect’s research shows that even when builders construct (A) houses of stone (B), they still (C)
use the hammer, more than any tool (D). No error (E).
Page: 8
Section 3: Mathematics 25x1=25 marks
Instructions: Solve each of the following problems and mark the correct answer on your Answer Sheet.
DO NOT USE A CALCULATOR. Figures are not drawn to scale.
46. When x is divided by 13 the result is y with remainder of 3. When x is divided by 7 the result is z with
... . yz
remainder of 3. If x, y and z are positive integers then what is the remainder
ofr^?
A. 3 B. 0 C. 12 D. 13 E. 10

47. Kishore asked Tazul, “What's your real age in years?” Tazul's reply was, “Take my age 3 years hence,
multiply it by 3 and subtract 3 times my age 3 years ago and you will know how old I am”. What was the
age of Tazul?
A. 18 B. 20 C. 25 D. 30 E. 35

48. The winning relay team in a high school sports competition clocked 48 minutes for a distance of 13.2
km. It’s runners A. B, C and D maintained speeds of 15 km/hr, 16 km/hr, 17 km/hr and 18 km/hr
respectively. What is the ratio of the time taken by B to the time taken by D?
A. 5: 16 B. 5: 17 C. 9: 8 D. 8: 9 E. cannot be determined

49. hr the tr iangle ABC, if AD = 16 and DC = 9 then what is the area of triangle ABC?
A. 300 B. 180 C. 175
D. 150 E. None of these

50. Avik and Niaz undertake a piece of work for Tk. 250. Avik alone can do that work in 5 days and Niaz
alone can do that work in 15 days. With the help of Ricky, they finish the work in 3 days. If everyone
gets paid in proportion to work done by them, the amount Ricky will get is:
A. Tk. 50 B. Tk. 100 C. Tk. 150 D. Tk. 200 E. None of these

51. From four comers of a rectangular sheet of dimensions 25cm X 20 cm, square of side 2 cm is cut from
four comers and a box is made. The volume of the box is:
A. 828 cm3 B. 672 cm3 C. 500 cm3 D. 1000 cm3 E. None of these

52. The average daily income of 7 men, 11 woman and 2 boys is Tk. 257.50. If the average daily income of
the men is Tk. 10 more than that of woman and the average daily income of the women is Tk. 10 more
than that of boys the average daily income of a man is:
A. Tk. 277.5 B. Tk. 250 C. Tk. 265 D. Tk. 257 E. None of these

53. A school wishes to place few desks and few benches, at least one each, along a corridor that is 16.5
meters long. Each desk is 2 meters long, and each bench is 1.5 meters long. How many maximum
numbers of desks and benches can be placed along the corridor?
A. 7 B. 8 C. 9 D. 10 E. None of these

54. Heinz produces tomato puree by boiling tomato juice. Tomato puree has 20% water whereas tomato
juice has 90% water. How many litres of tomato puree will be obtained from 20 litres of tomato juice?
A. 2 litres B. 3 litres C. 2.5 litres D. 5 litres E. None of these
Page: 9
55. Tlie number of prime factors in the expression 64 X 86 X 108 X 1210 is:
A. 80 B. 64 C. 72 D. 48 E. None of these

56. Tazul's salary & Sadib’s salary are respectively 20% and 50% more than Abir’s salary. What is the ratio
between Tazul's salary & Sadib’s salary?
A. 3:4 B. 4:5 C. 5: 6 D. 6: 7 E. 5: 7

57. Alavee and Mukit run a race with then speed in the ratio of 5:3. They prefer to run on a circular track of
circumference 1.5 km. What is the distance covered by Alavee when he passes Mukit for the seventh time?
A. 25.25 km B. 26.25 km C. 13.2 km D. 14.5 km E. None of these

58. hi a certain class, -of the male students and - of the female students speak French. If there are -as
2 3 4
many girls as boys, what fraction of the entire class speaks French?
1 2
B. - D. - E. -
4 4 3 5

59. There are five women and six men in a group. From this group a committee of 4 is to be chosen. How
many different ways can a committee be formed that contain three women and one man?
A. 10 B. 12 C. 15 D. 20 E. None of these

60. In the figure, BD and CD are angle bisectors of ABC and Z ACE, respectively. Then zBDC is equal to:

A. ZBAC B. 2ZBAC C. - ZBAC


2

D. -ZBAC E. None of these


3

61. Shanta’s salary is 75% more than Pulak’s. Shanta got a raise of 40% on her salary while Pulak got a
raise of 25% on his salary. By what percent is Shanta’s salary more than Pulak’s?
A. 96% B. 51.1% C. 90% D. 52.1% E. None of these

62. A girl sees a train passing over 1 km long bridge. The length of the train is half that of the bridge. If the
train crosses the bridge in 2 minutes, what is the speed of the train?
A. 50 km hr B. 45 km/hr C. 60 km/hr D. 30 km/hr E. 35 km/hr

63. How much Special rice costing BDT 42 per kg should a shopkeeper mix with 25 kg of ordinary rice
costing BDT 24 per kg so that he makes a profit of 25% on selling the mixture at BDT 40 per kg?
A. 20 kg B. 12.5 kg C. 16 kg D. 200 kg E. 15 kg

64. A picnic was attended by 240 persons. There were 20 more men than women and 20 more adults than
children. How many men were there in the picnic?
A. 75 B. 70 C. 55 D. 50 E. 20
Page: 10
65. A bag contains 4 red balls, 6 blue balls and 8 pink balls. One ball is drawn at random and replaced with 3
pink balls. What is the probability that the first ball drawn was either red or blue in colour and the
second drawn was pink in colour?
12 11
21
B. —
17 30
D. —
18
E. None of these

66. A bank offers 5% compound interest calculated on half-yearly basis. A customer deposits tk. 1600 each
on 1 January and 1 July of a year. At the end of the year, the amount he would have gained by way of
interest is:
A. Tk. 120 B. Tk. 122 C. Tk. 123 D. Tk. 124 E. None of these

67. Four gardeners with four grass mowers mow 400 sq. m of ground in 4 hours. How long would it take for
eight gardeners with eight grass mowers to mow 800 sq. m of ground?
A. 4 hours B. 6 hours C. 8 hours D. 12 hours E. None of these

68. If x2 + 4x - 32 < 0 , which of the following must be true?


I. | x + 2 1< 6 n. -8<x<4 III. | x + 7 |< 2
A. I only B. II only C. Ill only D. I and II only E. None of these

69. hi the following figure, ZABC = 69°, ZACB = 31° then ZBDC =?
D

A. 80° B. 60° C. 65° D. 75° E. None of these

70. A certain car used twice as many gallons from Town A to Town B as it did from Town B to Town C.
From Town A to Town B, the car averaged 12 miles per gallon, and from Town B to Town C, the car
averaged 18 miles per gallon. What is the average mile per gallon that the car achieved on its trip from
Town A through Town B to Town C?
A. 11 B. 12 C. 13 D. 14 E. 16

Part II: Writing Ability [30 marks]


1. Write about the incident that changed your life forever. 15
2. Write an essay on the following topic. "No one is wise by birth. Wisdom results from one’s own
efforts". 15

Page: 11
“In the name of .ACOih, the most beneficent, the most merciful”
CAPSTONE
Contact: 01972 277 866, 016 3031 3031 Web: www.capstonebd.com facebook.com/capstonebd

Mock Test for IBA (DU) MBA Admission

INSTRUCTION: Please read carefully! Test Booklet No: 0006


1. The test consists of two pails: I. Multiple Choice Questions (MCQ) and IL Written Ability.
This part to be filled in
2. Total duration of the test will be 120 minutes (90 for Pait-I &
30 minutes for Part II). The answer Sheet (For MCQs) will be by the Candidate
taken away after 90 minutes.
Admit No
3. On this Answer Sheet, write your Admit Number and Test Name
Booklet Number in the grids and your Name and Signature in
the boxes provided using a pen. Darken the corresponding
number boxes below the grids using a pen.

4. On this Test Booklet write your Admit Number and Name in the spaces provided using a pen.

5. Give all your answers in the Answer Sheet by completely darkening the lettered box, which represents


the correct answer to that the letter cannot be seen USE Pen ONLY.
®® ®©
6. Do rough work anywhere on the rest booklet,
This part to be filled in by the invigilator
except in the spaces provided for written answers.
Do not use the answer sheet or extra paper for Please verify that
rough work. Candidate’s Name & Admit Number matches the
information on the admit card.
7. For each MCQ, 25% of assigned marks will Candidate’s appearances resembles the photograph
be deducted for each wrong answer. Each on the Admit Card.
unanswered question will earn zero marks.
Admit No
8. Candidates are required to obtain a minimum Signature of the Invigilator
acceptable mark in each section and in each
part.

9. Switch off your mobile phones.

10. Your script will not be examined if you adopt any unfair means.

11. Do not use a calculator or any other electronic devices.

12. Remain seated silently at the end until you are asked to go.

|sTOP! DO NOT TURN THIS PAGE UNTIL YOU ARE ASKED TO DO SO.|
For Office Use Only
1 2 | 3
Part I: Multiple Choice Questions [70 marks]
Section 1: Mathematics 30x1=30 marks
Instructions: Solve each of the following problems and mark the correct answer on your Answer Sheet.
DO NOT USE A CALCULATOR. Figures are not drawn to scale.

1. 13 men and 7 boys can finish a job in 7 days, while 6 boys and 13 women can finish the same job in 6
days, hi how many days can 1 man, 1 boy and 1 woman working together finish the same job (assume
constant rates each for men, women, and boys)?
A. 21 B.24 C. 42 D. 54 E. None of these

2. After the division of a number successively by 3, 4 and 7, the remainder obtained is 2, 1 and 4
respectively. What will be remainder if 84 divides the same number?
A. 80 B.75 C. 42 D. 53 E. None of these

3. A certain game is played with a total of 10 cards, numbered 1 through 10. A player wins if his or her
hand contains two cards with numbers whose product is 6. What is the maximum number of cards that a
player can have in his or her hand without winning?
A. Five B. Six C. Seven D. Eight E. None of these

4. Raffle tickets numbered consecutively from 101 through 350 are placed in a box. What is the probability
that a ticket selected at Random will have a number with a hundreds digit of 2?
A.-5 B.-
7
C.-
83 —
D. 250 —
E. 249

5. A team contributes total of Tk 399 from its members. If each member contributed at least Tk 10, and no
one contributed Tk 19, what is the greatest number of members the club could have?
A. 37 B.38 C. 39 D. 40 E. None of these

6. A boat takes 90 minutes less to travel 36 miles downstream than to travel the same distance upstream. If
the speed of tire boat in still water is 10 mph, the speed of the stream is:
A. 2 mph B. 2.5 mph C. 3 mph D. 4 mph E. None of these

7. What is tire 25th digit to the right of the decimal point in the decimal form of
A. 3 B.4 C. 5 D. 6 E. 7

8. Tire average weight of 15 items is 8 kg. Tire least average weight of 5 items of these 15 is 5 kg. If no
item weighs less than 5 kg, what is the maximum number of items that can have the same weight?
A. 3 B.5 C. 6 D. 14 E. None of these

9. If 3 < y < 150, for how many values of y is the square root of y twice a prime number?
A. Two B. Three C. Four D. Five E. None of these
Page: 1
10. The present ratio of ages of A and B is 4:5. Eighteen years ago, this ratio was 11:16. Find the sum of
their present ages.
A. 90 B. 105 C. 110 D. 80 E. None of these

11 . Solve the inequality considering real numbers: |


^58
a
A. - oo < x < —53 ^8-<x<oo
B.
3
C. -oo< x <— or- < x <co
2 3
s8
2

D. - oo < x < and - < x < oo
3
E. None of these

12. The difference between compound interest and simple interest on an amount of tk. 15,000 for 2 years is
tk. 96. What is the rate of interest per annum?
A. 8 B. 10 C. 12 D. 14 E. None of these

13. Ricky attends a magic show and the magician asks him to pick three different numbers between 10 and
20 inclusive. Assuming that Ricky selects the numbers randomly, what is the probability that Ricky will
pick the numbers 12 and 15 as two of his three selections?
A.—
2
42
B—
55
3
—1
C. 15
1
D. -
9 —2
E. 11

14. Distance between Chittagong and Rajshahi is 310 miles. At noon Sarowar left Chittagong for Rajshahi,
Rakib left Rajshahi for Chittagong. If Sarowar travels at 42 miles per hour and Rakib travels 36 miles
per hour, how many miles apart will Rakib and Sarowar be 1 hour before they meet?
A. 86 B. 38 C. 78 D. 56 E. Cannot be determined

15. Pial and Avik invest in a business in the ratio 3:2. If 5% of the total profit goes to charity and Pial's share
is Tk. 855, the total profit is:
A. Tk. 1425 B. Tk. 1500 C. Tk. 1537.50 D. Tk. 1576 E. None of these

16 At the beginning of the school year, 50% of all students in Mr. Wells' math class answered "Yes" to the
question "Do you love math", and 50% answered "No." At the end of the school year, 70% answered
"Yes" and 30% answered "No." Altogether, x% of the students gave a different answer at the beginning
and end of the school year. What is the difference between the maximum and the minimum possible
values of x?
A. 0 B. 20 C. 40 D. 60 E. None of these

17. Two guns are fired from the same place at an interval of 6 minutes. Anik approaching the place observes
that 5 minutes 52 seconds have elapsed between the hearings of the sound of the two guns. If the
velocity of the sound is 330 m/sec, Anik was approaching that place at what speed (in km/h)?
A. 24 kmph B. 27 kmph C. 30 kmph D. 36 kmph E. None of these

18. What number must be added to (x" -18x) to make the result a perfect square?
A. 81 B.27 C. x+3 D. x + 9 E.x-3
Page: 2
19. If 75 percent of a class answered the first question on a certain test conectly, 55 percent answered die
second question on the test correctly, and 20 percent answered neither of the questions conectly, what
percent answered both correctly?
A. 10% B. 20% C. 25% D. 50% E. None of these

20. The angles of a triangle are in the proportion of 1:2:3 and the length of the smallest side is 1. What is the
length of the longest side of the triangle?
A. 4 B. 5 C. 2 D. 3 E. None of these

21 . A total of 22 men and 26 women were at a party. Average age of all the adults at the party was 35 years.
Average age of the men was 38 years, which of die following was closest to average age of women?
A. 31 B. 31.5 C. 32 D. 32.5 E. 33

22. A 270 meters long train running at the speed of 120 kmph crosses another train running in opposite
direction at the speed of 80 kmph in 9 seconds. What is the length of the other train?
A. 230 m B. 240 m C. 260 m D. 320 m E. None of these

23. Find the smallest number which when divided by 18 and 24 will have a remainder of 4 and 10
respectively.
A. 56 B.52 C. 58 D. 86 E. None of these

24. A large tanker can be filled by two pipes A and B in 60 minutes and 40 minutes respectively. How many
minutes will it take to fill the tanker from empty state if B is used for half the time and A and B fill it
together for the other half?
A. 15 B.20 C. 27.5 D. 30 E. None of these

25. Mary decided to save a certain amount of her monthly salary each month and her salary was unchanged from
month to month. If Mary's savings by the end of the year from drese monthly savings were thr ee times the
amount she spent per month, what should be the fraction of her salary that she spent each month?
A.-
2
B-3 C.-
4
D. -5 E. -
6

Questions 26 to 30 are based on the following information:


A. Statement (i) alone is sufficient but statement (ii) alone not sufficient to answer the question asked.
B. Statement (ii) alone is sufficient but statement (i) alone not sufficient to answer the question asked.
C. Both statement (i), (ii) together are sufficient to answer the question but neither statement is sufficient alone.
D. Each statement alone is sufficient to answer the question.
E. Statement (i) & (ii) are not sufficient to answer the question asked and additional data is needed to
answer the statements.

26. M andN are integers such that 6<M<N. What is the value ofN?
i. The greatest common divisor of M and N is 6 ii. The least common multiple of M and N is 36
Page: 3
'll. For all non - zero integers n, n* = what is the value of x?
i. x*=x ii. x*=-2-x

28. Does the integer k have a factor p such that 1 < p < k?
i.k>4! ii. 13!+2≤k≤13!+13

29. What is the remainder when integer n is divided by two?


i. When n is divided by 5, the remainder is even ii. When n is divided by 7, the remainder is even

30. Each employee who works in company X works in exactly one of departments A, B, and C. What
fraction of all the employees who work in company X work in department A?
i. The ratio of the total number of employees who work in departments A and C to the number of
employees who work in department B is 7 to 3.
ii. The average (arithmetic mean) of the numbers of employees who work in departments A, B, and C is
times the number of employees who work in department A.

Section 2: Language & Communication 25x1=25 marks


Questions 31 to 35 are based on the following instruction:
Instruction: Underlined parts of the following sentences may contain error(s) in grammar, diction, usage or
idiom. Choose the underlined portion that contains such error(s). If there is no error then select E.

31 . Whenever (A) leaders face a problem, they automatically (B) measure it (C) and begin solving it using (D)
tire law of Intuition. No error (E).

32. Opportunistic entrepreneurs are optimistic individuals with the ability!A) to pick out financial opportunities,
get in at(B) the right time, staying on(C) board during the time of growth, and exit when a business hits its
peak!D). No error (E).

33. These types of entrepreneurs are concerned with(A) profits and the wealth they will build; so(B) they are
attracted to(C) ideas where they can create(D) residual or renewal income. No error (E).

34. This js not (A) you (B) because (C) these people never pick up (D) a leadership book. No error (E).

35. Train yourself to (A) think in terms (B) of mobilizing people and harness (C) their resources (D). No error
(E).
Questions 36 to 40 are based on the following instruction:
Instruction: Choose the conect sentence from the following options.
36. A. I live here since childhood and I think I know everyone in our village.
B. I had been living here since childhood and I think I know everyone in our village.
C. I have lived here since childhood and I think I know everyone in our village.
D. I was living here since childhood and I think I know everyone in our village.
E. None of die sentences is con ect.
Page: 4
37. To judge the Tidy City contest, we picked an uninterested party.
A. picked a disinterested party.
B. picked an interested party!
C. picked an uninterested, party.
D. are in the process of picking an uninterested party.
E. picked an disinterest party

38. One must bear tire fact in mind that the significance of Indian Ocean is because .

A. of the increased activity in B. the activity have increased there


C. of the increasing activity therein D. activities has been increasing therein
E. of activity which are on the rise therein

39. A. If you had told me that you were in Bombay I had certainly contacted by you instead of getting bored there.
B. If you had told me that you were in Bombay I would have certainly contacted you despite getting bored there.
C. If yorr had told me that you were in Bombay I would have certainly contacted you instead of getting
bored there.
D. If yorr would have told me that you were in Bombay I had certainly contacted you instead of getting
bored there.
E. If you would have told me that you were in Bombay I would have certainly contacted you instead of
getting bored there.

40. Many writers of modem English have acquired careless habits that damage the clarity of their prose, but
these habits can be broken if they are willing to take the necessary trouble.
A. but these habits can be broken B. but these habits are breakable
C. which can be broken D. but they can break these habits
E. except that can be broken

Questions 41 to 45 are based on the following instruction:


Instruction: Fill in the blanks with the suitable words.
41. Although the Industrial revolution brought about a raft of new techniques in the area of machine design,
the methods used for the printing and binding of books continued till the early years of the 20th
century.
A. intermittently B. improving C. transforming D. unchanged E. advancing

42. We have criticized our university students for preferring the security of political silence and the safety of
to the excitement of social and humanitarian action.
A. acquiescence, dissent B. college, adventure C. concealment, revolution
D. tolerance, antagonism E. security, insecurity
Page: 5
43. He is so that lie immediately believed my stoiy of ghosts.
A. innocent B. credulous C. vociferous D. credible E. terrified

44. The party leader did expect some criticism of his decision, but was taken aback by the near of the
members.
A. mutiny B. unanimity C. mutilation D. momentum E. None of these

45. At one point, it looked as if an area of agreement would specially over the issue of productivity
linked wages.
A. develop B. come out C. emerge D. grow E. put out

Questions 46 to 50 are based on the following instruction:


Instruction: Choose the correct use of the idioms/ phases.

46. Frank, ! Don’t you see that we are driving through the same street again?
A. on the same page B. you brave C. use your loaf
D. rise and shine E. till the cows come home

47. I will pass the IBA admission test.


,

A. Come hell or high water B. Curiosity killed the cat C. Down to wire
D. Each cloud has a silver lining E. None of the above

48. These are . Enjoy them because the rainy autumn is at the doorstep.
A. funny days B. dog days of summer C. under the weather
D. cups of joe E. fool’s gold

49. Abir is . He is going to take the exams next week.


A. burning the midnight oil B. drinking like a fish C. going down like a lead balloon
D. going out on a limb E. None of the above

50. I have tried to pass the driving test twice, but I couldn’t succeed. You should definitely try again. After
all, !
A. keep body and soul together B. let bygones be bygones C. elvis have left the building
D. hell or high water E. the third times the charm
Questions 51 to 55 are based on the following comprehension:
Instruction: Read the following passage and answer the accompanying questions, base your answer on
what is stated or implied in the passage. Mark your answers on the Answer Sheet.

The object underlying the rules of natural justice "is to prevent miscarriage of justice" and secure "fair
play in action" As pointed out earlier the requirement about recording of reasons for its decision by an
administrative authority exercising quasi-judicial functions achieves his object by excluding changes of
arbitrariness and ensuring a degree of fairness in the process of decision making. Keeping in view the
Page: 6
expanding horizon of the principle of natural justice which govern exercise of power by administrative
authorities. Tire rules of natural justice are not embodied rules. Tire extent of their application depends upon
the particularly statutory framework where under jurisdiction has been conferred on the administrative
authority. With regard to tire exercise of particular power by an administrative authority including exercise
of judicial or quasi-judicial functions the legislature, while conferring the said power, may feel that it would
not be in the larger public interest that the reasons for the order passed by the administrative authority be
recorded in the order and be communicated to the aggrieved party and it may dispense with such a
requirement.

51. “The rules of the natural justice are not embodies rules” means that these rules
A. are left deliberately vague B. cannot be satisfactorily interpreted C. are flexible
D. cannot be visualized E. None of the above

52. From the passage it is clear that it is the legislature that


A. invests the administrative authority with enormous powers
B. embodies rules
C. has the larger interests of public welfare
D. leaves administrative authority enough discretion to interpret rules
E. None of the above

53. According to the passage, there is always a gap between


A. rules of natural justice and their application B. conception of a rule and its concretization
C. demand for natural justice and its realization D. intention and execution
E. None of the above

54. “To dispense with a requirement” means


A. to do without the demand B. to drop the charge C. to cancel all formal procedure
D. to alter the provisions of the case E. None of die above

55. According to the passage, natural justice can be brought about by


A. administr ative authority remaining vigilant
B. administrative authority upholding rules of natural justice
C. administrative authority fanning rules suitably
D. administrative authority observing the rules of fair play E. None of these

Section 3: Analytical Ability 15x1=15 marks


Instruction: Solve each of the following problems and mark the correct answer on your Answer Sheet.
56. Tire economy is doing badly. First, the real estate slump has been with us for some time. Second, car
sales are at their lowest in years. Of course, had either one or the other phenomenon failed to occur, this
would be consistent with the economy as a whole being healthy. But, their occurrence together makes it
quite probable that my conclusion is conect.
Page: 7
Which one of the following inferences is most strongly supported by the information?
A. If car sales are at their lowest in years, then it is likely that the economy is doing badly.
B. If the economy is doing badly, then either the real estate market or the car sales market is not healthy.
C. If the real estate market is healthy, then it is likely that the economy as a whole is healthy.
D. If the economy is in a healthy state, then it is unlikely that the real estate and car sales markets are
both in a slump.
E. The bad condition of the economy implies that both the real estate and the car sales markets are
doing badly.

57. The earth's resources are being depleted much too fast. To correct this, the United States must keep its
resource consumption at present levels for many years to come.
Which of the following, if true, would most strengthen the argument above?
A. New resource deposits are constantly being discovered.
B. The United States consumes one-third of all resources used in the world.
C. Other countries need economic development more than the United States does.
D. Other countries have agreed to hold their resource consumption at present levels.
E. The United States has been conserving resources for several years.

Questions 58 to 61 are based on the following information:


Mehmood Trucking Company trips always begin in one of die five cities served by Mehmood-Noshara,
Okara, Pashwar, Queta, and Rahim yar khan - and always end in one of these cities. Mehmood drivers
travel from city to city only on the routes listed in the mileage table below.

The routes listed above have no stretch or road in common.


The maximum distance an Mehmood driver can travel without making an overnight stop is 300 miles.
Overnight slops must be made in cities served by Mehmood.

58. Which of the following is a trip an Mehmood driver could make without an overnight stop?
A. From Noshara to Rahim yar khan to Qrreta B. From Rahim yar khan to Queta to Pashwar
C. From Queta to Rahim yar khan to Noshara D. From Noshara to Pashwar to Queta
E. From Qrreta to Pashwar to Noshara to Okara

59. What is the minimum number of overnight stops that an Mehmood driver who travels from Okara to
Noshara to Rahim yar khan to Queta must make before arriving in Queta?
A. 1 B. 2 C. 3 D.4 E. 5

Page: 8
60. Which of the following is a list of the cities to which Mehmood driver can travel from Pashwar,
returning to Pashwar on the same day he or she left?
A. Noshara only B. Okara only C. Queta only
D. Both Noshara and Okara E. Both Noshara and Queta

61. Which of the following accurately describes the possibilities for overnight stops during trips by
Mehmood drivers that begin in Okara and end in Quaker-town after including two other cities served by
Mehmood?
A. Tire trips can be made without an overnight stop.
B. It is not possible to make an overnight stop at Noshara.
C. Rahim yar khan is the only possible place to make an overnight stop.
D. If exactly two overnight stops are made on a trip, they can be made only at Pashwar and Rahim yar khan.
E. Any one of three cities- Noshara, Pashwar, Rahim yar khan- is a possible overnight stop.

Questions 62 to 65 are based on the following information:


On a cost sheet, a manager is filling in numbers on eleven lines- J, N, O, P, S, T, U, W, X, V, and Z-
according to the following instructions and no others:
Line J must be derived from lines N, O, and V as N plus O minus V.
Line T. must be derived from lines P and S. as P plus S.
Line W must be derived from lines T and U as T minus U.
Line X must be derived from lines W and J is W minus J.
Line V must be derived from fine T as five percent of T
Line Z must be derived from line X as half of X.

62. V cannot be derived unless which of the following is known?


A. J B. N C. O D. P E. U

63. J cannot be derived unless which of the following is known?


A. S B. U C. W D. X E. Z

64. It is necessary for the manager to know O before the manager can derive
A. T B. U C. W D. X E. V

65. If the manager knows P, O, S, and U but not N. which of die following is a pair- of lines that the manager
can derive?
A. J and V B. T and W C. T and X D. X and V E. X and Z

Page: 9
Questions 66 to 70 are based on the following information:
The following line graph gives the ratio of the amounts of imports by a company to the amount of exports
from that company over the period from 1995 to 2001.

Ratio of Value of Imports to Exports by a Company Over the Years.

66. If the imports in 1998 were Tk. 250 crores and the total exports in the years 1998 and 1999 together was
Tk. 500 crores, then the imports in 1999 was?
A. 250 Cr B. 300 Cr C. 357 Cr D. 420 Cr E. None of these

67. The imports were miniminn proportionate to the exports of the company in the year?
A. 1995 B. 1996 C. 1997 D. 2000 E. None of these

68. What was the percentage increase in imports from 1997 to 1998?
A. 72 B.56 C. 28 D. 52 E. None of these

69. If the imports of the company in 1996 was Tk. 272 crores, the exports fr om the company in 1996 was?
A. 370 Cr B.320Cr C. 280 Cr D. 275 E. None of these

70. hr how many of the given years were the exports more than the imports?
A. 1 B. 2 C. 3 D. 4 E. None of these

Part II: Writing Ability [30 marks]


1. Write a paragraph using the following words. You must give an appropriate title and CAN change the
formation of the following words if necessary. You must underline the following words when you use.15
creativity overhauling education system problem-solving skills middle-income status
essential skills outdated disparities nation's advancement Technical education

2. “Technical education is imperative to reduce unemployment in out country”- Do you support the
statement. If you do so, show your logics. 15
Page: 10
“In the name of .ACOih, the most beneficent, the most merciful”
CAPSTONE
Contact: 01972 277 866, 016 3031 3031 Web: www.capstonebd.com facebook.com/capstonebd

Mock Test for IBA (DU) MBA Admission

INSTRUCTION: Please read carefully! Test Booklet No: 0007


1. The test consists of two pails: I. Multiple Choice Questions (MCQ) and IL Written Ability.
This part to be filled in
2. Total duration of the test will be 120 minutes (90 for Pait-I &
30 minutes for Part II). The answer Sheet (For MCQs) will be by the Candidate
taken away after 90 minutes.
Admit No
3. On this Answer Sheet, write your Admit Number and Test Name
Booklet Number in the grids and your Name and Signature in
the boxes provided using a pen. Darken the corresponding
number boxes below the grids using a pen.

4. On this Test Booklet write your Admit Number and Name in the spaces provided using a pen.

5. Give all your answers in the Answer Sheet by completely darkening the lettered box, which represents


the correct answer to that the letter cannot be seen USE Pen ONLY.
®® ®©
6. Do rough work anywhere on the rest booklet,
This part to be filled in by the invigilator
except in the spaces provided for written answers.
Do not use the answer sheet or extra paper for Please verify that
rough work. Candidate’s Name & Admit Number matches the
information on the admit card.
7. For each MCQ, 25% of assigned marks will Candidate’s appearances resembles the photograph
be deducted for each wrong answer. Each on the Admit Card.
unanswered question will earn zero marks.
Admit No
8. Candidates are required to obtain a minimum Signature of the Invigilator
acceptable mark in each section and in each
part.

9. Switch off your mobile phones.

10. Your script will not be examined if you adopt any unfair means.

11. Do not use a calculator or any other electronic devices.

12. Remain seated silently at the end until you are asked to go.

|sTOP! DO NOT TURN THIS PAGE UNTIL YOU ARE ASKED TO DO SO.|
For Office Use Only
1 2 | 3
Part I: Multiple Choice Questions [75 marks]
Section 1: Mathematics 30x1=30 marks
Instructions: Solve each of the following problems and mark the correct answer on your Answer Sheet.
DO NOT USE A CALCULATOR. Figures are not drawn to scale.

1. Emon received a 10% raise each month for 3 consecutive months. If his salary after the 3 raises is
13310, what was his starting salary?
A. 9000 B. 10000 C. 11000 D. 12000 E. None of these

2. Three numbers which are co-prime to each other are such that the product of the first two is 551 and that
of the last two is 1073. The sum of the three numbers is:
A. 75 B. 81 C. 85 D. 89 E. None of these

3. The cost of Type 1 rice is Tk. 15 per kg and Type 2 rice is Tk. 20 per kg. If both Type 1 and Type 2 are
mixed in the ratio of 2:3, then the price per kg of the mixed variety of rice is:
A. Tk. 18 B. Tk. 18.5 C. Tk. 19 D. Tk. 19.50 E. None of these

4. When 10 is divided by a positive integer n, the remainder isn-4, which of the following could be the
value of n?
A. 3 B. 4 C. 7 D. 8 E. 9

5. A car is traveling on a highway. At 10 am, it passes a truck traveling in the same direction. The truck
continues on the highway traveling at 50 mph while the car travels at 65 mph. How far apart are the car
and the truck at 2 pm?
A. 15 miles B. 30 miles C. 60 miles D. 200 miles E. 260 miles

6. Number of students in institutes A and B were in the ratio of 7:15 respectively in 2012. hr 2013, the
number of students in institute A increased by 25% and number of students in institutes B increased by
26%, then what was the respective ratio between number of students in institutes A and B?
A. 25:56 B. 25:54 C. 24:55 D. 25:53 E. None of these

7. Ifx>2yandy< -5 , which of the following must be tme?


x
A. ->2
y
B. x>y c.-<y D. —
y
<2 E. None of these

8. Mr. Siam is a vegetable fanner. His neighbor’s goat regularly enters his vegetable garden and destroys
Iris vegetables. To protect his vegetables Mr. Siam wants to put up fencing around three sides of Iris
rectangular yard and leave a side of 20 feet unfenced as that side is facing the canal and needs no fence.
If the garden has an area of 680 square feet, how many feet of fencing does he need?
A. 34 B. 40 C. 68 D. 88 E. None of these

9. Tire average (arithmetic mean) of four different positive integers is 20. What is the greatest possible
value of any of these integers?
A. 68 B. 70 C. 74 D. 73.4 E. 77
Page: 1
10. Slialiadat & Anik are the billionaires of Bangladesh. Shahadat and Anik pay 30% and 40% tax annually,
respectively. If Shahadat makes $56000 and Anik makes $72000, what is then combined tax rate?
A. 32% B. 34.4% C. 35% D. 35.6% E. None of these

11. In 2004, approximately |of the 37.3 million airline passengers traveling to or from Bangladesh used
Dhaka Airport. If the number of such passengers that used Chittagong Airport was|the number that
used Dhaka Airport and 4 times the number that used Jessore Airport, approximately how many millions
of these passengers used Jessore Airport that year?
A. 18.6 B. 9.3 C. 6.2 D. 3.1 E. 1.6

n
2435 x32n+l
12. :— =?
9“x3n-1
A. 1 B. 2 C. 9 D. 3n E. None of these

13. A starts business with Tk. 3500 and after 5 months, B joins with A as his partner. After a year, the profit
is divided in the ratio 2:3. What is B's contribution in the capital (Tk.)?
A. 7500 B. 8000 C. 8500 D. 9000 E. None of these

14. A man completes -h of a job in 10 days. At this rate, how many more days will it takes him to finish the job?
A. 5 B. 6 C. 7 D. 7.5 E. None of these

15. A train overtakes two persons walking along a railway track. Tire first one walks at 4.5 km/hr. Tire other
one walks at 5.4 km/hr. Tire train needs 8.4 and 8.5 seconds respectively to overtake them. What is the
speed (kph) of the train if both the persons are walking in the same direction as the train?
A. 66 B. 72 C. 78 D. 81 E. None of these

16. hr triangle ABC to the right, if BC = 3 and AC = 4, then what is the length of
segment CD? /4 X.
A. 3 B.y C. 5 D.^ E. None of these B 3 C D

17. Tire average weight of 45 students in a class is 52 kg. Five of them whose average weight is 48 kg leave
the class and other 5 students whose average weight is 54 kg join the class. What is the new average
weight (in kg) of the class?
112 7
A. 52- B. 52- C. 52- D. 52- E. None of these
3 2 3 8

18. Recently, a small village, in Cox’s Bazar where only male shepherd reside with four sheep each, was
devastated by Tsunami waves. Therefore 8 persons and 47 sheep were found to be dead and the person
who luckily survived, left the village with one sheep each since 21 sheep were too injured to move so
have left on their luck, in the village. The number of sheep which were earlier in the village was:
A. 84 B. 120 C. 80 D. 90 E. None of these

Page: 2
19. A total of 5 liters of gasoline is to be poured into two empty containers with capacities of 2 liters and 6
liters, respectively, such that both containers will be filled to the same percent of their respective
capacities. What amount of gasoline, in liters, must be poured into the 6-liter container?
A. 4- B. 4 C. 3- D. 3 E. 1-
2 4 4

20. A trader bought 48 tubes and found that 8 of the tube lights were broken during transportation. As a
result his per unit cost was increased by tk. 24. What was the cost of each tube light?
A. 100 B. 104 C. 112 D. 120 E. None of these

21. If x and y are nonzero integers such that - 12 x 1 and - 4 y 1 0, what is the sum of the minimum
and the maximum possible values of-?
A. 0 B. —4 —
C. 8 D. 4 E. None of these

22. Tire curved surface area of a cylindrical pillar is 264 sq.m and its volume is 924 cubic m. Find the ratio
of its diameter to its height.
A. 3:7 B. 7:3 C. 6:7 D. 7:6 E. None of these

23. (441 + 442+ 443) is divisible by


A. 21 B. 13 C. 15 D. 17 E. 11

24. hr a hockey championship, there are 153 matches played. Every two-team played one match with each
other. The number of teams participating in the championship is:
A. 18 B. 19 C. 17 D. 16 E. None of these

25. At the beginning of the year, an item had a price of A. At the end of January, the price was increased by
60%. At the end of February, the new price was decreased by 60%. At the end of March, the new price
was increased by 60%. At the end of April, the new price was decreased by 60%. On May 1st, the final
price was approximately what percent of A?
A. 41% B. 64% C. 100% D. 136% E. 159%

26. The number of degrees that the hour hand of a clock moves through between noon and 2.30 in the
afternoon of the same day is
A. 80 B. 75 C. 70 D. 65 E. 60

27. Four identical semi-circles overlap with a square on four sides, respectively. If the sides of the square is
4, and the entire figure is inscribed in a circle. What is the area of the shaded area?
A. 4k - 8
D. 8k -16
B. 4k 12 —
E. None of these
C. 8k 12 —
28. A wheel has a diameter of x inches and a second wheel has a diameter of y inches. The first wheel covers a
distance of d feet in 100 revolutions. How many revolutions does the second wheel make in covering d feet?
A. lOOxy B. lOOy-x C. lOOx-y D. E.
Page: 3
29. Of 45 families in a locality, 25 families have working mothers and 10 families have retired individuals as
members to look after the childr en at home. Of the families, 8 have both working mothers and retired
individuals as member. How many of the families have working mothers but no retired individuals as
members?
A. 0 B. 2 C. 18 D. 17 E. None of these

30. Half the people on a bus get off at each stop after the first, and no one gets on after the first stop. If only
one person gets off at stop number 7, how many people got on at the first stop?
A. 128 B. 64 C. 32 D. 16 E. 8

Section 2: Language & Communication 25x1=25 marks


Questions 31 to 35 are based on the following instruction:
Instruction: Fill in the gaps with suitable word from the alternatives.
It is clear from recent news reports that the condition of Rohingya refugees in Bangladesh is becoming
increasingly vulnerable. On Wednesday, a Rohingya man was (31) his house in Balukhali refugee
camp and shot dead. The day before, another was killed at the Kutupalong camp. Last week, a young
Rohingya man was killed by miscreants, not long after his father was also killed in the Tajnimar Kliola 19
camp. hi the meantime, the Myanmar military junta has started die process of transferring the ownership of
land that originally belonged to Rohingya refugees, often for generations, before they were forced to flee in
2017 - thus adding a new layer of (32). hi the midst of all this, it is reprehensible that criminal
elements have been preying on this vulnerable group of refugees and dangling the carrot of Bangladeshi
citizenship in front of them in exchange for exorbitant amounts of money. According to a report in this
daily, a syndicate operating from Chattogram city has been charging Tk. 1.30 lakh per head to issue fake
national identity (NID) cards for Rohingya refugees. This is die diird time that such a scam has been
discovered in the area. Out of the 10 people who were arrested by Chattogram Metropolitan Police in this
connection, we are bonified to find that five are local Election Commission (EC) employees. Police
investigations have revealed that these are data entry operators engaged in creating updated voter lists, and
they have been abusing their positions to make (33). The fact that some employees of the institution
that is charged with organizing free and fair elections in the country (34) indeed alarming. If NIDs
can be issued so easily from a random high school in Chattogram, does that mean voter lists can be
manipulated with equal ease? If so, where else are they being tampered with? And what is the EC doing to
ensure that their employees (35)?
31. A. picked on from B. picked for C. picked to D. picked by E. picked up from

32. A. complications of any process B. complications to any potential repatriation process


C. complications of any processes D. complications of process E. None of these

33. A. a quick buck at the expense of national security B. an buck of the expenses of security
C. a quick buck among the expense for national security D. a quick buck of the expense
E. a quick buck to the expense

34. A. are engaging in such exploitative practices is B. are engaged in such exploitative practice is
C. are engaged in such exploitative practices is D. are engaged in such an exploitative practices is
E. are engaged in such exploitative practices are
Page: 4
35. A. are not engaging this kind of corruption B. are not engaging in this kind of corruption
C. are not engaging this kinds of corruption D. are engaging this kind of corruption
E. are not engaging these kind of corruption

Questions 36 to 39 are based on the following instruction:


Instruction: Which of the following set of words belong to the same group [i.e. similar meaning/ synonymous]
36. A. Abhorrent, Loathsome, Repugnant B. Heinous, Renounce, Wayward
C. Defamation, Copious, Extol D. Acquit, Allegation, Censure
E. No similar group

37. A. Forfeit, Bizarre, Abundant B. Commendable, Condemnable, Deplorable


C. Antagonistic, Bellicose, Quarrelsome D. Accord, Alleviate, Allure
E. No similar group

38. A. Amiable, Affable, Amicable B. Apex, Apathy, Ancient C. Appentice, Appropriate, Ardent
D. Arduous, Altercation, Arrogant E. No similar group

39. A. Articulate, Atrocious, Ascetic B. Banal, Hackneyed, Trite C. Banish, Banter, Boor
D. Knell, Procreate, Daze E. No similar group

Questions 40 to 43 are based on the following instruction:


Instruction: Underlined parts of the following sentences may contain error(s) in gr ammar, diction, usage or
idiom. Choose the underlined portion that contains such enor(s). If there is no error then select E.

40. That (A) way they would (B) not have to cany every (C) bit of its (D) supplies with them the whole
trip. No error (E).

41. Followers need leaders bg (A) able to effectively (B) navigate (C) for them (D). No error (E).

42. Tire truth is that nearly (A) anyone can steer the ship, but it (B) takes a leader to chart (C) the course (D).
No error (E).

43. Wiry do I (A) even mention (B) something that seem (C) so basic (D)? No error (E).

Questions 44 to 46 are based on the following instruction:


Instruction: The underlined part of the given sentence may contain error. Select the choice that best
replaces the underlined part keeping the original meaning intact. If you think that the original
sentence is the best choice then select A.
44. Umar-bin-Abdul Aziz was a Khalif of the Muslim world, while earlier his great grandfather Umar (ra)
has been a companion of Prophet Muhammad (pbuh) & the 2nd Khalif of the Muslim world.
A. while earlier his great grandfather Umar (ra) has been
B. where his great grandfather Umar (ra) earlier is
C. just as earlier his gr eat gr andfather Umar (ra) had been
D. as his earlier great gr andfather Umar (ra) has been
E. his great gr andfather Umar (ra) earlier being
Page: 5
45. Two disabled children, one with crutches and the other one w ith a wheelchair, enters the class on
Monday.
A. the other one with a wheelchair, enters B. the other one a wheelchair, enter
C. the other with a wheelchair, enters D. the other with a wheelchair, enter
E. one with a wheelchair, enters

46. The incidence of rape in rural areas is equally high or more so than in urban areas.
A. equally high or more so than in urban areas B. equal to or higher than in urban areas
C. as high as in urban areas or more D. equal to, if not more, than in urban areas
E. as high as it is in urban areas, if not higher

Questions 47 to 50 are based on the following instruction:


Instruction: Fill in the blanks with the most appropriate word.

47. Uber because we for our car for 5 hours. We totally exhausted.
A. aplogised / had been waiting / had been B. had apologised / waited / were
C. apologised / were waiting / were D. apologised / had been waiting / were
E. None of these

48. Poverty eclipses the brightest virtues, and is very sepulcher of brave designs, what nature has
fitted him for and stifling the noblest thoughts in their embryo.
A. deprive man of means to have accomplished B. depriving a man of the means to accomplish
C. deprived man of the means in accomplishing D. deprive a man in means to accomplish
E. depriving a man of the means for accomplishing

49. Although he did not consider himself , he felt that the inconsistencies in her story a certain
degree of incredulity on his part.
A. an apostate justified B. an optimist intimated C. a hypocrite demonstrated
D. a charlatan dignified E. a skeptic warranted

50. Critics were misled by Williams obvious exaggerated theatrical gestures into his plays as
mere melodramas, “full of sound and fury, signifying nothing”.
A. disinclination for disparaging B. repudiation of misrepresenting
C. indulgence in acclaiming D. penchant for denigrating
E. indifference to lauding

Questions 51 to 55 are based on the following comprehensions:


Instruction: Each of the following passages is followed by a set of questions. Read each passage and
answer the accompanying questions, base your answer on what is stated or implied in the
passage. Mark your answers on the Answer Sheet.
The hypothesis that people ascribe more value to things merely because they own them is at the heart of
a highly influential theory in behavioral economics, known as the endowment effect. Economists ascribing
to tire theory have tried to illustrate this effect with the help of several experiments that seem to show that
people value the things they own more than they value identical products they do not own and that they
prefer to hold on to those endowed items rather than trade or sell them. Endowment theory says that such
Page: 6
observable events, whatever their other possible causes, are at least partially explained by the general
phenomenon of loss aversion, which holds that people give possible losses more weight than potential gains
of the same magnitude. Endowment theory is an application of this phenomenon, adding the hypothesis that
ownership determines whether one experiences a change as a gain or as a loss. Endowment theory posits that
ownership sets one’s reference point, the movement from which triggers either a perceived gain or loss, and
that people perceive the transfer or sale of endowments as losses.
Although endowment theory continues to enjoy considerable influence, recent experimental data have
cast doubt on the hypothesis that ownership sets the reference point and that loss aversion generates the
reluctance to trade. The empirical support for endowment theory was never perfect, hi recent years,
experimentalists have published data suggesting that the results of earlier laboratory experiments were not
caused by loss aversion but by other factors. By making a few changes to the experimental design —such as
better training subjects in the auction mechanisms used in the experiments, changing the way subjects were
given the items, and modifying the procedures for eliciting choices— to rule out alternative explanations,
experimentalist were able to make “endowment effects” that had been observed in the laboratory disappear.
Many other researchers have since replicated these results. The new data suggest that ownership alone is not
enough to change people’s expressed preferences. These results have led experimental economists and
cognitive psychologists to develop alternatives to endowment theory.

51 . The author is primarily concerned with


A. describing a theory in detail while analyzing the influence it enjoys in the field of economics
B. explaining a theory and its underlying basis to review the individuals experiments done in its support
C. discussing a theory in detail and bringing out its deficiencies that were deliberately ignored by its
proponents
D. critically evaluating a theory and its underlying basis
E. describing a theory and suggesting that it should be discarded since it has no empirical soundness

52. Which of the following is most supported by the information given in the passage?
A. The value of the endowed item increases with the passage of time.
B. The way a response is extracted out of a participant in a study could have an effect in the ultimate
outcome of that study.
C. The endowment theory lacks any sort of factual support.
D. As per the endowment theory, the sole reason that people are reluctant to transfer or sell their
endowed item is because of the workings of the loss aversion phenomenon.
E. The real value of an endowed product is normally considerably less than the value ascribed to it by people.

53. According to the passage, which of the following is NOT true?


A. The endowment theory enjoys more than limited influence
B. The ownership of an endowed product has an important part to play in whether a particular change is
perceived as a gain or loss.
C. There are holes in the empirical data that support results not in favor of the endowment theory.
D. Loss aversion is possibly not the only factor that the theory believes causes the endowment effect.
E. Some experimental economists have been influenced by the recent data that does not favor the
endowment theory.
Page: 7
“Uncle” said Luke to the old Sean “You seem to be well fed, though I know no one looks after you. Nor
have I seen you leave your residence at any time. Tell me how you manage it?”
“Because” Sean replied, “I have a good feed every night at Emperor's orchard. After dark, I go there
myself and pick out enough fruits to last a fortnight.”
Luke proposed to accompany his uncle to the orchard. Though reluctant because of Luke’s habit of
euphoric exhibition of extreme excitement, Sean agreed to take him along. At the orchard while Sean
hurriedly collected the fruits and left, Luke on the other hand at the sight of unlimited supply of fruits was
excited and lifted his voice which brought Emperor’s men immediately to his side. They seized him and
mistook him as the sole cause of damage to the orchard. Although Luke reiterated that he was a bird of
passage, they pounded him mercilessly before setting him fr ee.
54. What does "bird of passage”; mean in the context of the given passage?
A. Tliere was a bird in the orchard B. Other people stole fr om the orchard
C. Emperor knew him personally D. He did not visit orchard regularly
E. Bird’s song alerted Emperor’s men

55. How often did Sean visit the orchard?


A. Daily B. Weekly C. Once in a month D. Every day after midnight E. Never

Section 3: Analytical Ability 20x1=20 marks


Instruction: Solve each of the following problems and mark the correct answers on your Answer Sheet.
Questions 56 to 60 are based on the following information:
A. Statement (i) alone is sufficient but statement (ii) alone not sufficient to answer the question asked.
B. Statement (ii) alone is sufficient but statement (i) alone not sufficient to answer the question asked.
C. Both statement (i), (ii) together are sufficient to answer the question but neither statement is sufficient alone.
D. Each statement alone is sufficient to answer the question.
E. Statement (i) & (ii) are not sufficient to answer the question asked and additional data is needed to
answer the statements.
56. Ajar is filled with blue and green marbles. What is the probability that two green marbles are drawn in a
row, without replacement?

i. There are 36 blue marbles in the jar. ii. The probability of choosing a green marble on the first tiy is —4
57. If a certain positive integer is divided by 9, the remainder is 3. What is the remainder when the integer is
divided by 5?
i. If the integer is divided by 45, the remainder is 30 ii. The integer is divisible by 2

58. If xy?K), what is the value of


+
(x-2y) ^ ? i.
(x —
— —=2
+ y)
ii.x— 2y = 4

59. Is 48 divisible by a? i. a is divisible by 9. ii. a is multiple of 16.

60. A group consisting of N couples are going to see a movie. The seats in each row of the theater is gr eater
than 2N. If the group decides to all sit in the same row, each couple is indifferent to empty seats next to
them, and each couple insists on sitting together, how many seating arrangements are possible?
i. N = 5 ii. The group will all sit next to one another, starting with the first seat in the row.
Page: 8
61. Recent studies show that even after expensive measures have been taken to decrease the amount of
pollutants in a lake, the pollution level of the water remains high. For this reason, many senators are
skeptical about whether certain pollution control measures are worth their tremendous cost.
Which of the following, if true, casts the most serious doubt on the senator's reasoning?
A. For a lake to recover from serious pollution, not only must measures be taken to decrease pollutants
but also enough time must be allowed for these measures to work.
B. Environmentalists claim that the sources of pollution that are being eliminated are but a small
fraction of the causes of the pollution
C. Tire level of animal life in a lake remains constant throughout the implementation of pollution
control measures.
D. Scientists say that the tools to facilitate more thorough lake clean-ups are now being perfected
E. Cleaning up bodies of water, including lakes, rivers, and oceans, is a critical pari of a senators
responsibility.

62. Healthy lungs produce a natural antibiotic that protects them from infection by routinely killing harmful
bacteria on airway surfaces. People with cystic fibrosis, however, are unable to fight off such bacteria,
even though their lungs produce normal amounts of the antibiotic. The fluid on aiiway surfaces in the
lungs of people with cystic fibrosis has an abnormally high salt concentration; accordingly, scientists
hypothesize that the high salt concentration is what makes the antibiotic ineffective.
Which of the following, if true, most strongly supports the scientist's hypothesis?
A. When the salt concentration of the fluid on the aiiway surfaces of healthy people is raised artificially,
the salt concentration soon returns to normal.
B. A sample of the antibiotic was capable of killing bacteria in an environment with an unusually low
concentration of salt.
C. When hmg tissue from people with cystic fibrosis is maintained in a solution with a normal salt
concentration, the tissue can resist bacteria .
D. Many lung infections can be treated by applying synthetic antibiotics to the aiiway surfaces.
E. High salt concentrations have an antibiotic effect in many circumstances.

63. Hie percentage of students with grade point averages higher than 3.0 in the English department is higher
than the percentage in any other department. However, the percentage of students with grade point
averages higher than 3.5 is highest in the Physics department.
If the statements above are true, which of the following must also be true?
A. The percentage of students with grade point averages higher than 3.7 is higher in the Physics
department than it is in the English department.
B. The English department has the second highest percentage of students with grade point averages
higher than 3.5.
C. Some students in the English department have grade point averages higher than 3.0 but lower than 3.5.
D. The English department has more total students than does the Physics department.
E. The average grade point average of the Physics department is higher than the average grade point
average of the English department.
Page: 9
64. If A + B means A is the brother of B; A % B means A is the father of B and A x B means A is the sister
of B. Which of the following means M is the uncle ofP?
A. M % N x P B. NxP%M C. M + S%R%P D.M + K%TxP E. None of these

65. P is the mother of K; K is the sister of D; D is the father of J. How is P related to J?


A. Mother B. Grandmother C. Aunt D. Wife E. None of these

Questions 66 to 70 are based on the following information:


Three desk shelves- 1, II, and III- are being stocked with seven types of articles. Bread, Biscuits, Pizzas,
Snacks, Cake, Sweet, and Sandwich are to be placed in the shelves so that the goods belonging to any
given type are all together in one shelf and no shelf contains more than three types of goods. Tire
arrangement of the types of goods is subject to the following further constraints:
Bread and Cake must be in a shelf together.
Neither Biscuits nor Snacks can be in the same shelf as Pizzas.
Neither Biscuits nor Snacks can be in the same shelf as Sweet.
Tire Sweet must be in either shelf I or shelf II.
Each type of goods must be in some shelf or other.
66.
Shelf I Shelf II Shelf III
A Biscuits, Sweet Sandwich Bread, Pizzas, Cake
B Biscuits, Pizzas, Sweet Snacks Bread, Cake
C Pizzas, Sweet, Sandwich Bread, Biscuits, Snacks Cake
D Pizzas, Sweet, Sandwich Biscuits, Snacks Bread, Cake
E Pizzas, Sandwich Biscuits, Snacks Bread, Cake, Sweet

67. If Pizzas are in I and Sweet is in II, which of the following must be true?
A. Bread is in I. B. Bread is in II. C. Bread is in III.
D. Biscuits are in II. E. Biscuits are in III.

68. If Pizzas are in II and Sweet is in I. any of the following can be true EXCEPT:
A. Bread are in II. B. Bread are in III. C. Cakes are in I.
D. Cakes are in II. E. Sandwich are in III.

69. If Bread, Cake, and Sandwich are in I, which of the following must be true?
A. Biscuits are in II. B. Pizzas are in I. C. Pizzas are in III.
D. Snacks are in II. E. Snacks are in III.

70. If Cake is in II, which of the following is acceptable?


A. Bread is in 1 and sandwich are in II. B. Biscuits are in I and Snacks are in II.
C. Biscuits are in I and Snacks are in III. D. Snacks are in I and Sandwich are in n.
E. Sweet is in I and Pizzas are in II.

Page: 10
Questions 71 to 75 are based on the following information:
The bar graph given below shows the data of the production of paper (in lakli tons) by three different
companies X, Y and Z over the years.
Production of Paper (in lakh tones) by Three Companies X, Y and Z over the Years.

71. For which of the following years, the percentage rise/fall in production from the previous year is the
maximum for Company Y?
A. 1997 B. 1998 C. 1999 D. 2000 E. None of these

72. What is the ratio of the average production of Company X in the period 1998-2000 to the average
production of Company Y in the same period?
A. 1:1 B. 15:17 C. 23:25 D. 27:29 E. None of these

73. The average production for five years was maximum for which company?
A.X B.Y C. Z D. BothX&Z E. None of these

74. hi which year was the percentage of production of Company Z to the production of Company Y the
maximum?
A. 1996 B. 1997 C. 1998 D. 1999 E. None of these

75. What is the percentage increase in the production of Company Y from 1996 to 1999?
A. 30% B. 45% C. 50% D. 60% E. None of these

Part II: Writing Ability [25 marks]


1 . Hard work is the key to success, talent is a myth - do you agree? Defend your perspective. 15

2. Write a paragr aph on "Gig Economy" 10

Page: 11
“In the name of .ACOih, the most beneficent, the most merciful”
CAPSTONE
Contact: 01972 277 866, 016 3031 3031 Web: www.capstonebd.com facebook.com/capstonebd

Mock Test for IBA (DU) MBA Admission

INSTRUCTION: Please read carefully! Test Booklet No: 0008


1. The test consists of two parts: I. Multiple Choice Questions (MCQ) and IL Written Ability.
This part to be filled in
2. Total duration of the test will be 120 minutes (90 for Part-I &
30 minutes for Part II). The answer Sheet (For MCQs) will be by the Candidate
taken away after 90 minutes.
Admit No
3. On this Answer Sheet, write your Admit Number and Test Name
Booklet Number in the grids and your Name and Signature in
the boxes provided using a pen. Darken the corresponding
number boxes below the grids using a pen.

4. On this Test Booklet write your Admit Number and Name in the spaces provided using a pen.

5. Give all your answers in the Answer Sheet by completely darkening the lettered box, which represents
the correct answer to that the letter cannot be seen USE Pen ONLY.
® @
6. Do rough work anywhere on the rest booklet,
This part to be filled in by the invigilator
except in the spaces provided for written answers.
Do not use the answer sheet or extra paper for Please verify that
rough work. Candidate’s Name & Admit Number matches the
information on the admit card.
7. For each MCQ, 25% of assigned marks will Candidate’s appearances resembles the photograph
be deducted for each wrong answer. Each on the Admit Card.
unanswered question will earn zero marks.
Admit No
8. Candidates are required to obtain a minimum Signature of the Invigilator
acceptable mark in each section and in each
part.

9. Switch off your mobile phones.

10. Your script will not be examined if you adopt any unfair means.

11. Do not use a calculator or any other electronic devices.

12. Remain seated silently at the end until you are asked to go.

|STOP! DO NOT TURN THIS PAGE UNTIL YOU ARE ASKED TO DO SO.|
For Office Use Only
1 2 | 3
Part I: Multiple Choice Questions [70 marks]
Section 1: Language & Communication 30x1=30 marks
Questions 1 to 5 are based on the following comprehension:
Instruction: Read the following passage and answer the accompanying questions, base your answer on
what is stated or implied in the passage. Mark your answers on the Answer Sheet.

Tire term anesthesia was used by the ancient Greeks to mean absence of feelings in the sense of
boorishness - Since then, however, much has happened in the sense of evolvement to make the experience
painless. At the beginning of the 19th century, the principal aim of anesthesia was to relieve the pain of
surgery rather than to produce oblivion to it. As one can easily imagine, the use of anesthesia to make the
patient compliant while under the surgeon's knife was a vast improvement to the earlier practice of
employing a squad of heavily muscled goons to hold the patient. Finer refinements, such as enabling the
patients to actually enjoy the experience, came much later.
Tire word anesthesia was again coined in 1884 by the sage Oliver Wendell Holmes, anatomist, physician
and poet - to describe the modem practice of this science, as a “loss of feeling” caused by drugs, especially
ether, leading to a state of oblivion. Today, anesthesia is an entirely different proposition. A sense of feeling
good is as important for the patient as the relief it provides horn pain inflicted during surgery.
Patients requiring anesthesia are often terrified of ‘going under' for mainly two reasons: (a) will I feel
pain dining surgery? (b) will I ever come out of anesthesia? For a large number of patients, the second
question is by far the more crucial one. Most people, naturally, are quite willing to endure any amount of
pain if there is a guarantee that he will live to talk about it.
Today, we have come to die stage where anesthesia is primarily concerned with not only the relief of
pain din ing surgery, but also the overall safety and well-being of the patient. An ‘anesthetic trip' these days
is accompanied by almost nil mortality and more importantly, reversible morbidity. Safety of the patient is
the prime objective. Keeping that as the basic premise, anesthesia is being continuously refined and fine¬
tuned so that it actually can be far from unpleasant experience. More often tiian not, the only unpleasantness
the patient has to go through is a single needle-prick for which, unfortunately, there is still no substitute.
From there on, the patient’s experience under a skillful anesthetist is one, which he or she generally recalls
with a dreamy-eyed expression.
hi this day, an age of sophisticated monitoring devices and customized drugs, patients can be
continuously supervised so that the possibility of any life-threatening accidents during anesthesia is reduced
to the barest minimum. What is wonderfill is that these monitors are now available in the hospitals and the
better private clinics in Dhaka. The previously oft-repeated phrase ‘the operation was successful but the
patient died, does not hold water in contemporary anesthetic practice.

1 . The passage clearly states that the person who coined the word “anesthesia” was all of the following EXCEPT
A. A poet B. An anatomist C. A doctor D. A wise man E. An Englishman

2. Anesthesia is being continuously refined with which of the following objectives as a basic premise?
A. minimum pain reduction for the patient B. greatest degree of patient comfort
C. safety of the patient D. making it a pleasant experience
E. maintenance of dreamy-eyed expression
Paae: 1
3. Now-a-days the risk of serious accidents during anesthesia has been significantly reduced as a result of
A. continuous supervision using modem equipment and special dings
B. state-of-the art hospital facilities C. effective emergency treatment producers
D. an increase of quality clinics in Dhaka city E. an increase in surgeon’s skill

4. According to the passage, before the development of anesthesia, people who underwent surgery
A. consumed large amounts of alcoholic drinks prior to the operation.
B. were basically non-compliant in their general attitude C. had to bear the pain of the surgery
D. employed as squad of heavily muscled goons E. had a high degree of pain tolerance

5. The author is of the opinion that most people would be willing to suffer any amount of a pain if there is a
guarantee that they
A. will suffer less pain in future B. will not die C. will get to talk about it
D. will be financially rewarded E. will go to haven

Questions 6 to 10 are based on the following instruction:


Instruction: The underlined part of the given sentence may contain error. Select tire choice that best
replaces the underlined part keeping the original meaning intact. If you think that the original
sentence is the best choice then select A.
6. Balancing the need for sufficient food supplies with what constitutes a manageable load to cany was
undoubtedly a concern at times for many ancient hunters and gatherers, like that for modern long¬
distance backpackers.
A. like that for modern long-distance backpackers B. as that of modem long-distance backpackers
C. just as modem long-distance backpackers do D. as do modern long-distance backpackers
E. as it is for modern long-distance backpackers

7. An increase in the number of protons, when unaccompanied correspondingly by an increase in the number of
neutrons, almost always produce an unstable isotope of an element higher on the Periodic Table.
A. when unaccompanied correspondingly by an increase in the number of neutrons, almost always produce
B. when not accompanied by a corresponding increase in the number of neutrons, almost always produce
C. when that is unaccompanied correspondingly by an increase in the number of neutrons, almost
always produces
D. if not accompanied by a correspondingly increased number of neutrons, almost always produce
E. if not accompanied by a corresponding increase in the number of neutrons, almost always produces

8. She explained she both wants a large house in the city, so that she can make visits the museruns
regularly, and also a cottage in the mountains, to which she can go on free weekends.
A. both wants a large house in the city, so that she can make visits the museums regularly, and also a
cottage in the mountains, to which
B. wants both a large house in the city, for the purpose of visiting the museums regularly, and a cottage
in the mountains, where
C. wants both a large house in the city, from which she can visit the museums regularly, and a cottage
in the mountains, to which
D. wants both a large house in the city, so that she can make visits the museums regularly, and as well a
cottage in the mountains, where
E. both wants a large house in the city, from which she can visit the museums regularly, and a cottage
in the mountains, to which
Page: 2
9. The war has had a negative impact over the economy of the country.
A. over the economy B. in the economy C. in economy
D. on the economy E. to the economy

10. In the days following Hurricane Ian, the saltwater flooding in coastal areas caused the lithium-ion
batteries in electric vehicles to combust.
A. hi the days following Hurricane Ian, the saltwater flooding in coastal areas caused the lithium-ion
batteries in electric vehicles to combust.
B. hi the days follow Hurricane Ian, the saltwater flooded in coastal areas caused the lithium-ion
batteries in electric vehicles to combust.
C. hi the days following Hurricane Ian, the saltwater flooding in coastal areas causing the lithium-ion
batteries in electric vehicles to combust.
D. hi the day following Hurricane Ian, the saltwater flooding in coastal areas has caused the lithium-ion
batteries in electric vehicles to combust.
E. hi the days following Hurricane Ian, the saltwater is flooding in coastal areas caused the lithium-ion
batteries in electric vehicles to combust.

Questions 11 to 15 are based on the following instruction:


Instruction: Identify the sentence(s) that is/are showing the incorrect use of the underlined word.

11 . i. Anything that allows them to amalgam their purchasing power will be helpfill.
ii. The amalgamated Transit Union Local 1005 represents about 2,500 bus drivers, light trail operators
and other Metro Transit workers.
iii. So the final image isn’t technically one photo, but an amalgamation of many.
A. i only B. ii only C. iii only D. i and iii E. None of these

12. i. Twitter, Facebook and other social media outlets are the town criers of our petulant, righteous age.
ii. Here are 11 shades of White House astonishment, in tints of petulance, wonder, disingenuousness,
and more.
iii. Mr. Trump claimed the better temperament even as he petulantly hectored and interrupted Ms.
Clinton through most of the debate.
A. i only B. ii only C. i and ii D. iii only E. None of these

13. i. Not many people are considered a close, let alone the closest, confidant of the notoriously reticent leader.
ii. However coincidental, Bush and Obama’s comments capped periods of reticence for both men
during Trump’s tumultuous first months in office.
iii. But investors haven’t been tested like this, they might be more inreticent than in the past.
A. iii only B. ii only C. i and iii only D. ii and iii only E. None of these

14. i. hi a country that unvenerates business acumen, many admire his financial success.
ii. The U.S. industrial conglomerate’s stock is the lone original component of the iuvenerable blue-chip
index that debuted in 1896.
iii. It started with a tribute to Steve Jobs that was unnerving in the depth of its un venerable approach.
A. i only B. ii only C. i & ii D. iii only E. All of the above
Page: 3
15. i. Boseman plays Marshall as cocky and smart but with no inklingness of the giant he would become.
ii. As we get older, there are inklings, of course, that our parents were people before they decided to
enhance their lives by creating us.
iii. Tire criminal was inkling after judge had given his order.
A. i only B. ii only C. i and iii only D. i and ii only E. iii only

Questions 16 to 20 are based on the following instruction:


Instruction: Choose the appropriate preposition/ phrase.
Not only do people want to get to know those whom drey (16) , but they also strive to be like their
favorite stars. Ads on TV encourage viewers to “Be like Mike [Michael Jordan]”. On Halloween, teenage
girls can emulate their favorite pop singer by a Britney Spears costume. Although many people admittedly
would not choose a life of glamour and fame, diere is something alluring about the lifestyle, and therefore
admirers of people (17) are driven to discover personal facts about those whom they admire.
Knowing these intimate details makes a famous person with seem more (18) and thus allows die
ordinary person to feel like they have something (19) the rich and famous. The media makes a
concerted effort to give viewers a chance to become acquainted (20) public figures.

16. A. look to B. look up to C. look for D. look into E. look by


1 7. A. in the limelight B. on the limelight C. spotlight D. under the light E. None of these
1 8. A. down on earth B. down in C. downing earth D. down to earth E. down-to-earth
1 9. A. in common for B. in common with C. in common D. common with E. common for
20. A. with B. for C.by D. to E. on

Questions 21 to 25 are based on the following instruction:


Instruction: Underlined parts of the following sentences may contain error(s) in grammar, diction, usage or
idiom. Choose the underlined portion that contains such error(s). If there is no error then select E.

21. The Scarecrow and the (A) Tin Woodman stood up (B) in a comer and kept (C) quiet all night,
although of course, they cannot (D) sleep. No Error (E).

22. You can make a good (A) impression on (B) your boss and coworkers if you offer to lend a hand
with important assignments even if (C) those assignments don't fall directly tg (D) your purview.
No Error (E).

23. Milages (A), home to the Rohingya community, in the fractious state of Rakhine in western Myanmar,
are being (B) mercilessly (C), horrifically (D) burnt down. No Error (E).

24. If someone is far (A) enough away (B) from the action so that (C) the names and numbers have
disappeared, Messi will still (D) be easy to spot. No Error (E).

25. Recently, however (A) as is fitting in a saner (B) real estate market than that of previous one house
prices have been rising (C) on line (D) with personal income and other economic fundamentals in local
areas. No Error (E).
Page: 4
Questions 26 to 30 are based on the following instruction:
Instruction: Which of the following phases given against each sentence can replace the phase given in
bold in the sentence grammatically and contextually? If none of the phrase can replace the
word/ phrase given in bold in the sentence, select ‘E. None of these’ as your answer.
26. His apparent heresy is not that of the smooth talking cleric, but the statistician specializing in the field of
criminology.
I. piousness II. hearsay III. rambling
A. Ill B. I, II C. II D. II, III E. None of these

27. We were kept on tenterhooks for hows while the judges chose the winner.
I. waiting nervously II. in suspense III. kept in dark
A. I, II B. II, in C.I, III D. I E. None of these

28. Vineet pulled a long face when he was taken to task by his teacher for his carelessness.
I. showed arrogance II. looked disappointed III/ mocked him
A. I, III B. II, III C. II D. I, III E. None of these

29. Despite all the goodies that participants get, I am not going for the competition.
I. gunning for II. competing in III. taking part in
A. II B. I, II C. 11,111 D ili E. None of these

30. The Swedes are not alone in finding their language under pressure from the ubiquitous spread of English.
I. omnipresent 11. all-pervasive III. percolate
A. II B. I, III C. Ill D.I, II E. None of these

Section 2: Mathematics 25xl=25 marks


Instructions: Solve each of the following problems and mark the correct answer on your Answer Sheet.
DO NOT USE A CALCULATOR. Figures are not drawn to scale.

31. Tire salaries of A, B and C were in the ratio 6:5:7 in 2010, and in the ratio 3:4:3 in 2015. If A’s salary
increased by 25% during 2010-2015, then the percentage increase in C’s salary during this period is
closest to-
A. 7 B.8 C. 9 D. 10 E. None of these

32. If x > 2 and y > -1, then which of the following statements must be true?
A. xy>-2 B. -x<2y C. xy < -2 D. -x>2y E. None of these

33. The difference of two numbers is 2 and the difference of their squares is 16. Tire difference of their
cubes will be between:
A. 50 and 60 B. 60 and 70 C. 80 and 90 D. 90 and 100 E. 100 and 110

34. Two boats, travelling at 5 km/h and 10 km/h, head directly towards each other. They begin at a distance
of 20 km from each other. How far apart are they (in km) one minute before they collide?
A.— B.-6 C.- D. - E. None of these
12 4 3
Paae: 5
35. A ship with a faulty engine sailed for only 7 horns over a period of 2 days and covered a distance of 59 kin.
Its average speed on the first days was 5 km per horn slower than that on the second day. But it sailed 3 hours
longer on the first day than that on the second day. Wliat was its average speed on second day?
A. 7 B. 12 C. 14 D. 15 E. 16

36. How many positive 4-digit integers are divisible by 20 if the repetition of digits is not allowed?
A. 168 B. 196 C. 224 D. 288 E. None of these

37. Each circle in the diagram right has a radius of r = 6. What is the total area of the shaded regions?
A. 3471 B. 3671 C. 3971 t
*
D 40.571 E 42tt UdUUU

38. Foyez purchased 140 shirts and 250 trousers @Tk. 450 and @Tk. 550 respectively. What should be the
overall average selling price of shuts and trousers so that 40% profit is earned? (Rounded off to next integer)
A. Ik. 700 B. Tk. 710 C. Tk. 720 D. Tk. 725 E. None of these

39. A train met with an accident 120 km from station A. It completed the remaining journey at - of its
previous speed and reached 2 horns late at station B. Had the accident taken place 300 km further, it
would have been only 1 how late. What is tire speed of the train?
A. 100 kph B. 12 kph C. 60 kph D. 50 kph E. None of these

40. HCF and LCM of two numbers are 7 and 140 respectively. If the numbers are between 20 and 45, the
sum of the numbers is?
A. 63 B.77 C. 70 D. 56 E. None of these

41. Wliat is the unit's digit of the number 61024- 41024?


A. 0 B. 1 C. 2 D. 3 E. 4

42. A box contains 20 electric bulbs, out of which 4 are defective. Two balls are chosen at random from this
box. The probability that at least one of them is defective, is -
A.—
19
B.—
19
C.—
19
D. 95 — E. None of these

43. hi a survey among students, it was found that 48% preferred coffee, 54% liked tea and 64% smoked. Of
the total, 28% liked coffee and tea, 32% smoked and drank tea and 30% smoked and drank coffee. Only
6% did none of these. If the total number of students is 2000 then, the ratio of number of students who
like only coffee to the number who like only tea is-
A. 5:3 B.8:9 C. 2:3 D. 3:2 E. None of these

44. If x is not equal to 0 and xy = 1, then which of the following must be true?
I. x - 1 n. x = 1 and y = 0 III. x = 1 or y = 0
A. I only B. II only C. Ill only D. I and III only E. None of these
Paae: 6
45. If n denotes a number to the left of 0 on the number line such that the square
1 —
of n is less than 100’, then
the reciprocal of n must be-
A. Less than —10
D. Greater than 1 0

B. Between 1 and
E. None of these
——
io
C. Between — — and 0
io

46. A box contains 48 socks, some red and some black. Which of the following could be the ratio of red to
black socks except:
A. 1:1 B. 3:1 C. 5:3 D. 4:3 E. 5:1

47. How many integers between 100 and 150, inclusive, cannot be evenly divided by 3 nor 5?
A. 35 B.27 C. 25 D. 26 E. None of these

48. At a certain hospital, 75% of the interns receive fewer than 6 hours of sleep and report feeling tired
during their shifts. At the same time, 70% of the interns who receive 6 or more hours of sleep report no
feelings of tiredness. If 80% of the interns receive fewer than 6 horns of sleep, what percent of the
interns report no feelings of tiredness during their shifts?
A. 6 B. 14 C. 19 D. 20 E. None of these

49. Nafiz can build up a structure in 8 days and Asif can break it in 3 days. Nafiz has worked for 4 days and
then Asif joined to work with Nafiz for another 2 days only, hr how many days will Nafiz alone build up
the remaining part of the structure?
A. 7 B. 10 C. 9 D. 12 E. None of these

50. There are four prime numbers written in ascending order. The product of the first three is 385 and that of
the last three is 1001. The last number is:
A. 11 B. 13 C. 17 D. 19 E. 23

51 . A vessel has three pipes connected to it, two to supply liquid and one to draw liquid. The first alone can
fill the vessel in 4| hours, the second in 3 hours and the third can empty it in 11 hours. If all the pipes are
opened simultaneously when the vessel is half full, how soon will it be emptied?
A. 4.5 Ins B. 5.5 Ins C. 6.5 hrs D. 7.5 Ins E. None of these

52. Bus 1 and Bus 2 nm between cities A and B. Bus 1 leaves city A at the same time Bus 2 leaves city B,
each at a constant rate of speed. Their first meeting is 50 miles from city A. After reaching their
respective destinations and immediately tinning around, their second meeting is 30 miles from city B.
What is the distance in miles between city A and city B?
A. 90 B. 120 C. 125 D. 150 E. None of these

Paae: 7
53. A square garden is surrounded by a path of uniform width. If the path and the garden both have an area
of x, then what is the width of the path in terms of x?

A. W2 B.2Vx-V2 C. —
2 4
D. —-—
2 2
E. None of these

54. A person deposited a sum of Tk. 6000 in a bank at 5% per annum simple interest. Another person deposited
Tk. 5000 at 8% per annum compound interest. After 2 years, the difference of their interest will be?
A. Tk. 230 B. Tk. 232 C. Tk. 832 D. Tk. 600 E. None of these

55. Tayef appeared in GED for four consecutive years, but coincidentally each time his net score was 75. He
told me drat there was negative marking for eveiy wrong answer and 1 mark was allotted for every
correct answer. He has attempted all the questions every year, but certainly some answers have been
wrong due to conceptual problem. Which is not the total number of questions asked for GED in any
year, in that period?
A. 231 B. 163 C. 150 D. 123 E. None of these

Section 3: Analytical Ability 15x1=15 marks


Instruction: Solve each of the following problems and mark the correct answer on your Answer Sheet.
Questions 56 to 59 are based on the following information:
A. Statement (i) alone is sufficient but statement (ii) alone not sufficient to answer the question asked.
B. Statement (ii) alone is sufficient but statement (i) alone not sufficient to answer the question asked.
C. Botli statement (i), (ii) together are sufficient to answer the question but neither statement is sufficient alone.
D. Each statement alone is sufficient to answer die question.
E. Statement (i) & (ii) are not sufficient to answer the question asked and additional data is needed to
answer the statements.

56. Is quadrilateral ABCD a rectangle?


i. Line segments AC and BD bisect one another. ii. Angle ABC is a right angle.

57. There are two solutions, A and B of spirit and water. The ratio of spirit and water in solution A is 3:4 and
4:5 in solution B, if 28 litre of solution is taken from A and added to Solution B, what will be the final
concentration of spirit in solution B.
i. After the addition, water in solution B is 24 L more than spirit
ii. Initial amount of solution A is MOL

58. Students in a class are arranged to form groups of 4 members each. After forming the groups, 3 students
are left. If the students had been arranged in groups of 9 members each, however, 4 students would be
left. What is the total number of students in the class?
i. The number of students is a two-digit number less than 70
ii. Tire number of students is a two-digit number greater than 50
Paae: 8
59. If x and y are positive integers such that the product of x and y is prime, what is the units’ digit of 7X + 9y?
i. 24<y<32 ii. x=l

60. Not all tenured faculties are full professors. Therefore, although every faculty member in the linguistics
department has tenure, it must be the case that not all of the faculty members in the linguistics
department are full professors. The flawed pattern of reasoning exhibited by the argument above is most
similar to that exhibited by which one of the following?
A. Although all modem office towers are climate-controlled buildings, not all office buildings are climate-
controlled. Therefore, it must be the case that not all office buildings are modem office towers.
B. All municipal hospital buildings are massive, but not all municipal hospital buildings are forbidding
in appearance. Therefore, massive buildings need not present a forbidding appearance.
C. Although some buildings designed by famous architects are not well proportioned, all government
buildings are designed by famous architects. Therefore, some government buildings are not well
proportioned.
D. Not all public buildings are well designed, but some poorly designed public buildings were originally
intended for private use. Therefore, the poorly designed public buildings were all originally designed
for private use.
E. Although some cathedrals are not built of stone, every cathedral is impressive. Therefore, buildings
can be impressive even though they are not built of stone.

61 A movie publicist accompanied a new movie release with the following endorsement: This movie is a
must-see for men and women who are in or want to have a committed relationship in the Nineties. While
the leading actor in the movie has unrefined edges, he eventually becomes sensitive and caring. The
leading actress begins to understand and appreciate her lover's uniquely male characteristics without
booting him out the door. If you are a true Nineties man or woman—self-reliant, yet perceptive and
caring—you must see this movie!
If the publicist's claims about the movie are true, which one of the following is also true?
A. Men and women who are perceptive and caring are also self-reliant.
B. A man who has seen the movie but is still not perceptive and caring misrepresents himself as a true
Nineties man.
C. People who go to the movie described in the passage are more sensitive than people who go to action
thriller movies.
D. No other movie recognizes true Nineties men and women.
E. Most everyone would recognize the type of man or woman who would go to see the movie described
in the passage.

62. Statistician: A financial magazine claimed that its survey of its subscribers showed that North
Americans are more concerned about their personal finances than about politics. One question was:
“Which do you think about more: politics or the joy of earning money?” This question is clearly biased.
Also, the readers of the magazine are a self-selecting sample. Tirus, there is reason to be skeptical about
the conclusion drawn in the magazine’s survey.
Paae: 9
Each of the following, if true, would strengthen the statistician’s argument EXCEPT:
A. The credibility of the magazine has been called into question on a number of occasions.
B. Tire conclusions drawn in most magazine surveys have eventually been disproved.
C. Other surveys suggest that North Americans are just as concerned about politics as they are about
finances.
D. There is reason to be skeptical about tire results of surveys that are biased and unrepresentative.
E. Other surveys suggest that North Americans are concerned not only with politics and finances, but
also with social issues.

Questions 63 to 66 are based on the following information:


A conference organizer must select exactly three speakers to respond to special issue to be presented by
an invited speaker. The three speakers will be selected from seven volunteers, of whom four- Irfan,
Kamran, Liaquat, and Mohsin- are known to be positively to the speaker's theoretical point of view. The
other three- Shabir, Tahir, and Usman- are known to be negatively to the speaker's theoretical point of
view, hi selecting the three speakers, the conference organizer must observe the following restrictions:
At least one positively speaker and at least one negatively speaker must be among those selected.
If Irfan is selected, Tahir cannot be selected.
If either Liaquat or Mohsin is selected, the other must also be selected.
If either Kamran or Usman is selected, the other must also be selected.

63. Which of the following could be the group of speakers selected?


A. Irfan, Liaquat, and Mohsin B. hfan, Shabir, and Tahir C. Kamr an, Shabir, and Usman
D. Liaquat, Shabir, and Tahir E. Mohsin, Tahir, and Usman

64. If Irfan is selected as a speaker, which of the following must also be among those selected?
A. Liaquat B. Mohsin C. Shabir D. Tahir E. Usman

65. Which of the following is a pair of volunteers that can be selected together as speakers?
A. hfan and Liaquat B. h fan and Shabir C. Kamr an and Lraquat
D. Mohsin and Shabir E. Mohsin and Usman

66. Ure group of speakers selected must include either-


A. hfan or Shabir B. Kamran or Shabir C. Kamr an or Tahir
D. Liaquat or Mohsin E. Liaquat or Usman

Questions 67 to 69 are based on the following information:


Ure relative solidity of five materials- G, H, I, K, and L- is to be determined. One material is harder than
another if drawing an edge of the first material across a surface of the second material produces a gr aze;
other-wise, the first material is either equally hard or not as hard as the second. Tire following results
have so far been obtained:
G grazes H. I grazes K. H does not graze L.
Page: 10
67. Which of the following could be the five materials in order from the most solid to the lowest solid if no
two of them are equally hard?
A. G, I, L,K, H B. G, L, K, H, I C. I, G, H, K, L D. I, H, K, L, G E. L, G, K, I, H

68. If H gl azes K, which of the following must be true?


A. G is more solid than I B. G is more solid than L. C. I is more solid than G
D. I is more solid than L E. L is more solid than K

69. If K gl azes L, any of the following pairs of materials could be the same solidity as each other EXCEPT
A. G and I B. GandK C. G and L D. H and L E. I and L

70. A is the son of C; C and Q are sisters; Z is the mother of Q and P is the son of Z. Which of the following
statements is true?
A. P and A are cousins B. P is the maternal uncle of A C. Q is the maternal grandfather of A
D. C and P are sisters E. None of these

Part II: Writing Ability [30 marks]


1 . Prepare a write up by using the words/phrases given below and underline those words after using them.
You may use a word more than once. Do not change the form of the words. Give a suitable title to your
note. 15
Export Remittance Import Liability
Letter of credit Pressure Risk Foreign exchange

2. Explain-Personal Finance Management is key to success in life. 15

Page: 11
“In the name of .ACOih, the most beneficent, the most merciful”
CAPSTONE
Contact: 01972 277 866, 016 3031 3031 Web: www.capstonebd.com facebook.com/capstonebd

Mock Test for IBA (DU) MBA Admission

INSTRUCTION: Please read carefully! Test Booklet No: 0009


1. The test consists of two pails: I. Multiple Choice Questions (MCQ) and IL Written Ability.
This part to be filled in
2. Total duration of the test will be 120 minutes (90 for Pait-I &
30 minutes for Part II). The answer Sheet (For MCQs) will be by the Candidate
taken away after 90 minutes.
Admit No
3. On this Answer Sheet, write your Admit Number and Test Name
Booklet Number in the grids and your Name and Signature in
the boxes provided using a pen. Darken the corresponding
number boxes below the grids using a pen.

4. On this Test Booklet write your Admit Number and Name in the spaces provided using a pen.

5. Give all your answers in the Answer Sheet by completely darkening the lettered box, which represents


the correct answer to that the letter cannot be seen USE Pen ONLY.
®® ®©
6. Do rough work anywhere on the rest booklet,
This part to be filled in by the invigilator
except in the spaces provided for written answers.
Do not use the answer sheet or extra paper for Please verify that
rough work. Candidate’s Name & Admit Number matches the
information on the admit card.
7. For each MCQ, 25% of assigned marks will Candidate’s appearances resembles the photograph
be deducted for each wrong answer. Each on the Admit Card.
unanswered question will earn zero marks.
Admit No
8. Candidates are required to obtain a minimum Signature of the Invigilator
acceptable mark in each section and in each
part.

9. Switch off your mobile phones.

10. Your script will not be examined if you adopt any unfair means.

11. Do not use a calculator or any other electronic devices.

12. Remain seated silently at the end until you are asked to go.

|sTOP! DO NOT TURN THIS PAGE UNTIL YOU ARE ASKED TO DO SO.|
For Office Use Only
1 2 | 3
Part I: Multiple Choice Questions [75 marks]
Section 1: Mathematics 30x1=30 marks
Instructions: Solve each of the following problems and mark the correct answer on your Answer Sheet.
DO NOT USE A CALCULATOR. Figures are not drawn to scale.
1. At a certain supplier, a machine of type A costs Tk.20,000 and a machine of type B costs Tk. 50,000.
Each machine can be purchased by making a 20 percent down payment and repaying the remainder of
the cost and the finance charges over a period of time. If the finance charges 40 percent of the remainder
of the cost, how much less would 2 machines of type A cost than 1 machine of type B?
A. 10,000 B. 11,200 C. 12,000 D. 13,200 E. None of these

2. How many numbers between 200 and 400 meet one or both of the conditions given in the two statements below?
Statement 1: The number begins with 3 Statement 2: The number ends with 3
A. 20 B.60 C. 100 D. 110 E. 120

3. A new tower has just been built at the Verbico military hospital; the number of beds available for
patients at the hospital is now 3 times the number available before the new tower was built. Currently, |rd
of the hospital's original beds, as well as |th of the beds in the new tower, are occupied. For tire purposes
of renovating the hospital's original wing, all of the patients in the hospital's original beds must be
transferred to beds in the new tower. If patients are neither admitted nor discharged during the transfer,
what fraction of the beds in the new tower will be unoccupied once the transfer is complete?
.
A. 30—
11
B. 60—
29

17
C. 30 D. 30
TA

19
None of4-4-1these
E. AT
T'

4. Suppose you want to make lemonade. You have added excess amount of lemon juice and have 4 liters of
a solution that is 55% lemon juice. But in order to make perfect lemonade you need it to be 20% lemon
juice. How much water does he have to add to make the solution the correct strength?
A. 1.4 liters B. 1.75 liters C. 2.5 liters D. 5.4 liters E. 7 liters

5. Ayon takes a second to swim c meters at a constant rate from point P to point Q in a pool. Richard, who
is faster than Ayon, can swim the same distance in b seconds at a constant rate. If Ayon leaves point P
the same time that Richard leaves point Q, how many fewer meters will Ayon have swum than Richard
when the two swimmers pass each other?
A (a+b)
B e^
(a+b) (a—b)
p
(a+b)
E. None of these

6. hi triangle ABC, we have AC = BC = 7 and AB = 2. Suppose that D is a point on line AB such that B
lies between A and D and CD = 8. What is BD?
A. 3 B.2V3 C. 4^2 D. 5 E. None of these

7. Five digit numbers divisible by 9 are to be formed by using the digits 0, 1, 2, 3, 4, 7, 8 (without
repetition). The total number of such numbers that can be formed is?
A. 216 B.212 C. 214 D. 200 E. None of these

8. What is the remainder when (7 74 — 574) is divided by 24?


A. 0 B. 1 C. 2 D. 3 E. None of these
Page: 1
3 ... . . .
9. -of all students at Social High are in all three of the following clubs: Albanian, Bardic, and Checkmate.

- of all students are in Albanian, -8 are in Bardic and - are in Checkmate. If everyJ student is in at least one
2 4
club, what fraction of the student body is in exactly 2 clubs?
A. 8- B.-
4
C-8 D. -2 E. None of these

10. If V3 - x = Vx + 3 , Then x2 = ?
A. 1 B. 3 C. 2 - 3x D. 3x —9 E. None of these

11. Shawon has an estate worth of tk. 15 million that he will either divide equally among his 10 children or
among his 10 childr en and 5 nephews. How much more will each of his children inherit if his 5 nephews
are excluded?
A. 1.5 million B. 0.5 million C. 3.0 million D. 1.0 million E. 5.0 million

12. A perfect cube is an integer whose cube root is an integer. If p and q are perfect cubes, which of the
following will not necessarily be a perfect cube?
A. 8p B. pq C. pq + 27 D. —p E. (p — q)6
13. In a meeting 10% of the people were absent. Among the present, 20% were not interested and 30% were
sleepy. If the number of people invited for the meeting was 120, what is the number of people who were
interested?
A. 44 B.64 C. 74 D. 54 E. None of these

14. Tire quantities a, b, and c are positive and ab = What must happen to ‘c’ in order to remain true if ‘a’ is
increased by 50% and ‘b’ is decreased by 25%?
A. c decreases by 25% B. c decreases by 12.5% C. c increases by 25%
D. c increases by 12.5% E. c increases by 50%

15. A virus known to cause a deadly disease is also a rapid multiplier. It is known that it can double itself
every 30 minutes. If a container is completely full of tliis virus after 50 hours, when was the container
half empty?
A. 35 hours B. 49 horns C. 49.5 hows D. 20 hours E. None of these

16. Event X and event Y are independent, Tire probability that an event X occws is 0.5 and the probability
that an event Y occurs is |. What is the probability that event X occws but event Y does not occur?
A.-
6
B.-3 C-2 D. - 3
E. - 6

17. Tire LCM of two numbers is 45 times their HCF. If one of the numbers is 125 and the sum of HCF and
LCM is 11 50, the other number is:
A. 215 B.225 C. 220 D. 235 E. None of these

18. Tire ages of Tazul and Sadib are presently in the ratio of 5:6 respectively. Six years hence, this ratio
became 6:7 respectively. What was Sadib's age 5 years ago?
A. 25 years B. 30 years C. 31 years D. 36 years E. None of these
Page: 2
19. All employees of Company A and Company B work together on some project. Tire ratio of number of
employees in company A and company B is 5:6, and each employee of company A can do the work 3
times faster than each employee of company B. What fraction of the project is done by the employees of
company B?
5 2 2 5
A.— B- C.-5 D. - E. None of these
18 7 7

20. What is the smallest integer x, such that one-third of the integer 1 greater than x is at least 2 more than
one-fourth of the integer 1 less than x?
A. 14 B. 15 C. 16 D. 17 E. None of these

21. hr the first day Suzana paddles 10 miles downstream on a river flowing at 3 mph. On the second day she
paddles 12 miles down a river flowing at 4 mph. If she took exactly same time the both day, how fast
does she paddle in still water?
A. 4 mph B. 3.5 mph C. 3 mph D. 2.5 mph E. 2 mph

22. A bookseller procures 40 books for Tk. 3200 and sells them at a profit equal to the selling price of 8
books. What is the selling price of one dozen books in Tk., if the price of each book is same?
A. 720 B.960 C. 1200 D. 1440 E. None of these

23. A certain post office imposes a service charge of BDT 75 per order on any money order in the amount of
BDT 2500 or less, and BDT 100 per order on any money order in an amount from BDT 2501 through
BDT 70000. If Niaz purchases 3 money orders in the amounts of BDT 1825, BDT 2500, and BDT
12750, what is the total service charge for his money orders?
A. BDT 175 B. BDT 225 C. BDT 250 D. BDT 275 E. BDT 300

24. On a Saturday night, each of the rooms at a certain motel was rented for either BDT 400 or BDT 600. If
10 of the rooms that were rented for BDT 600 had instead been rented for BDT 400, then the total rent
the motel charged for that night would have been reduced by 25 percent. What was the total rent the
motel actually charged for that night?
A. BDT 6,000 B. BDT 8,000 C. BDT 10.000 D. BDT 16,000 E. BDT 24,000

25. There are 5 King cards, 6 Queen cards and 7 Jack cards in a box. Rohit draws 3 cards from it, one after
another. What is the probability of the second card drawn being a Jack, if the first card drawn is not
replaced?
A.—
153
B.—
18
C.—
153 —
D. 153 E. None of these

Questions 26 to 30 are based on the following information:


A. Statement (i) alone is sufficient but statement (ii) alone not sufficient to answer the question asked.
B. Statement (ii) alone is sufficient but statement (i) alone not sufficient to answer the question asked.
C. Both statement (i), (ii) together are sufficient to answer the question but neither statement is sufficient alone.
D. Each statement alone is sufficient to answer the question.
E. Statement (i) & (ii) are not sufficient to answer the question asked and additional data is needed to
answer the statements.
Page: 3
26. A tank containing water started to leak. Did the tank contain more than 30 gallons of water when it
started to leak? (Note: 1 gallon = 128 ounces)
i. Tire water leaked from the tank at a constant rate of 6.4 ounces per minute.
ii. Tire tank became empty less than 12 hours after it started to leak.

27. Tickets to a play cost $10 for children and S25 for adults. If 90 tickets were sold, were more adult tickets
sold than children's tickets?
i. Tire average revenue per ticket was $18 ii. The revenue from ticket sales exceeded $1600

28. Shanta has deposited money in a bank to earn interest on her savings, hi how many years, the amount x
will grow to be 9 times the original value?
i. Tlie amount x is tripled in the first 12 years
ii. Tlie amount x accumulated after 18 years of the deposit is 4x

29. Of the 60 animals on a certain farm, 2/3 are either pigs or cows. How many of the animals are cows?
i. Tlie farm has more than twice as many cows as pigs ii. Tlie farm has more than 12 pigs

. 1 a 2
30. If a and b are positive integers, is - = -? 1. -
2
< -b < -3 ii. b = 8

Section 2: Language & Communication 30x1=30 marks


Questions 31 to 35 are based on the following comprehension:
Instruction: Read the following passage and answer the accompanying questions, base your answer on
what is stated or implied in the passage. Mark your answers on the Answer Sheet.
No one can be a great thinker who does not realize that as a thinker it is her first duty to follow her
intellect to whatever conclusions it may lead. Truth gains more even by the errors of one who, with due
study and preparation, thinks for herself, than by the true opinions of those who only hold them because they
do not suffer themselves to think. Not that it is solely, or chiefly, to form great thinkers that freedom of
thinking is required. On the contrary, it is as much or even more indispensable to enable average human
beings to attain the mental stature which they are capable of. There have been, and may again be, great
individual thinkers in a general atmosphere of mental slavery. But there never has been, nor ever will be, in
that atmosphere an intellectually active people. Where any people has made a temporary approach to such a
character, it has been because the dread of heterodox speculation was for a time suspend. Where there is a
tacit convention that principles are not to be disputed; where the discussion of the greatest questions which
can occupy humanity is considered to be closed, we cannot hope to find that generally high scale of mental
activity which has made some periods of history so remarkable. Never when controversy avoided the
subjects which are large and important enough to kindle enthusiasm was the mind of a people stirred up
from its foundations and the impulse given which raised even persons of the most ordinary intellect to
something of the dignity of thinking beings.
She who knows only her own side of the case knows little of that. Her reasons may be good, and no one
may have been able to refute them. Birt if she is equally unable to refute the reasons of the opposite side; if
she does not so much as know what they are, she has no ground for preferring either opinion. Tire rational
position for her would be suspension of judgment, and unless she contents herself with that, she is either led
by authority, or adopts, like the generality of the world, the side to which she feels the most inchnation. Nor
is it enough that she should hear the arguments of adversaries from her own teacher, presented as they state
them, and accompanied by what they offer as refutations. That is not the way to do justice to the arguments,
Page: 4
or bring them into real contact with her own mind. She must be able to hear them from persons who actually
believe them; who defend them in earnest, and do their very utmost for them. She must know them in their
most plausible and persuasive form: she must feel the whole force of the difficulty which the tine view of
the subject has to encounter and dispose of; else she will never really posses herself of the portion of truth
which meets and removes that difficulty. Ninety nine in a hundred of what are called educated persons are in
this condition; even of those who can argue fluently for their opinions. Their conclusion may be true, but it
might be false for anything they know: they have never thrown themselves into the mental position of those
who think differently from them and considered what such persons may have to say; and consequently they
do not, in any proper sense of the word, know the doctrines which they themselves profess. They do not
know those parts of the doctrine which explain and justify the remainder; the considerations which show that
a fact which seemingly conflicts with another is reconcilable with it, or that, of two apparently strong
reasons, one and not the other ought to be preferred.
31 . According to the author, it is always advisable to-
A. have opinions which cannot be refuted
B. adopt the point of view to which one feels the most inclination
C. be acquainted with the arguments favoring the point of view with which one disagrees
D. suspend heterodox speculation in favor of doctrinaire approaches
E. ignore the accepted opinions of the vast majority

32. According to the author, in a great period such as the Renaissance we may expect to find-
A. acceptance of truth B. controversy over principles C. inordinate enthusiasm
D. a dread of heterodox speculation E. a suspension of judgment

33. It can be inferred from the passage that a person who knows only her own side of an issue is regarded by
the author as
A. timorous B. opinionated C. heterodox D. educated E. rational

34. According to the author, the person who holds orthodox belief without examination may be described in
all of the following ways EXCEPT as
A. enslaved by tradition B. less than frilly rational C. determined on controversy
D. having a closed mind E. unwilling to adopt new ideas

35. It can be inferred from the passage that the author would be most likely to agree with which of the
following statements?
A. A truly gr eat thinker makes no mistakes
B. Periods of intellectual achievement are periods of unorthodox reflection
C. The refutation of accepted ideas can best be provided by one’s own teachers
D. Excessive controversy prevents clear thinking
E. hr a period of mental slavery, no true intellectual thought is possible

Questions 36 & 37 are based on the following instruction:


Instruction: Tire underlined part of the given sentence may contain error. Select the choice that best
replaces the underlined part keeping the original meaning intact. If you think that the original
sentence is the best choice then select A.
36. If the room had been brighter, I would have been able to read for a while before bed time.
A. If the room was brighter B. If the room are brighter C. Had the room brighter
D. Had the room was brigliter E. Improvement Does not required
Page: 5
37. He found a wooden broken chair in the room.
A. wooden and broken chair B. broken wooden chair C. broken and wooden chair
D. wood laden chair E. Improvement Does not required

Questions 38 to 40 are based on the following instruction:


Instruction: Choose the best options to fill in the gap.
38. Discipline is the most important ingredient which connects dreams and aspirations them in reality.
A. to which of successfully achieve B. for that to successfully achieving
C. to that of successfill achievement D. to that of successfully achieving
E. for successfully achieving the

39. Tire e-govemance the accessibility of government information to citizens at any time as per their
convenience.
A. empower the citizens by improving B. empowering citizens from improving
C. empowered the citizens with improved D. empowering citizens to improving
E. empowers the citizens by improving

40. Over the last one and a half years, farmers and organizing mass gatherings.
A. has been protesting across the coruitry B. had to protest across the country
C. had been protesting over the coruitry D. has been protesting in the coruitry
E. have been protesting across the coruitry

Questions 41 to 45 are based on the following instruction:


Instruction: Fill the blanks with the appropriate words.
41. The editor reduced the introduction from ten pages to two paragraphs by all of
the unnecessary verbal flourishes that riddled its sentences.
A. florid . . . trenchant . . . expiating B. officious . . . sedulous . . . bolstering
C. immutable . . . intransigent . . . inhibiting D. tortuous . . . succinct . . . excising
E. turgid . . . concise . . . qualifying

42. Although the rigors of ballet dancing are primarily . this art is also emotionally and spiritually

A. illusory . . . taxing B. exaggerated . . . balanced C. physical . . . demanding


D. appealing . . . indulgent E. strenuous . . . dubious

43. It would be a waste of time for any reviewer to bother a book whose utter worthlessness is
to even the least discerning reader.
A. enjoying . . . doubtful B. mocking . . . figurative C. assessing . . . welcome
D. condemning . . . obvious E. ignoring . . . obnoxious

44. When two chemical compounds are combined, a effect can be achieved: the resulting
combination can be more potent than either of tire individual compounds alone.
A. synergistic B. naturalistic C. competitive D. retroactive E. neutralizing

45. Although the bystander's account of the car accident at first seemed , the police officer was
surprised, on further investigation, to find that it was .
A. dubious . . . erroneous B. incongruous . . . inconsistent C. implausible . . . correct
D. logical . . . pertinent E. probable . . . coherent
Page: 6
Questions 46 to 50 are based on the following instruction:
Instruction: Underlined parts of the following sentences may contain enor(s) in grammar, diction, usage or
idiom. Choose the underlined portion that contains such error(s). If there is no error then select E.
46. The men (A) who served (B) under sobel mocked him (C) and underpinned (D) him. No error (E).

47. The public service has a bias towards (A) the status quo and the public generally don’t like (B) the
expense of (C) radical shifts to existing policies (D). No error (E).

48. Uris emerging threat has forced (A) local fire departments to divert (B) resources away of (C)
humcane recovery to control (D) and contain these dangerous fires. No error (E).

49. Areas of low pressure across eastern parts (A) of the US have helped drag (B) cold air southwards
across (C) many central and southern parts of the country over the last (D) few days. No error (E).

50. Too many (A) people were already getting (B) serious gains from these policies by the time (C) Key
was elected for him getting rid of them (D). No error (E).

Questions 51 to 55 are based on the following comprehension:


Instruction: Read the following passage and answer the accompanying questions, base your answer on
what is stated or implied in the passage. Mark your answers on the Answer Sheet.
Despite the wave of privatization that has occurred over the past fifteen years. State Owned Enterprises
(SOEs) continue to occupy a central role in many economies in terms of value added relative to GDP,
employment, and investment. Yet the performance of SOEs has been largely disappointing with the negative
impact on macro-economic stability. Inefficient public enterprises have undermined the operation of
financial system, fuelled inflation, increased public debt while acting as an obstacle to private business.
hr advanced economies worldwide - and in virtually all economies in transition - the corporation is the
defining characteristics of the modem industrial enterprise. Tire modem corporate form has evolved over
four centuries to become the most efficient institution through which large scale commercial activity- in
essence the transformation of inputs into outputs into outputs- can be carried out. The focus of the analysis
is on how SOEs might be reformed i.e. structure, governed, operated, and financed as modem corporation;
what incentives are needed to achieve the goals of the reform program; and there may be systemic limits to
SOE reforms which do not address the role of competition, factor markets, and increased private
participation in corporate finance, management, and ownership.
51. hr the passage, what is meant by commercial activity?
A. the modern corporate form B. most efficient institution C. large scale operation
D. transformation of inputs into outputs E. characteristics of the economies in transition

52. What is meant by reform of SOE?


A. how SOEs be structured B. how SOEs be financed C. how SOEs be operated
D. how SOEs be governed E. All of these

53. What are the areas of private participation?


A. providing incentives for achieving the goals of the reform program
B. exploring options for reform
C. defining characteristics of modern industrial enterprise
D. getting involved with corporate management, ownership & finance E. None of these
Page: 7
54. Which of the following statements, according to the passage, is wrong?
A. the role of public enterprise is supportive to private business
B. SOEs still occupy a central role in many economies
C. Public enterprise is a cause of inflation
D. SOEs performance is not generally satisfactory E. None of these

55. Which is an appropriate title for the passage?


A. Efficiency through large scale commercial activity B. Employment and Investment
C. Modern Industrial Enterprise D. Reforming SOEs E. Value Addition to GDP

Questions 56 to 60 are based on the following instruction:


Instruction: hr each of the following questions, arrange the given sentences in a meaningful sequence and
thus find the correct answer from alternatives.
56. i. Books have a lot of special qualities.
ii. One habit that people must instill in them is the reading habit.
iii. It is good to read a book now or then.
iv. Urey open up a whole new world to us.
v. They can be anybody’s best friend during hours of solitary confinement.
A. i, ii, iv, iii, v B. ii, iii, i, v, iv C. ii, iv, v, i, iii D. ii, iv, i, iii, v E. i, iii, ii, v, iv

57. i. I feel that such victories are twisted.


ii. People preach us that victory knocks the door of those who wait.
iii. By the time we achieve them, they mean nothing to us.
iv. And those victories do not end up fulfilling our life.
v. Becarrse our life priorities would have changed.
A. ii, i, iii, v, iv B. ii, i, iii, iv, v C. iii, iv, v, i, ii D. iii, i, ii, v, iv E. iii, i, iv, v, ii

58. i. He felt secrue under them.


ii. So, on reaching the mid of tire jungle, he chose the shadiest tree spot to rest for the afternoon.
iii. Kiran always loved the trees.
iv. He tied his horse near a shrub by area for it to graze.
v. He cozily rested down and took a nap.
A. ii, i, iii, iv, v B. i, iv, v, ii, iii C. iii, ii, i, iv, v D. iii, iv, v, i, ii E. iii, i, ii, iv, v

59. i. But, we all helped in the first few days.


ii. Modonpur is considered as rural area.
iii. Shawon was transferred to oru office recently.
iv. Initially he was not getting adjusted to the city life.
v. Before that he was working in Modonpur' branch of our office.
A. iii, ii, v, iv, i B. ii, iii, v, i, iv C. iii, v, ii, iv, i D. iii, v, i, iv, ii E. ii, iii, v, iv, i

60. i. Hris statement applies in our life too.


ii. Blue being the color of sky and water, is the color of protection.
iii. Red is the color of blood and becomes the color of our survival.
iv. Red, green and blue are said to be the primary colors.
v. Green is the color of nature and therefore, is the color of our environment.
A. iv, i, iii, ii, v B. v, iv, ii, i, iii C. i, ii, v, iv, iii D. i, v, iv, iii, ii E. v, ii, i, iv, iii
Page: 8
Section 3: Analytical Ability 15x1=15 marks
Instruction: Solve each of the following problems and mark the correct answer on your Answer Sheet.
61. Tire fimction of government is to satisfy the genuine wants of the masses, and government cannot satisfy
those wants unless it is informed about what those wants are. Freedom of speech ensures that such
information will reach the ears of government officials. Therefore, freedom of speech is indispensable
for a healthy state.
Which one of the following, if true, would NOT undermine the conclusion of the argument?
A. People most often do not know what they genuinely want.
B. Freedom of speech tends ultimately to undermine social order, and social order is a prerequisite for
satisfying the wants of the masses.
C. Tire proper fimction of government is not to satisfy wants, but to provide equality of opportunity.
D. Freedom of speech is not sufficient for satisfying the wants of the masses: social order is necessary
as well.
E. Rulers already know what the people want.

62. We have a moral obligation not to destroy books, even if they belong to us. The reason is quite simple: If
preserved, books will almost certainly contribute to the intellectual and emotional enrichment of future
generations.
Which one of the following most accurately expresses the principle underlying the argument?
A. It is morally incumbent upon us to devote effort to performing actions that have at least some chance
of improving other people’s lives.
B. We are morally obligated to preser ve anything that past generations had preserved for our intellectual
and emotional enrichment.
C. Tire moral commitments we have to future generations supersede the moral commitments we have to
the present generation.
D. We are morally obligated not to destroy anything that will most likely enrich, either intellectually or
emotionally, our posterity.
E. Being morally obligated not to destroy something requires that we be reasonably assured that that
thing will lead to the betterment of someone we know.

63. At one sitting, a typical doughnut eater consumes 4 doughnuts containing a total of 680 calories and 40
grams of fat. Tire typical bagel eater consumes exactly one bagel, at 500 calories and one or two grams
of fat per sitting, though the addition of spreads can raise calorie and fat content to tire four-doughnut
range. Tims, as far as total calorie content is concerned, there is very little difference between what a
typical doughnut eater and a typical bagel eater each consumes at one sitting.
The argument depends on assuming which one of the following?
A. Tire calories and fat in bagels have the same health impact on bagel eaters as the calories and fat in
doughnuts have on doughnut eaters.
B. Most bagel eaters are not fully aware of the calorie and fat content of a bagel.
C. Eating bagels instead of eating doughnuts provides no real health benefit.
D. Most typical doughnut eaters are not also bagel eaters.
E. Tire typical doughnut eater does not add to doughnuts any substances that increase the total caloric
intake.
Page: 9
64. Book publishers have traditionally published a few books that they thought were of intrinsic merit even
though these books were unlikely to make a profit. Nowadays, however, fewer of these books are being
published. It seems, therefore, that publishers now, more than ever, are more interested in making money
than in publishing books of intrinsic value.
Which one of the following statements, if true, most seriously weakens the argument?
A. Book publishers have always been veiy interested in making money.
B. There has been a notable decline in the quality of books written in recent years.
C. hi the past, often books of intrinsic value would unexpectedly make a sizable profit.
D. There have always been authors unwilling to be published unless a profit is guaranteed.
E. hi recent years, profits in the book publishing industry have been declining.

65. The news media is often accused of being willing to do anything for ratings. However, recent action by a
television network indicates that the news media is sometimes guided by moral principle. This network
had discovered through polling voters on the east coast that the Republican candidate for President had
garnered enough votes to ensure victory before the polls closed on the west coast. However, the network
withheld this information until the polls on the west coast closed so that the information would not affect
the outcome of key congressional races.
Which one of the following most strengthens the argument?
A. The network had endorsed the Republican candidate for President.
B. The network expected its ratings to increase if it predicted the winner of the presidential race, and to
decrease if did not predict the winner.
C. A rival network did predict a winner of the presidential race before the polls on die west coast closed.
D. The network believed that it would receive higher ratings by not predicting the winner of the
presidential race.
E. Tire network feared that predicting the winner of the presidential race could so anger Congress that it
might enact legislation preventing all future polling outside of voting centers.

Questions 66 to 70 are based on the following information:


A map representing cities F, G, H, I, J, and K. is to be drawn. Neighboring cities cannot be the same
color on the map. Tire only cities neighboring to each other are as follows:
F, G, I & J are each neighboring to H I is neighboring to J F & G are each neighboring to K

66. Which of the following is a pair of cities that must be different in color from each other?
A. F and I B. Gand I C. GandK D. I and K E. JandK

67. If I is the same color as K, then it must be true that


A. F is the same color as J B. G is the same color as I C. I is the same color as J
D. G is a different color from any other city E. H is a different color from any other city

68. Which of the following is a pair of cities that can be the same color as each other?
A.FandG B.GandH C. H and I D. H and J E. I and J

69. Which of the following cities can be the same color as H?


A.F B. G C. I D. J E. K
Page: 10
70. If the fewest possible colors are Used and one of the cities is the only one of a certain color that city
could be
A. H, but not any of tire other cities B. K, but not any of the other cities
C. F or G, but not any of the other cities D. H or I or J, but not any of the other cities
E. H or J or K, but not any of the other cities
Questions 71 to 75 are based on the following information:
A cosmetic company provides five different products. The sales of these five products (in lakh number
of packs) during 1995 and 2000 are shown in the following bar graph.
Sales (in lakh number of packs) of five different products of Cosmetic Company during 1995 and 2000

71. The sales of lipsticks in 2000 was by what percent more than the sales of nail enamels in 2000?
(Rounded off to nearest integer)
A. 33% B. 31% C. 28% D. 22% E. None of these
72. During the period 1995-2000, the minimum rate of increase in sales is in the case of?
A. Shampoos B. Nail enamels C. Talcum powders D. Lipsticks E. None of these
73. What is the approximate ratio of the sales of nail enamels in 2000 to the sales of Talcum powders in
1995?
A. 7:2 B. 5:2 C. 4:3 D. 2:1 E. None of these
74. The sales have increased by nearly 55% from 1995 to 2000 in the case of?
A. Lipsticks B. Nail enamels C. Talcum powders D. Shampoos E. None of these
75. Tire sales of conditioners in 1995 was by what percent less than the sales of shampoos in 1995?
(Rounded off to nearest integer)
A. 57% B. 36% C. 29% D. 25% E. None of these

Part II: Writing Ability [25 marks]


1. With the rise of artificial intelligence, what policies should be in place to protect workers from job
displacement and ensure a just transition? 10
2. Write a paragaph using the following words. Give a proper tilte of the paragaph.You CAN NOT change the
formation of the following words while using. You must underline the following words when you use. 15
natural calamities experienced devastating Riverbank erosion contamination
Coastal districts precautionary modem technology Epidemic swallow
Page: 11
“In the name of .ACOih, the most beneficent, the most merciful”
CAPSTONE
Contact: 01972 277 866, 016 3031 3031 Web: www.capstonebd.com facebook.com/capstonebd

Mock Test for IBA (DU) MBA Admission

INSTRUCTION: Please read carefully! Test Booklet No: 0010


1. The test consists of two parts: I. Multiple Choice Questions (MCQ) and IL Written Ability.
This part to be filled in
2. Total duration of the test will be 120 minutes (90 for Part-I &
30 minutes for Part II). The answer Sheet (For MCQs) will be by the Candidate
taken away after 90 minutes.
Admit No
3. On this Answer Sheet, write your Admit Number and Test Name
Booklet Number in the grids and your Name and Signature in
the boxes provided using a pen. Darken the corresponding
number boxes below the grids using a pen.

4. On this Test Booklet write your Admit Number and Name in the spaces provided using a pen.

5. Give all your answers in the Answer Sheet by completely darkening the lettered box, which represents
the correct answer to that the letter cannot be seen USE Pen ONLY.
® @
6. Do rough work anywhere on the rest booklet,
This part to be filled in by the invigilator
except in the spaces provided for written answers.
Do not use the answer sheet or extra paper for Please verify that
rough work. Candidate’s Name & Admit Number matches the
information on the admit card.
7. For each MCQ, 25% of assigned marks will Candidate’s appearances resembles the photograph
be deducted for each wrong answer. Each on the Admit Card.
unanswered question will earn zero marks.
Admit No
8. Candidates are required to obtain a minimum Signature of the Invigilator
acceptable mark in each section and in each
part.

9. Switch off your mobile phones.

10. Your script will not be examined if you adopt any unfair means.

11. Do not use a calculator or any other electronic devices.

12. Remain seated silently at the end until you are asked to go.

|STOP! DO NOT TURN THIS PAGE UNTIL YOU ARE ASKED TO DO SO.|
For Office Use Only
1 2 | 3
Part I: Multiple Choice Questions [70 marks]
Section 1: Language & Communication 25x1=25 marks
Questions 1 to 3 are based on the following instruction:
Instruction: Identify the sentence(s) that is/are showing the correct use of the underlined word.

1 . i. This seemed to satisfy him, and he grew more solicitous.


ii. He solicitously helped her arrange her paper and ink before returning to the stage.
iii. Greenwood is a gleefill force of alternating solicitv and malevolence.
A. i only B.i&iionly C. iii only D. ii & iii only E. All of the above

2. i. hi Nigeria, the military viewed ISIS as a terrorist group and the authorities got it proscribed as such.
ii. The far-right organization was added to a list of proscribed groups by the home secretary last year.
iii. Evidently, these proscriptions are ignored by many passionate fans.
A. i only B. ii only C. i & ii D. ii & iii E. All of the above

3. i. Ronaldo’s finish, from an awkward, wildly obtuse angle to the right of goal, skimmed into the net.
ii. Surprisingly, for once, that obtuseness is not produced by our politics but by the evolutionary history
of our brains.
iii. If this week’s spectacle of moral obtusity isn't the breaking point, may God save our republic.
A. i only B. ii only C. i & iii D. i & ii E. All of tire above

Questions 4 to 6 are based on the following instruction:


Instruction: Each sentence below has one or two blanks, each blank indicating that something has been
omitted. Beneath the sentence are five lettered words or sets of words. Choose the word or set
of words for each blank that best fits the meaning of the sentence as a whole.
4. Even though previous reporters had lampooned the candidate throughout the campaign, he further
interviews.

A. resisted B. halted C. sidestepped D. welcomed E. dreaded

5. Soap operas and situation comedies, though given to distortion, are so derivative of contemporary
culture that they are inestimable the attitudes and values of our society in any particular decade.
A. contraventions B. antidotes of C. indices of D. prerequisites for E. determinants of

6. Perry's critics in the scientific world that many of the observations he has made during more than a
decade of research in Costa Rica have been reported as in popular magazines rather than as
carefully documented case studies in technical journals.
A. intimate hypotheses B. charge anecdotes C. applaud rumors
D. claim scholarship E. apologize fabrications
Paae: 1
Questions 7 to 10 are based on the following instruction:
Instruction: For each question, select the word that cannot be replaced for the word underlined. If no such
word is found, mark (E) in the answer sheet:
7. His sermons are often laced with invectives and profanities aimed at religious minorities and opponents.
A. Denunciation B. Diatribe C. Tirade D. Praise E. None of these

8. There he accused his former student of using "grandiloquent language” to distort and misrepresent the
record on brain fingerprinting.
A. Brief B. Pompous C. Pretentious D. Overblown E. None of these

9. People hate to be wrong and hate to admit being wrong and will engage in cognitively dissonant actions
to equivocate and justify past actions.
A. Beat around bush B. Be honest C. Mislead D. Prevaricate E. None of these

10. hr the aftermath of Rajon’s death and the concomitant suspension of legal proceedings against him,
many questions linger.
A. Consequent B. Subsequent C. Accidental D. Incidental E. None of these

Questions 11 to 13 are based on the following instruction:


Instruction: For each question, select the option that represents the most correct sentence. Choose the
conect answer to replace the underlined preposition or pairs of preposition in each of the
following sentences. If the sentence is correct, choose (E).
11. When I came through the customs at the airport, I had to pay tax about an electric knife I had in my luggage.
A. for.... In B. with .... with C. to ....on D. on.... in E. No error

12. This ticket entitles you to a free meal in our new restaurant.
A. with .... in B. for .... at C. for .... in D. to .... on E. No error

13. hi the beginning of his career he had very little success.


A. Till .... of B. With .... of C. At .... of D. At .... to E. No error

Questions 14 to 16 are based on the following instruction:


Instruction: Fill in the gaps with suitable words.
14. To the dangers and distresses to which the holy men were exposed, nine noble knights formed a
holy brotherhood to protect the pilgrims through the passes and defiles of the mountains.
A. compound B. incinerate C. allay D. fabricate E. rarefy

15. Almost all cultures throughout history have had their own creation myth, suggesting that the origin of
life is a subject of interest.
A. mundane B. imiversal C. unconventional D. blatant E. radical
Page: 2
16. Toddlers become at walking by imitating their parents and other elders.
A. malignant B. adept C. scrupulous D. professional E. exuberant

Questions 17 to 19 are based on the following instruction:


Instruction: Underlined parts of the following sentences may contain error(s) in grammar, diction, usage or
idiom. Choose the underlined portion that contains such error(s). If there is no error then select E.

1 7. The presence of strong feeling, the cause of which (A) is not fully understood, always has the effect (B)
of making us (C) Human beings (D) uneasy. No Error (E).

1 8. They are for the most part young firms led (A) by founders and their teams, bold leaders who continually
prioritized (B) new gr owth over (C) efficiencies to their (D) core businesses. No Error (E).

19. The change that enterprises need to undergo in order to (A) regain their growth (B) trajectories is
profound (C), and they (D) must start at the very top. No Error (E).

Questions 20 & 21 are based on the following instruction:


Instruction: Tire underlined part of the given sentence may contain error. Select tire choice that best
replaces the underlined part keeping the original meaning intact. If you think that the original
sentence is the best choice then select A.

20. After removing their skins, the children sliced the carrots into sticks for dipping.
A. After removing their skins, B. After they removed their skins,
C. After they had removed their skins, D. After removing the carrots’ skins,
E. After they had removed the skins from the carrots,

21. The adjacent homes were dissimilar enough to justify their radically different prices.
A. to justify their radically different prices B. to justify its radically different prices
C. to be justified by their radically different prices D. to justify their radically different price
E. to be a justification for their radically different Prices

Questions 22 to 25 are based on the following comprehension:


Instruction: Read the following passage and answer the accompanying questions, base your answer on
what is stated or implied in the passage. Mark your answers on the Answer Sheet.

A world conference on poverty is going to be held at Copenhagen in March. Ending poverty is, of
course, no mean task, for to date, any and every attempt to alleviate its terrible effects has ended in failure.
Yet as large numbers of people continue to live ‘below the international bread-line' this, in itself, is an
affront to mankind.
Page: 3
The main impediment to forming a concrete UN Plan of Action appears to circle around the rich
countries’ unwillingness to carry the burden of the Third World any longer which is apparent from the
prevalence of cutbacks in aid. Such aid fatigue can spell doom for much of the developing world, burdened
with debts they cannot repay.
As many of those people are from our own domain we have more than a passing interest in a plan for
ending poverty even if we tend to look on it with a jaundiced eye. We know that several countries have hied
many methods for dealing with this problem without making much of a dent. Of one thing we are aware,
however: any hope for even alleviating poverty just the tiniest bit involves a gr eat deal of money. For it is a
truism that money begets money and, without enough to lend to the poor, there is no way they can pull
themselves out of their respective nits. If the rich countries cannot understand the need for fresh funding or
for debt relief, poor economies with their poor people cannot find the momentum to get the ball rolling
again. Obviously there is a need for greater clarity of focus if focus if there is to be any agreement on the
basis of such a plan.
The many studies undertaken by many international agencies serve to focus on the different aspects of
poverty by ignoring an important fundamental: those living the poverty are not just statistics but living,
breathing human beings with same hopes and aspirations as anybody but who, for an accident of birth, are
destined to live out their lives in penury if no truly pragmatic plan is forthcoming.
We who live in the Third World need no lessons on the effect of poverty on people and, what is more
pertinent to the issue, on childr en bom into poor environments. We are surrounded by it on all sides and we
are saddened, for we do not have the means to prevent it even should we know the answer. However, the
end result of inaction by the international community will be to see the world slowly being overrun by
people replete with diseases which will as inevitably spread across the globe. Therefore the need for
reaching a consensus on this important issue now before the United Nations is pressing. It would be well for
all of us to remember that a partial attack on poverty will be rendered useless if all that is achieved is
improved health, better education but no opportunity to work, for the creation of jobs generally lag behind,
hi other words, when it all comes down to it, the bottom line is the ability to sell skills in the labor market at
rates which afford the seller a guaranteed minimum living wage.

22. A suitable title for the passage would be


A. The UN Plan to give foreign aid B. The UN Plan to improve skill
C. The UN Plan to end poverty D. The UN Plan to hold conference
E. The UN Plan to improve health.

23. For alleviating poverty even to a small extent, we need


A. foreign technical assistance B. enough money to lend to the poor
C. schools, colleges and hospitals D. enough food supply
E. distribution of income globally

Paae: 4
24. Studies of many international agencies on poverty do not
A. give sufficient data B. cover all aspects
C. relate to government efforts D. look into the human aspects
E. ignore fimd constraints

25. If developed countries ignore poverty in the 3rd World, eventually


A. there will be global impact B. the Second World will come up
C. the First World will develop more D. there will be World War
E. each one will stand alone

Section 2: Mathematics 30x1=30 marks


Instructions: Solve each of the following problems and mark the correct answer on your Answer Sheet.
DO NOT USE A CALCULATOR. Figures are not drawn to scale.

26. Pulak shoot 6 penalties to Jumman’s 4 penalties but Pulak scores only once in 4 shoots while Jumman
scores once in 3 shoots. When Jumman has missed 32 times, Pulak has scored:
A. 30 goals B. 18 goals C. 72 goals D. 36 goals E. can’t be determined

27. A necklace is formed by stringing 133 colored beads on a thin wire in the following order: red, orange,
yellow, green, blue, indigo, violet; red, orange, yellow, green, blue, indigo, violet. If this pattern
continues, what will be the color of the 101st bead on the string?
A. Orange B. Yellow C. Green D. Blue E. hr digo

28. Alvie gives 90tk plus one towel as salary to his servant for one year. Tire servant leaves after 9 months
and receives 65tk and the towel. Find the price of the towel.
A. 16 B.20 C. 10 D. 8 E. 15

29. If a + b < 0 and a + 2b = 3, then which of the following must be true?


A. a >4 B. a < 4— C. a —
< 2 D. 2 <a <4 E. None of these

30. hr the figure above, a square is inscribed in a circle, if the area of shaded region is 5n, what is the area of
the square?

Paae: 5
31 . 400 bananas were divided equally among a certain number of students. If there were 20 more students
each would have received one banana less. What is the number of students?
A. 125 B. 100 C. 90 D. 80 E. None of these

32. hr a global competition, students from five top business schools of the world have assembled in IBA,
DU. From the five schools the students have come to represent are 42, 60, 210, 90 and 84. What is the
minimum number of rooms that would be required to accommodate so that each room has the same
number of occupants and occupants are all from the same schools?
A. 44 B.62 C. 81 D. 96 E. 56

33. A man takes 2.2 times as long to row a distance upstream as to row the same distance downstr eam. If he
can row 55 km downstream in 2 hour 30 minutes, what is the speed of the boat in the still water?
A. 40 kph B. 8 kph C. 16 kph D. 24 kph E. None of these

34. Tire C.P. of two watches taken together is Tk. 840. If by selling one at a profit of 16% and the other at a
loss of 12% , there is no loss or gain in the whole transaction, then the C.P. (Tk.) of the two watches are
respectively.
A. 360 & 480 B. 480 & 360 C. 380 & 460 D. 400 & 440 E. None of these

35. hi an innings of a cricket match, three players A, B and C scored a total of 361 inns. If the ratio of the
number of mns scored by A to that scored by B and also number of inns scored by B to that scored by C
be 3 : 2, the number of mns scored by A was -
A. 161 B. 171 C. 181 D. 185 E. None of these

36. The degree measure of each of the three angles of a triangle is an integer. Which of the following
CANNOT be the ratio of their measures?
A. 2:3:4 B. 3:4:5 C. 4:5:6 D. 5:6:7 E. 6:7:8

37. hi two alloys A and B the ratio of Zinc and Tin is 5:2 and 3:4 respectively. 7 kg of the alloy A and 21 kg
of the alloy B are mixed to form a new alloy. What will be the ratio of Zinc and Tin tin the new alloy?
A. 2:1 B. 1:2 C. 2:3 D. 1:1 E. None of these

38. hi a primary school the average weight of male students is 65.9 kg and the average weight of female
students is 57 kg. If the average weight of all the students (both male and female) is 60.3 kg and the
number of male students in the school is 66, what is the number of female students in the school?
A. 162 B. 168 C. 180 D. 112 E. None of these

Paae: 6
39. What sum of money will amount to Tk. 520 in 5 years and to Tk. 568 in 7 years at simple interest?
A. Tk. 400 B. Tk. 120 C. Tk. 5 10 D. Tk. 220 E. None of these

40. The circumference of the front wheel of a cart is 30 ft long and that of the back wheel is 36 ft long. What
is the distance traveled by the cart, when the front wheel has done five more revolutions than the real'

wheel?
A. 20 ft B. 25 ft C. 750 ft D. 900 ft E. None of these

41. If a < b < 0, which of the following must be true?


A. a2<b2 B.b-10<a C.b + a>a D. ab<b2 E. None of these

42. Tina can do as much work in 2 days as Leena can do in 3 days and Leena can do as much work in 4 days
as Meena can do in 5 days. If all three work together, they take 20 days to finish a work. How long
would Leena take to finish the work if she works alone?
A. 50 days B. 60 days C. 66 days D. 72 days E. None of these

43. Coins are to be put into 7 pockets so that each pocket contains at least one coin. At most 3 of die pockets
are to contain die same number of coins and no two of the remaining pockets are to contain an equal
number of coins. What is the least possible number of coins needed for the pockets?
A 7 B 13 C 17 D 22 E. None of these

44. Rann bought 30 kg of wheat at the rate of Tk. 9.50 per kg and 40 kg of wheat at the rate of Tk. 8.50 per
kg and mixed them. She sold the mixture at the rate of Tk. 8.90 per kg. Her total profit or loss in the
transaction was -
A. Tk. 2 loss B. Tk. 2 profit C. Tk. 7 loss D. Tk. 7 profit E. None of these

45. A man walks a certain distance in certain time, if he had gone 3 km per hour faster, he would have taken
1 hour less than die scheduled time. If he had gone 2 km per hour slower, he would have taken one hour
longer on the road. The distance (in km) is:
A. 60 B.45 C. 65 D. 80 E. None of these

46. How many numbers lie between 11 and 1111 which when divided by 9 leave a remainder of 6 and when
divided by 21 leave a remainder of 12?
A. 16 B. 17 C. 18 D. 19 E. None of these

47. Of the 1 80 shidents in a school, at least 45% attended the prom night and at least 40% took part in the
debating session. What is the maximum number of shidents who would have neither attended die prom
night nor the debating session?
A. 27 B.81 C. 90 D. 99 E. None of these
Paae: 7
48. hi a certain store, the profit is 250% of the cost. If the cost increases by 50% but the selling price
remains constant, approximately what percentage of the selling price is the profit?
A. 80% B.70% C.40% D. 25% E. 58%

49. The letters I, S, M, E, S and are rearranged to form the word 'MESSI'. Find its probability:

A.—
54
B.—
24
C —
102
D. —
76
E. None of these

50. P, Q and R together earn tk. 1620 in 9 days. P and R can earn 600tk in 5 days. Q and R in 7 days can
earn 91 Otk. How much amount does R can earn per day?
A. 65 B. 70 C. 75 D. 80 E. 85

51 . hi the figure given below, ABC is a right angled, isosceles triangle. If BC 1 1 DE, BC = x and DE = y,
what is the area of the plot of DBCE?

A.^ 2
b
2 4
D_^y
2
E. None of these

52. A snack mix that is 20% raisins is blended with a second mix that is 30% raisins. The initial blend is 5
pounds of the first mix and 10 pound of the second. To make the final blend 22% raisins, how many
additional pounds of the first mix must be added to the initial blend?
A. 25 B.30 C. 35 D. 40 E. None of these

53. If a and b are non-zero integers, and a/b > 1 , then which of the following must be true?
A. a>b B.2a>b C. a2<b2 D. ab>b E. None of these

54. A three-digit positive integer is chosen at random. What is the probability that the product of its digits is even?
1 49 7
A.-
2
B.—
36
C.—
54
D. -8 E. None of these

2
55. If - of the air in a tank is removed with each stroke of a vacuum pump, how many str okes does it take

before less than 1% of the original amount of air in the tank remains?
A. 2 B. 3 C. 4 D. 5 E. None of these

Paae: 8
Section 3: Analytical Ability 15x1=15 marks
Instruction: Solve each of the following problems and mark the correct answer on your Answer Sheet.
Questions 56 to 58 are based on the following information:
A. Statement (i) alone is sufficient but statement (ii) alone not sufficient to answer the question asked.
B. Statement (ii) alone is sufficient but statement (i) alone not sufficient to answer the question asked.
C. Both statement (i), (ii) together are sufficient to answer the question but neither statement is sufficient alone.
D. Each statement alone is sufficient to answer the question.
E. Statement (i) & (ii) are not sufficient to answer the question asked and additional data is needed to
answer the statements.
56. If x and y are integers, is y divisible by 3?
i. y = x2 + x + 2
ii. y = 3x + 2

57. Is the average of n consecutive integers equal to 1 ?


i. n is even
ii. if S is the sum of the n consecutive integers, then 0 < S < n

58. What is the perimeter of a certain right triangle?


i. The length of hypotenuse is 4
ii. The area of triangle is 4.5

59. The brains of identical twins are genetically identical. When only one of a pair of identical twins is a
schizophrenic, certain areas of the affected twin's brain are smaller than corresponding areas in the brain
of the imaffected twin. No such differences are found when neither twin is schizophrenic. Therefore, this
discovery provides definitive evidence that schizophrenia is caused by damage to the physical structure
of the brain.
If the statements on which the conclusion above is based are all true, each of the following could be
true EXCEPT:
A. People who lack a genetic susceptibility for tire disease will not develop schizophrenia.
B. Medications can control most of the symptoms of schizopluenia in most patients but will never be
able to cure it.
C. The brains of schizophrenics share many of the characteristics found in those of people without the
disorder.
D. It will eventually be possible to determine whether or not someone will develop schizopluenia on the
basis of genetic information alone.
E. Brain abnormalities associated with schizopluenia are the result of childhood viral infections that
inhibit the development of brain cells.

Paae: 9
60. Teacher: Journalists who conceal the identity of the sources they quote stake their professional reputations
on what may be called the logic of anecdotes. This is so because the statements reported by such
journalists are dissociated from the precise circumstances in which they were made and thus will
be accepted for publication only if the statements are high in plausibility or originality or interest
to a given audience - precisely die properties of a good anecdote.
Student: But what you are saying, then, is that the journalist need not bother widi sources in the first
place. Surely, any reasonably resourceful journalist can invent plausible, original, or
interesting stories faster than they can be obtained from unidentified sources.
The student's response contains which one of the following reasoning flaws?
A. confiising a marginal journalistic practice with the primary work done by journalists
B. ignoring the possibility diat the teacher regards as a prerequisite for the publication of an unattributed
statement that the statement have actually been made
C. confiising the characteristics of reported statements with the characteristics of the situations in which
the statements were made
D. judging the merits of the teacher's position solely by the most extreme case to which the position
applies
E. falsely concluding that if three criteria, met jointly, assure an outcome, then each criterion, met
individually, also assures that outcome

Questions 61 to 65 are based on the following information:


A company president wishes to select four members of a marketing-finance committee as representatives
to meet with the company’s board of trustees. The marketing-finance committee consists of exactly four
marketing members— J, K, L, and M—and four finances—W, X, Y, and Z. The president can select any
of the committee members as representatives as long as he observes the following restrictions:
The group of four representatives must consist of exactly two marketing members and two finances.
Either J or K must be one of the representatives, but J and K cannot both be representatives.
If W is a representative, L must also be a representative.
If Y is a representative, K cannot be a representative.

61. If W is a representative, which of the following CANNOT also be a representative?


A.L B.M C.X D. Y E. Z

62. If neither X nor Z is a representative, which of the following is the pair of marketing members who must
be representatives?
A. J and K B.JandL C.JandM D. KandL E.KandM

63. If K is representative, which of the following can be the other three representatives?
A.JXZ B.LMW C.LWX D. LXY E.MWZ

64. If K, M, and X are representatives, which of the following must also be a representative?
A. J B.L C.W D. Y E.Z
Page: 10
65. If Y is a representative and L is not a representative, the group of representatives would be completely
determined if it were also true that
A. J is a representative B. M is a representative C. K is not a representative
D. W is not a representative E. Z is not a representative

Questions 66 to 70 are based on the following information:


hr order to solve a robbery that has taken place at the Mehmood's factory, Inspector Rehan has been
instructed to search exactly seven locations- F, K, L, G, H, M and N. Locations F, K, and L are located
ground floor, whereas locations G, H, M, and N are located first floor. During the first visit to search for
evidence, Inspector Rehan has time to search exactly three locations.
This search must be conducted according to the following conditions:
The three locations searched must be neither all first floor nor all ground floor.
If the H is searched, then the F must also be searched.
If the M is not searched, then the G cannot be searched.
K and the L cannot both be searched.
The three locations searched must include the L or the N, or both.

66. Which of the following is a selection of locations that conforms to the conditions for Inspector Rehan
first visit?
A.F,G, H B.F, H, L C. F, K, L D. G, H, K E. G, M,N

67. During the fir st visit, if the K is searched, which of the following must also be searched?
A.F B.G C. H D.M E.N

68. During the first visit, if the M is searched, which of the following is a pair of locations that can both also
be searched?
A. FandH B.GandK C. HandL D.KandL E.KandN

69. During the first visit, if the G is searched, which of the following must also be searched?
A.F B.H C. K D.L E.N

70. During the first visit, if the K is the only downstairs location searched, which of the following must also
be searched?
A.MandN B.HandN C. HandM D. GandN E. GandM

Part II: Writing Ability [30 marks]


1. Write an Essay on "Cyber Crime in BD" 15
2. Write a paragraph on “Road Accident”. 15

Page: 11

You might also like